test series: august, 2017 mock test paper 1 final course ... · the company incurred losses of rs....

170
1 Test Series: August, 2017 MOCK TEST PAPER – 1 FINAL COURSE: GROUP – I PAPER – 1: FINANCIAL REPORTING Question No. 1 is compulsory. Attempt any five questions from the remaining six questions. Working notes should form part of the answer. Wherever necessary, suitable assumption(s) may be made by the candidates. 1. (a) A Ltd. leased a machinery to B Ltd. on the following terms: (` in Lakhs) Fair value of the machinery 20.00 Lease term 5 years Lease Rental per annum 5.00 Guaranteed Residual value 1.00 Expected Residual value 2.00 Internal Rate of Return 15% Depreciation is provided on straight line method @ 10% per annum. Ascertain unearned financial income. (b) ABC Ltd. is installing a new plant at its production facility. It has incurred these costs: 1. Cost of the plant (cost per supplier’s invoice plus taxes) ` 25,00,000 2. Initial delivery and handling costs ` 2,00,000 3. Cost of site preparation ` 6,00,000 4. Consultants used for advice on the acquisition of the plant ` 7,00,000 5. Interest charges paid to supplier of plant for deferred credit ` 2,00,000 6. Estimated dismantling costs to be incurred after 7 years ` 3,00,000 7. Operating losses before commercial production ` 4,00,000 Please advise ABC Ltd. on the costs that can be capitalized in accordance with AS 10 (Revised).

Upload: others

Post on 03-Jun-2020

5 views

Category:

Documents


0 download

TRANSCRIPT

Page 1: Test Series: August, 2017 MOCK TEST PAPER 1 FINAL COURSE ... · The company incurred losses of Rs. 40,000 and Rs. 60,000 in 2015-2016 and 2016-2017 after charging depreciation of

1

Test Series: August, 2017

MOCK TEST PAPER – 1

FINAL COURSE: GROUP – I

PAPER – 1: FINANCIAL REPORTING

Question No. 1 is compulsory.

Attempt any five questions from the remaining six questions.

Working notes should form part of the answer.

Wherever necessary, suitable assumption(s) may be made by the candidates.

1. (a) A Ltd. leased a machinery to B Ltd. on the following terms:

(` in Lakhs)

Fair value of the machinery 20.00

Lease term 5 years

Lease Rental per annum 5.00

Guaranteed Residual value 1.00

Expected Residual value 2.00

Internal Rate of Return 15%

Depreciation is provided on straight line method @ 10% per annum. Ascertain

unearned financial income.

(b) ABC Ltd. is installing a new plant at its production facility. It has incurred these

costs:

1. Cost of the plant (cost per supplier’s invoice plus taxes) ` 25,00,000

2. Initial delivery and handling costs ` 2,00,000

3. Cost of site preparation ` 6,00,000

4. Consultants used for advice on the acquisition of the plant ` 7,00,000

5. Interest charges paid to supplier of plant for deferred credit

` 2,00,000

6. Estimated dismantling costs to be incurred after 7 years ` 3,00,000

7. Operating losses before commercial production ` 4,00,000

Please advise ABC Ltd. on the costs that can be capitalized in accordance with AS

10 (Revised).

Page 2: Test Series: August, 2017 MOCK TEST PAPER 1 FINAL COURSE ... · The company incurred losses of Rs. 40,000 and Rs. 60,000 in 2015-2016 and 2016-2017 after charging depreciation of

2

(c) Prakash Ltd. purchased assets costing Rs. 5,000 lakh on 01.04.2016 payable in

foreign currency (US$) on 05.04.2017. Exchange rate of 1 US $ = Rs. 50.00 and

Rs. 54.98 as on 01.04.2016 and 31.03.2017 respectively.

The company also obtained a soft loan of US $ 1 lakh on 01.04.2016 payable in

three annual equal instalments. First instalment was due on 01.05.2017.

You are required to state, how these transactions would be accounted for in the

books of accounts ending 31st March, 2017 assuming that the company has availed

the option under para 46A of AS 11.

(d) V Ltd. received a specific grant of Rs. 30 lakhs for acquiring the plant of Rs. 150

lakhs during 2013-2014 having useful life of 10 years. The grant received was

credited to deferred income in the balance sheet. During 2016-2017, due to non-

compliance of conditions laid down for the grant, the company had to refund the

whole grant to the Government. Balance in the deferred income on that date was

Rs. 21 lakhs and written down value of plant was Rs. 105 lakhs.

(i) What should be the treatment of the refund of the grant and the effect on cost

of the fixed asset and the amount of depreciation to be charged during the

year 2016-2017 in profit and loss account?

(ii) What should be the treatment of the refund, if grant was deducted from the

cost of the plant during 2013-2014 assuming plant account showed the

balance of Rs. 84 lakhs as on 1.4.2016? (4 x 5 = 20 Marks)

2. S Ltd. carried on manufacturing business. Its products were sold to wholesalers and the

company had its own retail shop. B Pvt. Ltd. carried on similar manufacturing business,

but all goods produced were sold through the company’s own retail shops.

The summarised balance sheets of the two companies as at 31st March, 2017 were as

follows:

S Ltd.

Rs.

B Pvt. Ltd.

Rs.

S Ltd. Rs.

B Pvt. Ltd.

Rs.

Share Capital Fixed Assets :

Authorised Equity Shares of Rs. 10

40,00,000

6,00,000

Freehold Properties at cost

10,00,000

2,50,000

Issued & fully paid up

25,00,000

6,00,000

Plant & Machinery at cost less depreciation

13,00,000

1,00,000

P & L A/c 3,40,000 90,000 23,00,000 3,50,000

Trade payables 4,20,000 70,000 Current Assets :

Inventory 4,80,000 1,20,000

Page 3: Test Series: August, 2017 MOCK TEST PAPER 1 FINAL COURSE ... · The company incurred losses of Rs. 40,000 and Rs. 60,000 in 2015-2016 and 2016-2017 after charging depreciation of

3

Trade receivables 2,30,000 80,000

Bank 2,50,000 2,10,000

32,60,000 7,60,000 32,60,000 7,60,000

The original cost of Plant and Machinery was:

S Ltd. Rs. 26,00,000

B Pvt. Ltd. Rs. 2,00,000

The following arrangements were made and carried out on April 1, 2017:

(1) S Ltd. purchased from the shareholders of B Pvt. Ltd. all the issued shares

@ Rs. 14 per share.

(2) The shareholders of B Pvt. Ltd. took over one of the freehold properties of B Pvt.

Ltd. for Rs. 60,000, at the book value of the same. It was agreed that the amount

should be set off against the amount due to them under (1) above and the balance

due to them to be satisfied by the issue of an appropriate number of equity shares

in S Ltd. at Rs. 19.50 per share.

The necessary transfer in regard to the setting off the price of the property taken

over by the shareholders against the amount due to them from S Ltd. were made in

the books of the two companies.

(3) All manufacturing was to be carried on by S Ltd. and all retail business is to be

carried on by B Pvt. Ltd. in this connection.

(i) S Ltd. purchased the whole of B Pvt. Ltd.’s plant and machinery for

Rs. 1,50,000 and certain of their free-hold property (cost Rs. 1,00,000) at

Rs. 1,20,000.

(ii) B Pvt. Ltd. purchased S Ltd.’s. freehold retail shop buildings (cost to S Ltd.

Rs. 75,000) at Rs. 60,000 and took over the retail inventory at Rs. 80,000 at

the book value.

(4) S Ltd. drew a cheque in favour of B Pvt. Ltd. for the net amount due, taking into

account all the matters mentioned above.

(5) Immediately after the transfer of shares in (1) above, B Pvt. Ltd. declared and paid a

dividend of Rs. 60,000 (ignore Dividend Distribution Tax).

You are required to prepare the Balance Sheets of S Ltd. and B Pvt. Ltd. immediately

after the completion of the above transaction. (16 Marks)

3. A Limited purchased 48,000 shares in S Limited on 31st March 2015, at 50% premium

over face value by issue of 8% Debentures at 20% premium. The Balance Sheets of

A Limited and S Limited as on 31-03-2015, i.e., on the date of purchase were as under:

Page 4: Test Series: August, 2017 MOCK TEST PAPER 1 FINAL COURSE ... · The company incurred losses of Rs. 40,000 and Rs. 60,000 in 2015-2016 and 2016-2017 after charging depreciation of

4

(in Rs.)

Liabilities A Ltd. S Ltd. Assets A Ltd. S Ltd.

Share capital of Rs. 10 each 10,50,000 6,00,000 Fixed Assets 6,50,000 2,00,000

General Reserve 1,20,000 40,000 Inventory in Trade 3,00,000 1,80,000

Profit and Loss A/c 80,000 - Trade receivables 3,40,000 2,10,000

Trade payables 1,00,000 60,000 Cash in hand 60,000 30,000

Profit and Loss A/c - 80,000

13,50,000 7,00,000 13,50,000 7,00,000

(a) Particulars of A Limited:

(i) Profits made: 2015-2016 Rs. 1,60,000

2016-2017 Rs. 2,00,000

(ii) The above profit was made after charging depreciation of Rs. 60,000 and

Rs. 40,000 respectively.

(iii) Out of profit shown above, every year Rs. 20,000 had been transferred to

General Reserve.

(iv) 10% Dividend had been paid in both the years.

(v) It has been decided to write down investment to face value of shares in

10 years and to provide for share of loss to subsidiary.

(b) Particulars of S Limited:

The company incurred losses of Rs. 40,000 and Rs. 60,000 in 2015-2016 and 2016-2017 after charging depreciation of 10% p.a. on the book value of Fixed Assets as on 01-04-2015.

Prepare Consolidated Balance Sheet of A Limited and its subsidiary as at 31 st March,

2017 as per requirement of Schedule III. (16 Marks)

4. (a) Given below is the summarised Profit and Loss Account of L Ltd.:

Summarised Profit and Loss Account

for the year ended 31st March, 2017

Notes Amount

(Rs. ’000)

Income

Sales 1 28,525

Other Income 756

29,281

Expenditure

Operating cost 2 25,658

Page 5: Test Series: August, 2017 MOCK TEST PAPER 1 FINAL COURSE ... · The company incurred losses of Rs. 40,000 and Rs. 60,000 in 2015-2016 and 2016-2017 after charging depreciation of

5

Excise duty 1,718

Interest on Bank overdraft 3 93

Interest on 10% Debentures 1,157

28,626

Profit before Depreciation 655

Less: Depreciation (255)

Profit before tax 400

Provision for tax 4 (275)

Profit after tax 125

Less: Transfer to Fixed Asset Replacement Reserve (25)

100

Less: Dividend paid (45)

Retained profit 55

Notes:

1. This represents the invoice value of goods supplied after deducting discounts,

returns and sales tax.

2. Operating cost includes Rs. (’000) 10,247 as wages, salaries and other

benefits to employees.

3. The bank overdraft is treated as a temporary source of finance.

4. The charge for taxation includes a transfer of Rs. (’000) 45 to the credit of

deferred tax account.

You are required to:

(i) Prepare a value added statement for the year ended 31st March, 2017.

(ii) Reconcile total value added with profit before taxation.

(b) J Ltd. furnishes the following information from which you are required to calculate

the prevailing Economic Value Added of the company and also explain the reason

for the difference, if any, between the EVA as calculated by you and the MVA

(Market Value Added) amounting to Rs. 14005 crores.

Common shares of Rs. 1,000 face value 1,58,200 units

12% Debentures Rs. 10 face value 50,00,000 units

Current tax rate 30%

Financial Leverage 1.1 times

Securities Premium Account (Rupees in lakhs) 155

Free Reserves (Rupees in lakhs) 154

Capital Reserve (Rupees in lakhs) 109

Page 6: Test Series: August, 2017 MOCK TEST PAPER 1 FINAL COURSE ... · The company incurred losses of Rs. 40,000 and Rs. 60,000 in 2015-2016 and 2016-2017 after charging depreciation of

6

It is a prevailing practice for companies in the industry to which Jindal Ltd. belongs to

pay at least a dividend of 15% p.a. to its common shareholders. (8 + 8 = 16 Marks)

5. (a) On 1st April, 2016, a company offered 100 shares to each of its 500 employees at

Rs. 50 per share. The employees are given a month to decide whether or not to

accept the offer. The shares issued under the plan (ESPP) shall be subject to lock -

in on transfers for three years from grant date. The market price of shares of the

company on the grant date is Rs. 60 per share. Due to post-vesting restrictions on

transfer, the fair value of shares issued under the plan is estimated at Rs. 56 per

share.

On 30th April, 2016, 400 employees accepted the offer and paid Rs. 50 per share

purchased. Nominal value of each share is Rs. 10.

Record the issue of shares in the books of the company under the aforesaid plan.

(b) Let us say on 30th March 2017 an entity enters into an agreement to purchase a

Financial Asset for Rs. 200 which is the Fair Value on that date.

On Balance Sheet date i.e. 31/3/2017 the Fair Value is Rs. 204 and on Settlement

date i.e. 2/4/2017 Fair Value is Rs. 206.

Pass necessary Journal entries on trade date and settlement date when the asset

acquired is measured at FVTOCI. (6 + 10 = 16 Marks)

6. The summarised Balance Sheet of D Ltd. as on 31st March, 2017 is as under:

Liabilities (Rs. in lakhs)

Assets (Rs. in lakhs)

Equity shares of Rs. 10 each 6,000 Goodwill 1,540

Reserves (including provision for taxation of Rs. 600 lakhs)

2,000

Premises and Land at cost Plant and Machinery Motor vehicles

800

6,000

5% Debentures 4,000 (purchased on 1.10.2016) 80

Secured loans 400 Raw materials at cost 1,840

Trade Payables 600 Work-in-progress at cost 260

Profit & Loss A/c: Finished goods at cost 360

Balance from previous year 64 Trade Receivables Investment 800

Profit for the year

(after taxation) 2,200

2,264

(meant for replacement of plant and machinery)

3,200

Cash at bank and cash in hand

384

15,264 15,264

Page 7: Test Series: August, 2017 MOCK TEST PAPER 1 FINAL COURSE ... · The company incurred losses of Rs. 40,000 and Rs. 60,000 in 2015-2016 and 2016-2017 after charging depreciation of

7

1. The resale value of premises and land is Rs. 2,400 lakhs and that of plant and

machinery is Rs. 4,800 lakhs.

2. Depreciation @ 20% is applicable to motor vehicles.

3. Applicable depreciation on premises and land is 2% and that on plant and

machinery is 10%.

4. Market value of the investments is Rs. 3,000 lakhs.

5. 10% of trade receivables is bad.

6. The company also revealed that the depreciation was not charged to Profit and

Loss account and the provision for taxation already made is sufficient.

7. In a similar company the market value of equity shares of the same denomination is

Rs. 25 per share and in such company dividend is consistently paid during last

5 years @ 25%. Contrary to this, D Ltd. is having a marked upward or downward

trend in the case of dividend payment.

8. In 2011-2012 and in 2012-2013, the normal business was hampered. The profit

earned during 2011-2012 is Rs. 134 lakhs, but during 2012-2013 the company

incurred a loss of Rs. 2,610 lakhs.

Past 3 years' profits of the company were as under:

2013-2014 Rs. 938 lakhs

2014-2015 Rs. 1092 lakhs

2015-2016 Rs. 810 lakhs

The unusual negative profitability of the company during 2012-13 was due to the lock out

in the major manufacturing unit of the company which happened in the beginning of the

second quarter of the year 2011-2012 and continued till the last quarter of 2012-2013.

Value the goodwill of the company on the basis of 4 years’ purchase of the super profit.

Ignore purchase goodwill given in the question at the time of valuation of new goodwill.

(16 Marks)

7. Answer any four of the following:

(a) B Ltd. on 1-1-2017 had made an investment of Rs.600 lakhs in the equity shares of

G Ltd. of which 50% is made in the long term category and the rest as temporary

investment. The realizable value of all such investment on 31-3-2017 became

Rs. 200 lakhs as G Ltd. lost a case of copyright. How will you recognize the

reduction in financial statements for the year ended on 31-3-2017.

(b) A Ltd. entered into agreement with B Ltd. for sale of goods of Rs. 8 lakhs at a profit

of 20% on cost. The sale transaction took place on 1 st February, 2017. On the same

day B Ltd. entered into another agreement with A Ltd. to resell the same goods at

Page 8: Test Series: August, 2017 MOCK TEST PAPER 1 FINAL COURSE ... · The company incurred losses of Rs. 40,000 and Rs. 60,000 in 2015-2016 and 2016-2017 after charging depreciation of

8

Rs. 10.80 lakhs on 1st August, 2017. State the treatment of this transaction in the

financial statements of A Ltd. as on 31.03.2017. The pre-determined re-selling

price covers the holding cost of B Ltd. Give the Journal Entries as on 31.03.2017 in

the books of A Ltd.

(c) A Company has an inter-segment transfer pricing policy of charging at cost less

10%. The market prices are generally 25% above cost. Is the policy adopted by the

company correct?

(d) X Ltd., has been successful jewellers for the past 100 years and sales are against

cash only. The company diversified into apparels. A young senior executive was put

in charge of Apparels business and sales increased 5 times. One of the conditions

for sales is that dealers can return the unsold stocks within one month of the end of

season. Sales return for the year was 25% of sales. Suggest a suitable Revenue

Recognition Policy, with reference to AS 9.

(e) On 30-6-2017, L Limited incurred Rs. 6,00,000 net loss from disposal of a business

segment. Also on 31-7-2017, the company paid Rs. 1,60,000 for property taxes

assessed for the calendar year 2017. How should the above transactions be

included in determination of net income of L Limited for the six months interim

period ended on 30-9-2017? (4 x 4 = 16 Marks)

Page 9: Test Series: August, 2017 MOCK TEST PAPER 1 FINAL COURSE ... · The company incurred losses of Rs. 40,000 and Rs. 60,000 in 2015-2016 and 2016-2017 after charging depreciation of

1

Test Series : August, 2017

MOCK TEST PAPER – 1

FINAL COURSE: GROUP – I

PAPER – 1: FINANCIAL REPORTING

SUGGESTED ANSWERS/HINTS

1. (a) Computation of Unearned Finance Income

Gross investment = Minimum lease payments + Unguaranteed residual value

= (Total lease rent + Guaranteed residual value) + Unguaranteed residual

value

= [(Rs. 5,00,000 5 years) + Rs. 1,00,000] + Rs. 1,00,000 = Rs. 27,00,000

Table showing present value of (i) Minimum lease payments (MLP) and

(ii) Unguaranteed residual value (URV).

Year MLP inclusive of URV Internal rate of

return (Discount

factor 15%)

Present

Value

Rs. Rs.

1 5,00,000 .8696 4,34,800

2 5,00,000 .7561 3,78,050

3 5,00,000 .6575 3,28,750

4 5,00,000 .5718 2,85,900

5 5,00,000 .4972 2,48,600

1,00,000 .4972 49,720

(guaranteed residual value)

17,25,820 (i)

1,00,000 .4972 49,720 (ii)

(unguaranteed residual value)

(i) + (ii) 17,75,540 (b)

Unearned Finance Income = (a) – (b)

= Rs. 27,00,000 – Rs. 17,75,540 = Rs. 9,24,460

(b) According to AS 10 (Revised), these costs can be capitalized:

1. Cost of the plant Rs. 25,00,000

2. Initial delivery and handling costs Rs. 2,00,000

Page 10: Test Series: August, 2017 MOCK TEST PAPER 1 FINAL COURSE ... · The company incurred losses of Rs. 40,000 and Rs. 60,000 in 2015-2016 and 2016-2017 after charging depreciation of

2

3. Cost of site preparation Rs. 6,00,000

4. Consultants’ fees Rs. 7,00,000

5. Estimated dismantling costs to be incurred after 7 years Rs. 3,00,000

Rs. 43,00,000

Note: Interest charges paid on “Deferred credit terms” to the supplier of the

plant (not a qualifying asset) of Rs. 2,00,000 and operating losses before

commercial production amounting to Rs. 4,00,000 are not regarded as directly

attributable costs and thus cannot be capitalized. They should be written off to the

Statement of Profit and Loss in the period they are incurred.

(c) Transaction 1: Calculation of exchange difference on fixed assets

Foreign Exchange Liability = 5,000

50 = US $ 100 lakhs

Exchange Difference = US $ 100 lakhs x (Rs. 54.98 – Rs. 50) = Rs. 498 lakhs.

Loss due to exchange difference amounting Rs. 498 lakhs will be capitalised and to

be added in the carrying value of fixed assets. Depreciation on the unamortised

amount will be provided in the remaining years

Transaction 2: Soft loan exchange difference (US $ 1 lakh i.e Rs. 50 lakhs)

Value of loan 31.3.2017 → US $ 1 lakh x Rs. 54.98 = Rs. 54,98,000

AS 11 also provides that in case of liability designated as long-term foreign currency

monetary item (having a term of 12 months or more at the time of origination) the

exchange difference is to be accumulated in the Foreign Currency Monetary Item

Translation Difference (FCMITD) and should be written off over the useful life of

such long-term liability, by recognition as income or expenses in each of such

periods.

Exchange difference between reporting currency (INR) and foreign currency (USD)

as on 31.03.2017 = US$ 1.00 lakh x Rs. (54.98 – 50) = Rs. 4.98 lakh.

Loan account is to be increased to Rs. 54.98 Iakh and FCMITD account is to be

debited by Rs. 4.98 lakh. Since loan is repayable in 3 equal annual instalments,

Rs. 4.98 lakh/3 = Rs. 1.66 lakh is to be charged to Profit and Loss Account for the

year ended 31st March, 2017 and balance in FCMITD Account Rs. (4.98 lakh – 1.66

lakh) = Rs. 3.32 lakh is to be shown on the 'Equity & Liabilities' side of the Balance

Sheet as a negative figure under the head 'Reserve and Surplus' as a separate line

item.

(d) As per para 21 of AS 12, ‘Accounting for Government Grants’, “the amount

refundable in respect of a grant related to specific fixed asset should be recorded by

reducing the deferred income balance. To the extent the amount refundable

Page 11: Test Series: August, 2017 MOCK TEST PAPER 1 FINAL COURSE ... · The company incurred losses of Rs. 40,000 and Rs. 60,000 in 2015-2016 and 2016-2017 after charging depreciation of

3

exceeds any such deferred credit, the amount should be charged to profit and loss

statement.

(i) In this case the grant refunded is Rs. 30 lakhs and balance in deferred income

is Rs. 21 lakhs, Rs. 9 lakhs shall be charged to the profit and loss account for

the year 2016-2017. There will be no effect on the cost of the fixed asset and

depreciation charged will be on the same basis as charged in the earlier years.

(ii) If the grant was deducted from the cost of the plant in the year 2013-2014 then,

para 21 of AS-12 states that the amount refundable in respect of grant which

relates to specific fixed assets should be recorded by increasing the book value

of the assets, by the amount refundable. Where the book value of the asset is

increased, depreciation on the revised book value should be provided

prospectively over the residual useful life of the asset. Therefore, in this case,

the book value of the plant shall be increased by Rs. 30 lakhs. The increased

cost of Rs. 30 lakhs of the plant should be amortized over 7 years (residual life).

Depreciation charged during the year 2016-2017 shall be (84 + 30)/7 years =

Rs. 16.286 lakhs presuming the depreciation is charged on SLM.

2. Balance Sheet of S Ltd. as on 31st March, 2017

Particulars Note No. (Rs.)

I. Equity and Liabilities

(1) Shareholder's Funds

(a) Share Capital 1 29,00,000

(b) Reserves and Surplus 2 7,05,000

(2) Current Liabilities

Trade Payables 4,20,000

Total 40,25,000

II. Assets

(1) Non-current assets

(a) Fixed assets

(b) Tangible assets 4 24,95,000

(c) Non-current Investments 5 7,80,000

(2) Current assets

(a) Inventories (4,80,000 – 4,00,000) 4,00,000

(b) Trade Receivables 2,30,000

(c) Cash & Cash equivalents 1,20,000

Total 40,25,000

Page 12: Test Series: August, 2017 MOCK TEST PAPER 1 FINAL COURSE ... · The company incurred losses of Rs. 40,000 and Rs. 60,000 in 2015-2016 and 2016-2017 after charging depreciation of

4

Notes to Accounts

(Rs.) (Rs.)

1. Share Capital

2,90,000 Shares of Rs.10 each fully paid up of which 40,000 shares were issued pursuant to a contract without payment being received in cash

29,00,000

2. Reserves and surplus

Securities Premium Account 3,80,000

Profit & Loss Account (3,40,000 – 15,000) 3,25,000 7,05,000

3. Tangible Assets

Freehold Properties

As per last balance sheet 10,00,000

Addition during the year 1,20,000

11,20,000

Less: Sold during the year (75,000) 10,45,000

Plant & Machinery

As per last balance sheet 13,00,000

Addition during the year 1,50,000 14,50,000 24,95,000

4. Non-current Investment

Shares in subsidiary company 7,80,000

Working Notes :

(1) Calculation of shares to be issued by S Ltd. to the shareholders of B Pvt. Ltd. [Basis

Point (1) and (2) of the question] Rs.

60,000 shares @ Rs. 14 per share 8,40,000

Less : Value of freehold property (60,000)

Net amount due 7,80,000

No of shares issued 7,80,000 / Rs. 19.5 = 40,000 shares

Amount credited to Share Capital 4,00,000

Amount credited to Securities Premium 3,80,000

7,80,000

(2) Net Amount Payable to B Pvt. Ltd.

Particulars Amount

Assets taken over by S Ltd.

Plant & Machinery Rs. 1,50,000

Page 13: Test Series: August, 2017 MOCK TEST PAPER 1 FINAL COURSE ... · The company incurred losses of Rs. 40,000 and Rs. 60,000 in 2015-2016 and 2016-2017 after charging depreciation of

5

Free-hold Property Rs. 1,20,000

Total (A) Rs. 2,70,000

Assets sold to B Pvt. Ltd.

Freehold Retail shop building Rs. 60,000

Retail Stock Rs. 80,000

Total (B) Rs. 1,40,000

Net Payable (A-B) Rs. 1,30,000

Add: Freehold properties taken over by the shareholders of B Pvt. Ltd. (Para 2 of point 2)

Rs. 60,000

Net Amount Payable Rs. 1,90,000

(3) Rs. 15,000 loss on the sale of Building to B Pvt. Ltd., has been debited to the Profit

& Loss Account of S Ltd.

(4) Investment A/c has been credited by dividend received Rs. 60,000 out of pre-

acquisition profit from B Ltd.

(5) Bank Balance:

As given 2,50,000

Ad: Dividend received 60,000

3,10,000

Less: Paid to B Ltd. including Rs. 60,000 for assets taken

over by its erstwhile shareholders (1,90,000)

1,20,000

Balance Sheet of B Pvt. Ltd. as on 31st March, 2017

Particulars Note No. (Rs.)

I. Equity and Liabilities

(1) Shareholder's Funds

(a) Share Capital 1 6,00,000

(b) Reserves and Surplus 2 1,00,000

(2) Current Liabilities

Trade Payables 70,000

Total 7,70,000

II. Assets

(1) Non-current assets

Page 14: Test Series: August, 2017 MOCK TEST PAPER 1 FINAL COURSE ... · The company incurred losses of Rs. 40,000 and Rs. 60,000 in 2015-2016 and 2016-2017 after charging depreciation of

6

Fixed assets

Tangible assets 4 1,50,000

(2) Current assets

(a) Inventories 2,00,000

(b) Trade Receivables 80,000

(c) Cash & Cash equivalents 3,40,000

Total 7,70,000

Notes to Accounts

(Rs.) (Rs.)

1. Share Capital

60,000 Shares of Rs.10 each fully paid 6,00,000

2. Reserves and surplus

Profit & Loss Account 1,00,000

3. Tangible Assets

Freehold Properties

As per last balance sheet 2,50,000

Addition during the year 60,000

3,10,000

Less: Sold during the year (1,60,000) 1,50,000

Plant & Machinery

As per last Balance Sheet 1,00,000

Less: Sold during the year (cost) (1,00,000) - 1,50,000

Working Notes:

Rs.

(1) Profit & Loss Account (given) 90,000

Add: Profit on sale of machinery and freehold property 70,000

1,60,000

Less: Dividend paid (60,000)

1,00,000

(2) Freehold properties have been reduced by Rs. 1,00,000 transferred to S Ltd. & Rs.

60,000 taken over by the shareholders of B Pvt. Ltd.

Page 15: Test Series: August, 2017 MOCK TEST PAPER 1 FINAL COURSE ... · The company incurred losses of Rs. 40,000 and Rs. 60,000 in 2015-2016 and 2016-2017 after charging depreciation of

7

(3) Cash at Bank: Rs.

Balance as given. 2,10,000

Add: Received from S Ltd. 1,90,000

4,00,000

Less: Dividend Paid (60,000)

3,40,000

3. Consolidated Balance Sheet of A Ltd. and its subsidiary S Ltd.

as at 31st March, 2017

Particulars Note No. (Rs.)

I. Equity and Liabilities

(1) Shareholder's Funds

(a) Share Capital 1 10,50,000

(b) Reserves and Surplus 2 3,42,000

(2) Minority Interest [W.N.6(b)] 92,000

(3) Non-current Liabilities

Long-term borrowings 3 6,00,000

Total 20,84,000

II. Assets

(1) Non-current assets

(a) Fixed assets 4 7,10,000

(b) Intangible assets 5 2,24,000

(2) Net current assets 6 11,50,000

Total 20,84,000

Notes to Accounts

Rs.

1. Share Capital

Issued and Subscribed:

1,05,000 shares of Rs. 10 each fully paid up 10,50,000

2. Reserves & surplus

Securities premium 1,20,000

General Reserve 1,60,000

Page 16: Test Series: August, 2017 MOCK TEST PAPER 1 FINAL COURSE ... · The company incurred losses of Rs. 40,000 and Rs. 60,000 in 2015-2016 and 2016-2017 after charging depreciation of

8

Profit and Loss Account [W.N.6(d)] 62,000 3,42,000

3. Long Term Borrowings

6,000, 8% Debentures of Rs. 100 each 6,00,000

4. Tangible Assets

A Ltd. [W.N.3] 5,50,000

S Ltd. [W.N.3] 1,60,000 7,10,000

5. Intangible assets

Goodwill [W.N.6(c)] 2,24,000

6. Net current assets

A Ltd. [W.N.5] 8,50,000

S Ltd. [W.N.5] 3,00,000 11,50,000

Working Notes:

Percentage of shareholding of A Ltd. in S Ltd.:

48,000 shares out of 60,000 shares i.e. 48,000

10060,000

= 80%

1. Investment in S Ltd.

Rs.

Face value of shares (48,000 shares x Rs. 10) 4,80,000

Premium (50%) over face value 2,40,000

Cost of investment 7,20,000

Acquired by issue of debentures at 20% premium:

Rs.

8% Debentures 6,00,000

(Nominal value = 7,20,000/120 100)

Add: Securities premium @ 20% 1,20,000

7,20,000

Writing down of investment to face value in 10 years

2015-2016 : 1/10 2,40,000 (24,000)

2016-2017: 1/10 2,40,000 (24,000)

Investment as on 31.3.2017 6,72,000

Page 17: Test Series: August, 2017 MOCK TEST PAPER 1 FINAL COURSE ... · The company incurred losses of Rs. 40,000 and Rs. 60,000 in 2015-2016 and 2016-2017 after charging depreciation of

9

2. Balance of Profit and Loss Account for the year ended on 31st March, 2017

A Ltd. S Ltd.

Rs. Rs.

Balance as on 31.3.2015 80,000 (80,000)

Profit/(Loss)

For 2015-2016 1,60,000 (40,000)

For 2016-2017 2,00,000 (60,000)

Less: Transfer to General Reserve

2015-2016 (20,000)

2016-20174-15 (20,000)

Dividend @ 10%

2015-2016 (1,05,000)

2016-2017 (1,05,000)

Investment written off

2015-2016 (24,000)

2016-2017 (24,000)

Provision for share of loss in subsidiary

2015-2016: (40,000 80%) (32,000)

2016-2017: (60,000 80%) (48,000)

62,000 (1,80,000)

Note: In the absence of information, taxation has not been considered.

3. Fixed Assets as on 31st March, 2017

A Ltd. S Ltd.

Rs. Rs.

Fixed assets on 31.3.2015 6,50,000 2,00,000

Less: Depreciation for

2015-2016 (60,000) (20,000)

2016-2017 (40,000) (20,000)

5,50,000 1,60,000

4. General reserve of A Ltd.

Rs.

Balance as on 31.03.2015 1,20,000

Add: Transfer from Profit and loss account

in 2015-2016 20,000

Page 18: Test Series: August, 2017 MOCK TEST PAPER 1 FINAL COURSE ... · The company incurred losses of Rs. 40,000 and Rs. 60,000 in 2015-2016 and 2016-2017 after charging depreciation of

10

in 2016-2017 20,000

1,60,000

5. Separate Balance Sheets as at 31st March, 2017 to calculate net current assets

Particulars Note No.

A Ltd. S Ltd.

(Rs.) (Rs.)

I. Equity and Liabilities

(1) Shareholder's Funds

(a) Share Capital 1 10,50,000 6,00,000

(b) Reserves and Surplus 2 3,42,000 (1,40,000)

(2) Non-Current Liabilities

Long-term borrowings 3 6,00,000

Long term provision 4 80,000

Total 20,72,000 4,60,000

II. Assets

(1) Non-current assets

(a) Fixed assets

(i) Tangible assets(W.N.3) 5,50,000 1,60,000

(b) Non-current investment 6,72,000

(2) Net current assets (bal. fig.) (see note) 8,50,000 3,00,000

Total 20,72,000 4,60,000

Notes to Accounts

A Ltd. S Ltd.

Rs. Rs.

1. Share Capital

Issued and Subscribed:

1,05,000 shares of Rs. 10 each fully paid up 10,50,000

60,000 shares of Rs. 10 each fully paid up 6,00,000

2. Reserves & surplus

Securities premium 1,20,000 -

General Reserve (W.N. 4) 1,60,000 40,000

Profit and Loss Account 62,000 3,42,000 (1,80,000)

Fixed assets given in the question are assumed to be tangible fixed assets.

Page 19: Test Series: August, 2017 MOCK TEST PAPER 1 FINAL COURSE ... · The company incurred losses of Rs. 40,000 and Rs. 60,000 in 2015-2016 and 2016-2017 after charging depreciation of

11

Total 3,42,000 (1,40,000)

3. Long Term Borrowings

8% Debentures 6,00,000

4. Long term Provision

Provision for loss in subsidiary 80,000

Note: In the absence of information about the movement in individual current

assets and current liabilities, balance sheets as on 31.3.2017 have been prepared

on the basis of net current assets, which is the difference of total current assets and

total current liabilities. However, Schedule III requires separate disclosure of

current liabilities and current assets in the balance sheet.

(6) Computations for Consolidation

(a) Analysis of General reserve and Profits (Losses) of S Ltd.

Pre-acquisition Capital Profit or loss

Post-acquisition Profit or loss

Rs. Rs.

General Reserve on 31.3.2015 40,000

Profit and Loss Account on 31.3.2015 (80,000)

Profit/(Loss) for the years 2015-2016 (40,000)

2016-2017 (60,000)

(40,000) (1,00,000)

Minority Interest (20%) (8,000) (20,000)

Share of A Ltd. (80%) (32,000) (80,000)

(b) Minority Interest

Rs.

Share Capital 1,20,000

Add: Pre-acquisition capital loss (8,000)

Post-acquisition loss (20,000)

92,000

(c) Cost of Control

Rs.

Investment in S Ltd. 6,72,000

Less: Paid up value of investment 4,80,000

Capital profit/(losses) (32,000) (4,48,000)

Goodwill 2,24,000

Page 20: Test Series: August, 2017 MOCK TEST PAPER 1 FINAL COURSE ... · The company incurred losses of Rs. 40,000 and Rs. 60,000 in 2015-2016 and 2016-2017 after charging depreciation of

12

(d) Consolidated Profit and Loss Account

Rs.

Balance of Profit and Loss Account of A Ltd. 62,000

Less: Share of loss in S Ltd. (80,000)

(18,000)

Add back: Provision for loss in subsidiary (mutual transaction) (32,000 + 48,000)

80,000

62,000

(e) Long term provision for loss in subsidiary as Rs.

Provision for loss in subsidiary as shown in A Ltd. 80,000

Less: Intra company transaction (80,000)

Nil

4. (a) (i) L Ltd.

Value Added Statement

for the year ended March 31, 2017

Rs. (’000)

Rs. (’000)

%

Sales 28,525

Less: Cost of bought in material and services:

Operating cost 15,411

Excise duty 1,718

Interest on bank overdraft 93 (17,222)

Value added by manufacturing and trading activities 11,303

Add: Other income 756

Total value added 12,059

Application of value added:

To pay employees:

Wages, salaries and other benefits 10,247 84.97

To pay government:

Corporation tax 230 1.91

To pay providers of capital:

Interest on 10% Debentures 1,157

Dividend 45 1,202 9.97

To provide for the maintenance and expansion of the company:

Page 21: Test Series: August, 2017 MOCK TEST PAPER 1 FINAL COURSE ... · The company incurred losses of Rs. 40,000 and Rs. 60,000 in 2015-2016 and 2016-2017 after charging depreciation of

13

Depreciation 255

Fixed Assets Replacement Reserve 25

Deferred Tax Account 45

Retained profit 55 380 3.15

12,059 100.00

(ii) Reconciliation between Total Value Added and Profit Before Taxation

Rs. (’000) Rs. (’000)

Profit before tax 400

Add back:

Depreciation 255

Wages, salaries and other benefits 10,247

Debenture interest 1,157 11,659

Total Value Added 12,059

Notes:

(1) Deferred tax could alternatively be shown as a part of ‘To pay government’.

(2) Bank overdraft, being a temporary source of finance, has been considered as

the provision of a banking service rather than of capital. Therefore, interest on

bank overdraft has been shown by way of deduction from sales and as a part

of ‘cost of bought in material and services’.

(b) Computation of Economic Value Added

Rs. in lakhs

Profit after tax 420

Add: Interest net of tax = 60 x

100 30

100 42

Return to providers of funds 462

Less: Cost of Capital (342)

Economic Value Added 120

MVA of Rs. 14005 crore:

The MVA of Rs. 14005 crore is the difference between the current Market Value of

J Ltd. and the capital contributed by the fund providers. While EVA measures current

earning efficiency of the company, MVA takes into consideration the EVA from not

only the assets in place but also from the future projects/activities of the company.

The difference between MVA over EVA thus represents the value attributed to the

Page 22: Test Series: August, 2017 MOCK TEST PAPER 1 FINAL COURSE ... · The company incurred losses of Rs. 40,000 and Rs. 60,000 in 2015-2016 and 2016-2017 after charging depreciation of

14

future potential of the company & may change from time to time based on market

sentiments. In short the MVA is the net present value of all future EVA’s.

Working Notes:

1. Calculation of Net Profit after interest and tax

Interest on Debentures = 50,00,000 units x 10 x 12% = Rs. 60,00,000

Therefore, Financial Leverage = Pr ofit before Interest & taxes (PBIT)

PBIT less Interest

1.10 =PBIT

PBIT - Rs. 60,00,000

1.10 (PBIT – Rs. 60,00,000) = PBIT

1.10 PBIT – Rs. 66,00,000 = PBIT

1.10 PBIT – PBIT = Rs. 66,00,000

0.10 PBIT = Rs. 66,00,000

PBIT = Rs. 6,60,00,000

Profit after interest but before tax = Rs. 6,60,00,000 – Rs. 60,00,000

= Rs. 6,00,00,000

Less: Income Tax @ 30% (Rs. 1,80,00,000)

Profit After Interest & Tax Rs. 4,20,00,000

2. Calculation of Weighted Average Cost of Capital (WACC)

Rs. in lakhs Amount (Rs.)

Weight Cost% WACC%

(1) (2) (3) (4)=2x3

Equity Shareholders’ fund

Common Shares 1,582

Securities Premium 155

Free Reserves 154

Capital Reserves 109

2,000 0.80 15 12.00

Debentureholders’ fund 500 0.20 8.4 1.68

2,500 1.00 13.68

Cost of Capital = Capital Employed x WACC%

Rate of interest on debentures is taken net of tax of 30%.

Page 23: Test Series: August, 2017 MOCK TEST PAPER 1 FINAL COURSE ... · The company incurred losses of Rs. 40,000 and Rs. 60,000 in 2015-2016 and 2016-2017 after charging depreciation of

15

= Rs. 2,500 lakhs x 13.68%

= Rs. 342 lakhs

5. (a) Fair value of an ESPP = Rs. 56 – Rs. 50 = Rs. 6

Number of shares issued = 400 employees x 100 shares/employee = 40,000 shares

Fair value of ESPP which will be recognized as expenses in the year 2016-2017

= 40,000 shares x Rs. 6 = Rs. 2,40,000

Vesting period = 1 month

Expenses recognized in 2016-2017= Rs. 2,40,000

Journal Entry

Date Particulars Dr. (Rs.) Cr. (Rs.)

30.04.2016 Bank A/c (40,000 shares x Rs. 50) Dr. 20,00,000

Employees compensation expense A/c Dr. 2,40,000

To Share Capital A/c (40,000 shares x Rs. 10) 4,00,000

To Securities Premium (40,000 shares x Rs. 46) 18,40,000

(Being shares issued under ESPP @ Rs. 50)

(b) Journal Entries

Financial Asset measured at FVTOCI – on the basis of Trade Date Accounting

Dates Journal Entry Amount Amount

30/3/2017 Financial Asset Dr. 200

To Payables 200

31/3/2017 Financial Asset Dr. 4

To OCI 4

2/4/2017 Financial Asset Dr. 2

To OCI 2

Payables Dr. 200

To Cash 200

Financial Asset measured at FVTOCI – on the basis of Settlement Date

Accounting

Dates Journal Entry Amount Amount

30/3/2017 No Entry

31/3/2017 Fair Value Change Dr. 4

To OCI 4

Page 24: Test Series: August, 2017 MOCK TEST PAPER 1 FINAL COURSE ... · The company incurred losses of Rs. 40,000 and Rs. 60,000 in 2015-2016 and 2016-2017 after charging depreciation of

16

2/4/2017 Fair Value Change Dr. 2

To OCI 2

Financial Asset Dr. 206

To Cash 200

To Fair Value Change 6

6. 1. Rectification of current year’s profit i.e. 2016-2017

Profit After Tax = Rs. 2,200 lakhs

Provision for taxation = Rs. 600 lakhs

Profit Before Tax = PAT + Provision for taxation

= Rs. 2,200 lakhs + Rs. 600 lakhs = Rs. 2,800 lakhs

Rate of tax = Provision for tax

100Profit before tax

= 600

1002,800

= 21.43% (approx.)

Rs. in lakhs

Profit for the year after tax 2,200

Less: Depreciation net of tax on motor vehicles

(Rs. 80 lakhs x 20% x 6/12) x (100-21.43)%

(6.29)

Depreciation net of tax on Premises and Land

(Rs. 800 lakhs x 2%) x (100-21.43)%

(12.57)

Depreciation net of tax on Plant and Machinery

(Rs. 6,000 lakhs x 10%) x (100-21.43)%

(471.42)

Provision for doubtful receivables net of tax

(Rs. 800 lakh x 10%) x (100-21.43)%

(62.86)

Rectified profit of 2016-2017 (1,646.86)

2. Calculation of Capital Employed

(Rs. in lakhs)

(Rs. in lakhs)

Premises and land 2,400

Plant and machinery 4,800

Motor vehicles (book value less depreciation for ½ year) 72

Raw materials 1,840

Work-in-progress 260

Finished goods 360

Trade Receivables (800 x 90%) 720

Investments (market value) 3,000

Page 25: Test Series: August, 2017 MOCK TEST PAPER 1 FINAL COURSE ... · The company incurred losses of Rs. 40,000 and Rs. 60,000 in 2015-2016 and 2016-2017 after charging depreciation of

17

Cash at bank and in hand 384

13,836

Less: Liabilities:

Provision for taxation 600

5% Debentures 4,000

Secured loans 400

Trade Payables 600 (5,600)

Total capital employed on 31.3.2017 8,236

Less: Half of current year’s rectified profit (1,646.86 x 1/2) (823.44)

Average Capital Employed 7412.56

3. Calculation of Future Maintainable Profits

(Rs. in lakhs)

2013-14 2014-15 2015-16 2016-17

Profit after tax 938 1092 810 1,646.86

Less: Depreciation net of tax on Premises and Land (Rs. 800 lakhs x 2%) x (100-21.43)%

(12.58)

(12.58)

(12.58)

Depreciation net of tax on Plant and Machinery (Rs. 6,000 lakhs x 10%) x (100-21.43)%

(471.42)

(471.42)

(471.42)

Adjusted Profit 454 608 326 1,646.86

Average adjusted profit (454 + 608 + 326 + 1,646.86)/4

758.72

Less: Excess depreciation (net of tax) due to upward revaluation of premises and land [(2400-800) x 2%] x (100 - 21.43)%

(25.14)

Depreciation on motor vehicle (net of tax) for remaining six months (in future depreciation on motor vehicle will be charged for full year) (Rs. 80 lakhs x 20% x 6/12) x (100-21.43)%

(6.28)

Add: Short depreciation (net of tax) due to downward revaluation of plant and machinery [(6,000 - 4,800) x 10%] x (100 - 21.43)%

94.28

Future Maintainable Profit 821.58

Page 26: Test Series: August, 2017 MOCK TEST PAPER 1 FINAL COURSE ... · The company incurred losses of Rs. 40,000 and Rs. 60,000 in 2015-2016 and 2016-2017 after charging depreciation of

18

4. Calculation of General Expectation

Similar Company pays Rs. 2.5 as dividend (25%) for each share of Rs. 10.

Market value of an equity share of the same denomination is Rs. 25 which fetches

dividend of 25%.

Therefore, share of Rs. 10 (Face value of shares of D Ltd.) is expected to fetch

(2.5/25) x 100 = 10% return.

A nominal rate of 1% or 2% may be added as Risk premium, to the normal rate of

return for uncertainty associated with dividend distribution.

Since, D Ltd. is not having a stable record in payment of dividend, therefore, the

expectation from it may be assumed to be slightly higher, say 11% instead of 10%.

5. Calculation of value of goodwill of D Ltd.

(Rs. in lakhs)

Future maintainable profit 821.58

Less: Normal profit i.e. 11% of average capital employed (7412.56 x 11%)

(815.38)

Super Profit 6.20

Goodwill at 4 years’ purchase of Super Profit (6.2 × 4) 24.80

Notes:

(1) It is assumed that ‘Provision for Taxation’ included in reserves is made in the

current year only.

(2) It is assumed that plant and machinery given in the balance sheet is at cost.

(3) It is assumed that depreciation on ‘Premises and Land’ and ‘Plant and Machinery’ is

charged on Straight Line method.

(4) It is assumed that resale value of ‘Premises and Land’ and ‘Plant and Machinery’ given

in the question is for depreciated value of respective assets. Therefore, no adjustment

for depreciation has been made in such assets while calculating capital employed.

(5) It is assumed that profit for the year 2013-2014, 2014-2015 and 2015-2016 given in

the questions is after tax and no depreciation was charged in the earlier years also.

(6) Average Capital employed has been taken for valuation of goodwill.

(7) While considering past profits for determining average profit, the years 2011-2012

and 2012-2013 have been left out, as during these years normal business was

hampered.

Page 27: Test Series: August, 2017 MOCK TEST PAPER 1 FINAL COURSE ... · The company incurred losses of Rs. 40,000 and Rs. 60,000 in 2015-2016 and 2016-2017 after charging depreciation of

19

7. (a) B limited invested Rs. 600 lakhs in the equity shares of G Ltd. Out of the same, the

company intends to hold 50% shares for long term period i.e. Rs. 300 lakhs and

remaining as temporary (current) investment i.e. Rs. 300 lakhs. Irrespective of the

fact that investment has been held by B Limited only for 3 months (from 1.1.2017 to

31.3.2017), AS 13 lays emphasis on intention of the investor to classify the

investment as current or long term even though the long term investment may be

readily marketable.

In the given situation, the realizable value of al l such investments on 31.3.2017

became Rs. 200 lakhs i.e. Rs. 100 lakhs in respect of current investment and

Rs. 100 lakhs in respect of long term investment.

As per AS 13, ‘Accounting for Investment’, the carrying amount for current

investments is the lower of cost and fair value. In respect of current investments for

which an active market exists, market value generally provides the best evidence of

fair value.

Accordingly, the carrying value of investment held as temporary investment should be

shown at realizable value i.e. at Rs. 100 lakhs. The reduction of Rs. 200 lakhs in the

carrying value of current investment will be included in the profit and loss account.

Standard further states that long-term investments are usually carried at cost.

However, when there is a decline, other than temporary, in the value of long term

investment, the carrying amount is reduced to recognise the decline.

Here, G Limited lost a case of copyright which drastically reduced the realisable

value of its shares to one third which is quiet a substantial figure. Losing the case

of copyright may affect the business and the performance of the company in long

run. Accordingly, it will be appropriate to reduce the carrying amount of long term

investment by Rs. 200 lakhs and shown the investments at Rs. 100 lakhs,

considering the downfall in the value of shares as decline other than temporary.

The reduction of Rs. 200 lakhs in the carrying value of long term investment will be

included in the profit and loss account.

Alternatively for treatment of long term investment.

If one assumes that the decline in the value of long term investment is temporary

and G Limited will overcome this downfall in short period by filing a case against

this decision of government, with strong arguments. In such a case, long term

investment will be shown at cost.

(b) In the given case, A Ltd. concurrently agreed to repurchase the same goods from

B Ltd. on 1st Feb., 2017. Also the re-selling price is pre-determined and covers

purchasing and holding costs of B Ltd. Hence, the transaction between A Ltd. and

Page 28: Test Series: August, 2017 MOCK TEST PAPER 1 FINAL COURSE ... · The company incurred losses of Rs. 40,000 and Rs. 60,000 in 2015-2016 and 2016-2017 after charging depreciation of

20

B Ltd. on 1st Feb., 2017 should be accounted for as financing rather than sale. The

resulting cash flow of Rs. 9.60 lakhs received by A Ltd., cannot be considered as

revenue as per AS 9 “Revenue Recognition”.

Journal Entries in the books of A Ltd.

(Rs. in lakhs)

1.02.2017 Bank Account Dr. 9.60

To Advance from B Ltd. 9.60

(Being advance received from B Ltd amounting [Rs. 8 lakhs + 20% of Rs. 8 lakhs = 9.60 lakhs] under sale and re-purchase agreement)

31.03.2017 Financing Charges Account Dr. 0.40

To B Ltd. 0.40

(Financing charges for 2 months at Rs. 1.20 lakhs [10.80 – 9.60] i.e. 1.2 lakhs x 2/6)

31.03.2017 Profit and Loss Account Dr. 0.40

To Financing Charges Account 0.40

(Being amount of finance charges transferred to P& L Account)

(c) AS 17 ‘Segment Reporting’ requires that inter-segment transfers should be measured

on the basis that the enterprise actually used to price these transfers. The basis of

pricing inter-segment transfers and any change therein should be disclosed in the

financial statements. Hence, the enterprise can have its own policy for pricing inter-

segment transfers and hence, inter-segment transfers may be based on cost, below

cost or market price. However, whichever policy is followed, the same should be

disclosed and applied consistently. Therefore, in the given case inter-segment transfer

pricing policy adopted by the company is correct if, followed consistently.

(d) As per AS 9 “Revenue Recognition”, revenue recognition is mainly concerned with

the timing of recognition of revenue in the Statement of Profit and Loss of an

enterprise. The amount of revenue arising on a transaction is usually determined by

the agreement between the parties involved in the transaction. When uncertainties

exist regarding the determination of the amount, or its associated costs, these

uncertainties may influence the timing of revenue recognition.

Effect of Uncertainty- In the case of the jewellery business the company is selling

for cash and returns are negligible. Hence, revenue can be recognized on sales.

The balance of B Ltd. account will be disclosed as an advance under the heading liabilities in the

balance sheet of A Ltd. as on 31st March, 2017.

Page 29: Test Series: August, 2017 MOCK TEST PAPER 1 FINAL COURSE ... · The company incurred losses of Rs. 40,000 and Rs. 60,000 in 2015-2016 and 2016-2017 after charging depreciation of

21

On the other hand, in Apparels Business, the dealers have a right to return the

unsold goods within one month of the end of the season. In this case, the company

is bearing the risk of sales return and therefore, the company should not recognize

the revenue to the extent of 25% of its sales. The company may disclose suitable

revenue recognition policy in its financial statements separately for both Jewellery

and Apparels business.

(e) Para 28 of AS 25 “Interim Financial Reporting” states that revenues and gains

should be recognised in interim reports on the same basis as used in annual

reports. As at September 30, 2017, L Ltd. would report the entire Rs. 6,00,000 loss

on the disposal of its business segment since the loss was incurred during the

interim period.

A cost charged as an expense in an annual period should be allocated among the

interim periods, which are clearly benefited from the expense, through the use of

accruals and/or deferrals. Since Rs. 1,60,000 property tax payment relates to the

entire 2017 calendar year, only Rs. 80,000 of the payment would be reported as an

expense at September 30, 2017, while out of the remaining Rs. 80,000, Rs. 40,000

for Jan. 2017 to March, 2017 would be shown as payment of the outstanding

amount of previous year and another Rs. 40,000 related to quarter October, 2017 to

December, 2017, would be reported as a prepaid expense.

Page 30: Test Series: August, 2017 MOCK TEST PAPER 1 FINAL COURSE ... · The company incurred losses of Rs. 40,000 and Rs. 60,000 in 2015-2016 and 2016-2017 after charging depreciation of

1

Test Series: August, 2017

MOCK TEST PAPER – 1

FINAL COURSE : GROUP – I

PAPER – 2 : STRATEGIC FINANCIAL MANAGEMENT

Question No. 1 is compulsory. Attempt any five questions from the remaining six questions.

Working notes should form part of the answer.

Time Allowed – 3 Hours Maximum Marks – 100

1. (a) BSE 5000

Value of portfolio Rs. 10,10,000

Risk free interest rate 9% p.a.

Dividend yield on Index 6% p.a.

Beta of portfolio 1.5

We assume that a future contract on the BSE index with four months maturity is

used to hedge the value of portfolio over next three months. One future contract is

for delivery of 50 times the index.

Based on the above information calculate:

(i) Price of future contract.

(ii) The gain on short futures position if index turns out to be 4,500 in three

months. (5 Marks)

(b) From the following details relating to a project, analyze the sensitivity of the project

to changes in initial project cost, annual cash inflow and cost of capital:

Initial Project Cost (Rs.) 1,20,000

Annual Cash Inflow (Rs.) 45,000

Project Life (Years) 4

Cost of Capital 10%

To which of the three factors, the project is most sensitive? (Use annuity factors: for

10% 3.169 and 11% 3.103). (5 Marks)

(c) ABN-Amro Bank, Amsterdam, wants to purchase Rs. 15 million against US$ for

funding their Vostro account with Canara Bank, New Delhi. Assuming the inter -

bank, rates of US$ is Rs. 51.3625/3700, what would be the rate Canara Bank would

quote to ABN-Amro Bank? Further, if the deal is struck, what would be the

equivalent US$ amount. (5 Marks)

Page 31: Test Series: August, 2017 MOCK TEST PAPER 1 FINAL COURSE ... · The company incurred losses of Rs. 40,000 and Rs. 60,000 in 2015-2016 and 2016-2017 after charging depreciation of

2

(d) An investor purchased 300 units of a Mutual Fund at Rs. 12.25 per unit on 31st

December, 2009. As on 31st December, 2010 he has received Rs. 1.25 as dividend

and Rs. 1.00 as capital gains distribution per unit.

Required:

(i) The return on the investment if the NAV as on 31st December, 2010 is

Rs. 13.00.

(ii) The return on the investment as on 31st December, 2010 if all dividends and

capital gains distributions are reinvested into additional units of the fund at

Rs. 12.50 per unit. (5 Marks)

2. (a) A company is considering engaging a factor, the following information is ava ilable:

(i) The current average collection period for the Company’s debtors is 80 days

and ½% of debtors default. The factor has agreed to pay money due after 60

days and will take the responsibility of any loss on account of bad debts.

(ii) The annual charge for the factoring is 2% of turnover payable annually in

arrears. Administration cost saving is likely to be Rs. 1,00,000 per annum.

(iii) Annual sales, all on credit, are Rs. 1,00,00,000. Variable cost is 80% of sales

price. The Company’s cost of borrowing is 15% per annum. Assume the year

is consisting of 365 days.

Should the Company enter into a factoring agreement? (8 Marks)

(b) Yes Ltd. wants to acquire No Ltd. and the cash flows of Yes Ltd. and the merged

entity are given below:

(Rs. In lakhs)

Year 1 2 3 4 5

Yes Ltd. 175 200 320 340 350

Merged Entity 400 450 525 590 620

Earnings would have witnessed 5% constant growth rate without merger and 6%

with merger on account of economies of operations after 5 years in each case. The

cost of capital is 15%.

The number of shares outstanding in both the companies before the merger is the

same and the companies agree to an exchange ratio of 0.5 shares of Yes Ltd. for

each share of No Ltd.

PV factor at 15% for years 1-5 are 0.870, 0.756; 0.658, 0.572, 0.497 respectively.

You are required to:

(i) Compute the Value of Yes Ltd. before and after merger.

(ii) Value of Acquisition and

Page 32: Test Series: August, 2017 MOCK TEST PAPER 1 FINAL COURSE ... · The company incurred losses of Rs. 40,000 and Rs. 60,000 in 2015-2016 and 2016-2017 after charging depreciation of

3

(iii) Gain to shareholders of Yes Ltd. (8 Marks)

3. (a) Armada Leasing Company is considering a proposal to lease out a school bus. The

bus can be purchased for Rs. 5,00,000 and, in turn, be leased out at Rs. 1,25,000

per year for 8 years with payments occurring at the end of each year:

(i) Estimate the internal rate of return for the company assuming tax is ignored.

(ii) What should be the yearly lease payment charged by the company in order to

earn 20 per cent annual compounded rate of return before expenses and

taxes?

(iii) Calculate the annual lease rent to be charged so as to amount to 20% after

tax annual compound rate of return, based on the following assumptions:

(a) Tax rate is 40%;

(b) Straight line depreciation;

(c) Annual expenses of Rs. 50,000; and

(d) Resale value Rs. 1,00,000 after the turn. (8 Marks)

(b) X Ltd., has 8 lakhs equity shares outstanding at the beginning of the year. The

current market price per share is Rs. 120. The Board of Directors of the company is

contemplating Rs. 6.4 per share as dividend. The rate of capitalisation, appropriate

to the risk-class to which the company belongs, is 9.6%:

(i) Based on M-M Approach, calculate the market price of the share of the

company, when the dividend is – (a) declared; and (b) not declared.

(ii) How many new shares are to be issued by the company, if the company

desires to fund an investment budget of Rs. 3.20 crores by the end of the year

assuming net income for the year will be Rs. 1.60 crores? (8 Marks)

4. (a) Zaz plc, a UK Company is in the process of negotiating an order amounting €2.8

million with a large German retailer on 6 month’s credit. I f successful, this will be

first time for Zaz has exported goods into the highly competitive German Market.

The Zaz is considering following 3 alternatives for managing the transaction risk

before the order is finalized.

(I) Mr. Peter the Marketing head has suggested that in order to remove

transaction risk completely Zaz should invoice the German firm in Sterling

using the current €/£ average spot rate to calculate the invoice amount.

(II) Mr. Wilson, CE is doubtful about Mr. Peter’s proposal and suggested an

alternative of invoicing the German firm in € and using a forward exchange

contract to hedge the transaction risk.

(III) Ms. Karen, CFO is agreed with the proposal of Mr. Wilson to invoice the

German first in €, but she is of opinion that Zaz should use sufficient 6 month

Page 33: Test Series: August, 2017 MOCK TEST PAPER 1 FINAL COURSE ... · The company incurred losses of Rs. 40,000 and Rs. 60,000 in 2015-2016 and 2016-2017 after charging depreciation of

4

sterling further contracts (to the nearest whole number) to hedge the

transaction risk.

Following data is available

Spot Rate € 1.1960 - €1.1970/£

6 months forward premium 0.60 – 0.55 Euro Cents.

6 month further contract is currently trading at € 1.1943/£

6 month future contract size is £62,500

After 6 month Spot rate and future rate € 1.1873/£

You are required to

(i) Calculate (to the nearest £) the £ receipt for Zaz plc, under each of 3 above

proposals.

(ii) In your opinion which alternative you consider to be most appropriate.

(8 Marks)

(b) Following is the data regarding six securities:

A B C D E F

Return (%) 8 8 12 4 9 8

Risk (Standard deviation) 4 5 12 4 5 6

(i) Assuming three will have to be selected, state which ones will be picked.

(ii) Assuming perfect correlation, show whether it is preferable to invest 75% in A

and 25% in C or to invest 100% in E. (8 Marks)

5. (a) Personal Computer Division of Distress Ltd., a computer hardware manufacturing

company has started facing financial difficulties for the last 2 to 3 years. The

management of the division headed by Mr. Smith is interested in a buyout on 1 April

2013. However, to make this buy-out successful there is an urgent need to attract

substantial funds from venture capitalists.

Ven Cap, a European venture capitalist firm has shown its interest to finance the

proposed buy-out. Distress Ltd. is interested to sell the division for Rs. 180 crore

and Mr. Smith is of opinion that an additional amount of Rs. 85 crore shall be

required to make this division viable. The expected financing pattern shall be as

follows:

Source Mode Amount (Rs. Crore)

Management Equity Shares of Rs. 10 each 60.00

Page 34: Test Series: August, 2017 MOCK TEST PAPER 1 FINAL COURSE ... · The company incurred losses of Rs. 40,000 and Rs. 60,000 in 2015-2016 and 2016-2017 after charging depreciation of

5

VenCap VC Equity Shares of Rs. 10 each 22.50

9% Debentures with attached warrant of Rs. 100 each

22.50

8% Loan 160.00

Total 265.00

The warrants can be exercised any time after 4 years from now for 10 equity shares

@ Rs. 120 per share.

The loan is repayable in one go at the end of 8 th year. The debentures are

repayable in equal annual installment consisting of both principal and interest

amount over a period of 6 years.

Mr. Smith is of view that the proposed dividend shall not be kept more than 12.5%

of distributable profit for the first 4 years. The forecasted EBIT after the proposed

buyout is as follows:

Year 2013-14 2014-15 2015-16 2016-17

EBIT (Rs. crore) 48 57 68 82

Applicable tax rate is 35% and it is expected that it shall remain unchanged at least

for 5-6 years. In order to attract VenCap, Mr. Smith stated that book value of equity

shall increase by 20% during above 4 years. Although, VenCap has shown their

interest in investment but are doubtful about the projections of growth in the value

as per projections of Mr. Smith. Further VenCap also demanded that warrants

should be convertible in 18 shares instead of 10 as proposed by Mr. Smith.

You are required to determine whether or not the book value of equity is expected

to grow by 20% per year. Further if you have been appointed by Mr. Smith as

advisor then whether you would suggest to accept the demand of VenCap of 18

shares instead of 10 or not. (8 Marks)

(b) On 31st March, 2013, the following information about Bonds is available:

Name of Security Face Value Rs.

Maturity Date Coupon Rate

Coupon Date(s)

Zero coupon 10,000 31st March, 2023 N.A. N.A.

T-Bill 1,00,000 20th June, 2013 N.A. N.A.

10.71% GOI 2023 100 31st March, 2023 10.71 31st March

10% GOI 2018 100 31st March, 2018 10.00 31st March & 30th September

Calculate:

(i) If 10 years yield is 7.5% p.a. what price the Zero Coupon Bond would fetch on

31st March, 2013?

Page 35: Test Series: August, 2017 MOCK TEST PAPER 1 FINAL COURSE ... · The company incurred losses of Rs. 40,000 and Rs. 60,000 in 2015-2016 and 2016-2017 after charging depreciation of

6

(ii) What will be the annualized yield if the T-Bill is traded @ 98500?

(iii) If 10.71% GOI 2023 Bond having yield to maturity is 8%, what price would it

fetch on April 1, 2013 (after coupon payment on 31st March)?

(iv) If 10% GOI 2018 Bond having yield to maturity is 8%, what price would it fetch

on April 1, 2013 (after coupon payment on 31st March)? (8 Marks)

6. (a) Derivative Bank entered into a plain vanilla swap through on OIS (Overnight Index

Swap) on a principal of Rs. 10 crores and agreed to receive MIBOR overnight

floating rate for a fixed payment on the principal. The swap was entered into on

Monday, 2nd August, 2010 and was to commence on 3rd August, 2010 and run for a

period of 7 days.

Respective MIBOR rates for Tuesday to Monday were:

7.75%,8.15%,8.12%,7.95%,7.98%,8.15%.

If Derivative Bank received Rs. 317 net on settlement, calculate Fixed rate and

interest under both legs.

Notes:

(i) Sunday is Holiday.

(ii) Work in rounded rupees and avoid decimal working. (8 Marks)

(b) A company has an old machine having book value zero – which can be sold for

Rs. 50,000. The company is thinking to choose one from following two alternatives:

(i) To incur additional cost of Rs. 10,00,000 to upgrade the old existing machine.

(ii) To replace old machine with a new machine costing Rs. 20,00,000 plus

installation cost Rs. 50,000.

Both above proposals envisage useful life to be five years with salvage value to be

nil.

The expected after tax profits for the above three alternatives are as under:

Year Old existing Machine (Rs.)

Upgraded Machine (Rs.)

New Machine (Rs.)

1 5,00,000 5,50,000 6,00,000

2 5,40,000 5,90,000 6,40,000

3 5,80,000 6,10,000 6,90,000

4 6,20,000 6,50,000 7,40,000

5 6,60,000 7,00,000 8,00,000

The tax rate is 40 per cent.

Page 36: Test Series: August, 2017 MOCK TEST PAPER 1 FINAL COURSE ... · The company incurred losses of Rs. 40,000 and Rs. 60,000 in 2015-2016 and 2016-2017 after charging depreciation of

7

The company follows straight line method of depreciation. Assume cost of capital to

be 15 per cent.

P.V.F. of 15%, 5 = 0.870, 0.756, 0.658, 0.572 and 0.497. You are required to advise

the company as to which alternative is to be adopted. (8 Marks)

7. Write short notes on any of four of the following:

(a) Discuss the Random Walk Theory.

(b) Mention the various techniques used in economic analysis .

(c) Explain the term 'Buy-Outs'.

(d) Nostro, Vostro and Loro Accounts

(e) Write a short note on Call Money. (4 × 4 = 16 Marks)

Page 37: Test Series: August, 2017 MOCK TEST PAPER 1 FINAL COURSE ... · The company incurred losses of Rs. 40,000 and Rs. 60,000 in 2015-2016 and 2016-2017 after charging depreciation of

1

Test Series: August, 2017

MOCK TEST PAPER – 1

FINAL COURSE: GROUP – I

PAPER – 2: STRATEGIC FINANCIAL MANAGEMENT

SUGGESTED ANSWERS/HINTS

1. (a)

(i) Current future price of the index

= 5000 + 5000 (0.09-0.06)12

4 = 5000 + 50 = 5,050

Price of the future contract = Rs.50 х 5,050 = Rs.2,52,500

(ii) Hedge ratio = 5.1252500

1010000 = 6 contracts

Index after there months turns out to be 4500

Future price will be = 4500 + 4500 (0.09-0.06) 12

1× = 4,511.25

Therefore, Gain from the short futures position is = 6 х (5050 – 4511.25) х 50 = Rs.1,61,625

Note: Alternatively we can also use daily compounding (exponential) formula

(b) CALCULATION OF NPV

Rs.

PV of cash inflows (Rs. 45,000 x 3.169) 1,42,605

Initial Project Cost 1,20,000

NPV 22,605

If initial project cost is varied adversely by 10%*

NPV (Revised) (Rs. 1,42,605 - Rs. 1,32,000) Rs. 10,605

Change in NPV (Rs. 22,605 – Rs. 10,605)/ Rs. 22,605 i.e. 53.08 %

If annual cash inflow is varied adversely by 10%*

Revised annual inflow Rs. 40,500

NPV (Revised) (Rs. 40,500 x 3.169) – (Rs. 1,20,000) (+) Rs. 8,345

Change in NPV (Rs. 22,605 – Rs. 8,345) / Rs. 22,605 63.08 %

If cost of capital is varied adversely by 10%*

Page 38: Test Series: August, 2017 MOCK TEST PAPER 1 FINAL COURSE ... · The company incurred losses of Rs. 40,000 and Rs. 60,000 in 2015-2016 and 2016-2017 after charging depreciation of

2

NPV (Revised) (Rs. 45,000 x 3.103) – Rs. 1,20,000 (+) Rs. 19,635

Change in NPV (Rs. 22,605 – Rs. 19,635) / Rs. 22,605 13.14 %

Conclusion: Project is most sensitive to ‘annual cash inflow’.

*Note: Students may please note that they may assume any other percentage rate

other than 10 % say 15%, 20 % 25 % etc.

(c) Here Canara Bank shall buy US$ and credit Rs. to Vostro account of ABN-Amro

Bank. Canara Bank’s buying rate will be based on the Inter -bank Buying Rate (as

this is the rate at which Canara Bank can sell US$ in the Interbank market)

Accordingly, the Interbank Buying Rate of US$ will be Rs. 51.3625 (lower of two)

Equivalent of US$ for Rs. 15 million at this rate will be

= 15,000,000

51.3625 = US$ 2,92,041.86

(d) Return for the year (all changes on a per year basis)

Particulars Rs. /Unit

Change in price (Rs. 13.00 – Rs. 12.25) 0.75

Dividend received 1.25

Capital gain distribution 1.00

Total Return 3.00

Return on investment = 10025.12

00.3 = 24.49%

If all dividends and capital gain are reinvested into additional units at Rs. 12.50 per

unit the position would be.

Total amount reinvested = Rs. 2.25 300 = Rs. 675

Additional units added = 50.12

675 .Rs = 54 units

Value of 354 units as on 31-12-2010 = Rs. 4,602

Price paid for 300 units on 31-12-2009 (300 Rs. 12.25) = Rs. 3,675

Return = 675,3 .Rs

675,3 .Rs602,4.Rs - =

675,3 .Rs

927 .Rs = 25.22%

Page 39: Test Series: August, 2017 MOCK TEST PAPER 1 FINAL COURSE ... · The company incurred losses of Rs. 40,000 and Rs. 60,000 in 2015-2016 and 2016-2017 after charging depreciation of

3

2. (a) The annual change in cash flows through entering into a factoring agreement is:

Savings Amount in (Rs.)

Amount in (Rs.)

Administration cost saved 1,00,000

Existing average debtors

[Rs. 1,00,00,000/365) x 80 days]

21,91,781

Average New Debtors 16,43,836

[(Rs. 1,00,00,000/365) x 60 days]

Reduction in debtors 5,47,945

Cost there of @80% 4,38,356

Add: Interest saving @15% p.a. on. Rs. 4,38,356 65,753

Add: Bad Debts saved @.005 of Rs. 1,00,00,000 50,000

Total 2,15,753

Less: Annual charges @2% of Rs. 1,00,00,000 2,00,000

Net annual benefits of factoring 15,753

Therefore, the factoring agreement is worthwhile and should be undertaken.

(b) (i) Working Notes:

Present Value of Cash Flows (CF) upto 5 years

Year

End

CF of Yes Ltd.

(Rs. lakhs)

PVF

@15%

PV of CF (Rs. lakhs)

CF of Merged Entity

(Rs. lakhs)

PV of CF of Merged Entity

(Rs. lakhs)

1 175 0.870 152.25 400 348.00

2 200 0.756 151.20 450 340.20

3 320 0.658 210.56 525 345.45

4 340 0.572 194.48 590 337.48

5 350 0.497 173.95 620 308.14

882.44 1679.27

PV of Cash Flows of Yes Ltd. after the forecast period

TV5 = 5

e

CF (1 g)

K g

=

350(1 0.05)

0.15 0.05

=

367.50

0.10= Rs.3675 lakhs

PV of TV5 = Rs.3675 lakhs x 0.497 = Rs.1826.475 lakhs

PV of Cash Flows of Merged Entity after the forecast period

Page 40: Test Series: August, 2017 MOCK TEST PAPER 1 FINAL COURSE ... · The company incurred losses of Rs. 40,000 and Rs. 60,000 in 2015-2016 and 2016-2017 after charging depreciation of

4

TV5 = 5

e

CF (1 g)

K g

=

620(1 0.06)

0.15 0.06

=

657.20

0.09= Rs.7302.22 lakhs

PV of TV5 = Rs.7302.22 lakhs x 0.497 = Rs.3629.20 lakhs

Value of Yes Ltd.

Before merger

(Rs. lakhs)

After merger

(Rs. lakhs)

PV of CF (1-5 years) 882.440 1679.27

Add: PV of TV5 1826.475 3629.20

2708.915 5308.47

(ii) Value of Acquisition

= Value of Merged Entity – Value of Yes Ltd.

= Rs.5308.47 lakhs – Rs.2708.915 lakhs = Rs.2599.555 lakhs

(iii) Gain to Shareholders of Yes Ltd.

Share of Yes Ltd. in merged entity = Rs.5308.47 lakhs x 1

1.5= Rs.3538.98 lakhs

Gain to shareholder = Share of Yes Ltd. in merged entity – Value of Yes Ltd.

before merger

= Rs.3538.98 lakhs - Rs.2708.915 = Rs.830.065 lakhs

3. (a) (i) Payback period = 000,25,1

000,00,5= 4.00

PV factor closest to 4.00 in 8 years is 4.078 at 18%

Thus IRR = 18%

Note: Students may also arrive at the answer of 18.63% instead of 18% if exact

calculation are made as follows:-

PV factor in 8 years at 19% is 3.9544

Interpolating for 4.00

IRR = %63.18 9544.3 - 0776.4

000.4 - 0776.4 %18

(ii) Desired lease rent to earn 20% IRR before expenses and taxes:

Lease Rent = 20% yr, 8PVIFA

5,00,000 =

837.3

000,00,5 = Rs. 1,30,310.14 p.a.

Page 41: Test Series: August, 2017 MOCK TEST PAPER 1 FINAL COURSE ... · The company incurred losses of Rs. 40,000 and Rs. 60,000 in 2015-2016 and 2016-2017 after charging depreciation of

5

(iii) Revised lease rental on school bus to earn 20% return based on the given

conditions.

PV factor [(X – E – D) (1 – T) + D] + (PV factor × S.V.) = Co

3.837 [(x – 50,000 – 50,000) (1 – .4) + 50,000] + (.233 × 1,00,000*) = 5,00,000

3.837 [6x – 60,000 + 50,000)] + 23,300 = 5,00,000

2.3022x = 5,15,070

x = 2,23,729.47

This may be confirmed as lease rental 2,23,729.47

Less: Expenses + Depreciation 1,00,000.00

EBT 1,23,729.47

Less tax 40% 49,491.79

PAT 74,237.68

Add: Depreciation 50,000.00

CFAT 1,24,237.68

CFAT

SV ofPV - Co =

68.237,24,1

23,300 - 5,00,000 = 3.837 or 20%

* Note: Alternatively, STCG can also be considered as net of tax.

(b) Modigliani and Miller (M-M) – Dividend Irrelevancy Model:

e

110

K 1

D P P

Where,

Po = Existing market price per share i.e. Rs. 120

P1 = Market price of share at the year-end (to be determined)

D1 = Contemplated dividend per share i.e. Rs. 6.4

Ke = Capitalisation rate i.e. 9.6%.

(i) (a) Calculation of share price when dividend is declared:

e

110

K 1

D P P

0.096 1

6.4 P 120 1

120 × 1.096 = P1 + 6.4

Page 42: Test Series: August, 2017 MOCK TEST PAPER 1 FINAL COURSE ... · The company incurred losses of Rs. 40,000 and Rs. 60,000 in 2015-2016 and 2016-2017 after charging depreciation of

6

P1 = 120 × 1.096 – 6.4

= 125.12

(b) Calculation of share price when dividend is not declared:

e

110

K 1

D P P

0.096 1

0 P 120 1

120 × 1.096 = P1 + 0

P1 = 131.52

(ii) Calculation of No. of shares to be issued:

(Rs. in lakhs)

Particulars If dividend declared

If dividend not declared

Net Income 160 160

Less: Dividend paid 51.20 ------

Retained earnings 108.80 160

Investment budget 320 320

Amount to be raised by issue of new shares (i) 211.20 160

Market price per share (ii) 125.12 131.52

No. of new shares to be issued (ii) 1,68,797.95 1,21,654.50

Or say 1,68,798 1,21,655

4. (a) (i) Receipt under three proposals

(a) Proposal of Mr. Peter

Invoicing in £ will produce = 1.1965

million2.8 € = £ 2.340 million

(b) Proposal of Mr. Wilson

Forward Rate = €1.1970-0.0055 = 1.1915

Using Forward Market hedge Sterling receipt would be 1915.1

million8.2€

= £ 2.35 million

(c) Proposal of Ms. Karen

Page 43: Test Series: August, 2017 MOCK TEST PAPER 1 FINAL COURSE ... · The company incurred losses of Rs. 40,000 and Rs. 60,000 in 2015-2016 and 2016-2017 after charging depreciation of

7

The equivalent sterling of the order placed based on future price

(€1.1943)

= 1943.1

million8.2€ = £ 2,344,470 (rounded off)

Number of Contracts = 500,62

470,344,2£ = 37 Contracts (to the nearest whole

number)

Thus, € amount hedged by future contract wi ll be = 37£62,500 =

£23,12,500

Buy Future at €1.1943

Sell Future at €1.1873

€0.0070

Total loss on Future Contracts = 37£62,500€0.0070 =€16,188

After 6 months

Amount Received €28,00,000

Less: Loss on Future Contracts € 16,188

€ 27,83,812

Sterling Receipts

On sale of € at spot = million3446.2 £=1873.1

812,83,27€

(ii) Proposal of option (b) is preferable because the option (a) & (c) produces least

receipts. Further, in case of proposal (a) there must be a doubt as to whether

this would be acceptable to German firm as it is described as a competitive

market and Zaz is moving into it first time.

(b) (i) Security A has a return of 8% for a risk of 4, whereas B and F have a higher

risk for the same return. Hence, among them A dominates.

For the same degree of risk 4, security D has only a return of 4%. Hence, D is

also dominated by A.

Securities C and E remain in reckoning as they have a higher return though

with higher degree of risk.

Hence, the ones to be selected are A, C & E.

(ii) The average values for A and C for a proportion of 3 : 1 will be :

Risk =4

)121()43( = 6%

Page 44: Test Series: August, 2017 MOCK TEST PAPER 1 FINAL COURSE ... · The company incurred losses of Rs. 40,000 and Rs. 60,000 in 2015-2016 and 2016-2017 after charging depreciation of

8

Return = 4

)121()83( = 9%

Therefore: 75% A E

25% C _

Risk 6 5

Return 9% 9%

For the same 9% return the risk is lower in E. Hence, E will be preferable.

Alternatively, it can also be computed by use of Graph as follows:

Since securities other than A, E and C are not on Efficient Frontier they are

rejected.

5. (a) Working Notes

Calculation of Interest Payment on 9% Debentures

PVAF (9%,6) = 4.486

Annual Installment = Rs. 22.50 crore

4.486

= Rs. 5.0156 crore

Year Balance Outstanding

(Rs. Crore)

Interest

(Rs. Crore)

Installment

(Rs. Crore)

Principal Repayment

(Rs. Crore)

Balance

(Rs. Crore)

1 22.5000 2.025 5.0156 2.9906 19.5094

2 19.5094 1.756 5.0156 3.2596 16.2498

3 16.2498 1.462 5.0156 3.5536 12.6962

4 12.6962 1.143 5.0156 3.8726 8.8236

Page 45: Test Series: August, 2017 MOCK TEST PAPER 1 FINAL COURSE ... · The company incurred losses of Rs. 40,000 and Rs. 60,000 in 2015-2016 and 2016-2017 after charging depreciation of

9

Statement showing Value of Equity

Particulars 2013-14

(Rs. Crore)

2014-15

(Rs. Crore)

2015-16

(Rs. Crore)

2016-17

(Rs. Crore)

EBIT 48.0000 57.0000 68.0000 82.0000

Interest on 9% Debentures

2.0250 1.7560 1.4620 1.1430

Interest on 8% Loan 12.8000 12.8000 12.8000 12.8000

EBT 33.1750 42.4440 53.7380 68.0570

Tax* @35% 11.6110 14.8550 18.8080 23.8200

EAT 21.5640 27.5890 34.9300 44.2370

Dividend @12.5% of EAT*

2.6955 3.4490 4.3660 5.5300

18.8685 24.1400 30.5640 38.7070

Balance b/f Nil 18.8685 43.0085 73.5725

Balance c/f 18.8685 43.0085 73.5725 112.2795

Share Capital 82.5000 82.5000 82.5000 82.5000

101.3685 125.5085 156.0725 194.7795

*Figures have been rounded off.

In the beginning of 2013-14 equity was Rs. 82.5000crore which has been grown to

Rs. 194.7795 over a period of 4 years. In such case the compounded growth rate

shall be as follows:

(194.7795/82.5000)¼ - 1 = 23.96%

This growth rate is slightly higher than 20% as projected by Mr. Smith.

If the condition of VenCap for 18 shares is accepted the expected share holding

after 4 years shall be as follows:

No. of shares held by Management 6.00 crore

No. of shares held by VenCap at the starting stage 2.25 crore

No. of shares held by VenCap after 4 years 4.05 crore

Total holding 6.30 crore

Thus, it is likely that Mr. Smith may not accept this condition of VenCap as this may

result in losing their majority ownership and control to VenCap. Mr. Smith may

accept their condition if management has further opportunity to increase their

ownership through other forms.

Page 46: Test Series: August, 2017 MOCK TEST PAPER 1 FINAL COURSE ... · The company incurred losses of Rs. 40,000 and Rs. 60,000 in 2015-2016 and 2016-2017 after charging depreciation of

10

(b) (i) Rate used for discounting shall be yield. Accordingly ZCB shall fetch:

= 10

10000

(1 0.075)= Rs. 4,852

(ii) The day count basis is actual number days / 365. Accordingly annualized yield

shall be:

FV-Price 365

Yield Price No. of days

= 100000-98500 365

98500 81

= 6.86%

Note: Alternatively, it can also computed on 360 days a year.

(iii) Price GOI 2023 would fetch

= Rs. 10.71 PVAF(8%, 10) + Rs. 100 PVF (8%, 10)

= Rs. 10.71 x 6.71 + Rs. 100 x 0.4632

= Rs. 71.86 + Rs. 46.32 = Rs. 118.18

(iv) Price GOI 2018 Bond would fetch:

= Rs. 5 PVAF (4%, 10) + Rs. 100 PVF (4%, 10)

= Rs. 5 x 8.11 + Rs. 100 x 0.6756

= 40.55 + 67.56 = 108.11

6. (a)

Day Principal (Rs.) MIBOR (%) Interest (Rs.)

Tuesday 10,00,00,000 7.75 21,233

Wednesday 10,00,21,233 8.15 22,334

Thursday 10,00,43,567 8.12 22,256

Friday 10,00,65,823 7.95 21,795

Saturday & Sunday (*) 10,00,87,618 7.98 43,764

Monday 10,01,31,382 8.15 22,358

Total Interest @ Floating 1,53,740

Less: Net Received 317

Expected Interest @ fixed 1,53,423

Thus Fixed Rate of Interest 0.07999914

Approx. 8%

(*) i.e. interest for two days.

Note: Alternatively, answer can also be calculated on the basis of 360 days in a

year.

Page 47: Test Series: August, 2017 MOCK TEST PAPER 1 FINAL COURSE ... · The company incurred losses of Rs. 40,000 and Rs. 60,000 in 2015-2016 and 2016-2017 after charging depreciation of

11

(b)

(A) Cash Outflow Rs.

(i) In case machine is upgraded:

Upgradation Cost 10,00,000

(ii) In case new machine installed:

Cost 20,00,000

Add: Installation cost 50,000

Total Cost 20,50,000

Less: Disposal of old machine

Rs. 50,000 – 40% tax 30,000

Total Cash Outflow 20,20,000

Working Note:

(i) Depreciation – in case machine is upgraded

Rs. 10,00,000 ÷ 5 = Rs. 2,00,000

(ii) Depreciation – in case new machine is installed

Rs. 20,50,000 ÷ 5 = Rs. 4,10,000

(iii) Old existing machine – Book Value is zero. So no depreciation.

(B) Cash Inflows after Taxes (CFAT)

Year

Old Existing Machine

Upgraded Machine

(i)

EAT/CFAT

Rs.

(ii)

EAT

Rs.

(iii)

DEP

Rs.

(iv)

CFAT

Rs.

= (iv)-(i)

Incremental CFAT Rs.

1 5,00,000 5,50,000 2,00,000 7,50,000 2,50,000

2 5,40,000 5,90,000 2,00,000 7,90,000 2,50,000

3 5,80,000 6,10,000 2,00,000 8,10,000 2,30,000

4 6,20,000 6,50,000 2,00,000 8,50,000 2,30,000

5 6,60,000 7,00,000 2,00,000 9,00,000 2,40,000

Page 48: Test Series: August, 2017 MOCK TEST PAPER 1 FINAL COURSE ... · The company incurred losses of Rs. 40,000 and Rs. 60,000 in 2015-2016 and 2016-2017 after charging depreciation of

12

Cash Inflow after Taxes (CFAT)

Year

New Machine

(vi)

EAT

Rs.

(vii)

DEP

Rs.

(viii)

CFAT

Rs.

(ix) = (viii) – (i)

Incremental CFAT

(Rs. )

1 6,00,000 4,10,000 10,10,000 5,10,000

2 6,40,000 4,10,000 10,50,000 5,10,000

3 6,90,000 4,10,000 11,00,000 5,20,000

4 7,40,000 4,10,000 11,50,000 5,30,000

5 8,00,000 4,10,000 12,10,000 5,50,000

P.V. AT 15% - 5 Years – on Incremental CFAT

Year

Upgraded Machine New Machine

Incremental CFAT

Rs.

PVF Total P.V.

Rs.

Incremental CFAT

PVF Total PV

Rs.

1 2,50,000 0.870 2,17,500 5,10,000 0.870 4,43,700

2 2,50,000 0.756 1,89,000 5,10,000 0.756 3,85,560

3 2,30,000 0.658 1,51,340 5,20,000 0.658 3,42,160

4 2,30,000 0.572 1,31,560 5,30,000 0.572 3,03,160

5. 2,40,000 0.497 1,19,280 5,50,000 0.497 2,73,350

Total P.V. of CFAT 8,08,680 17,47,930

Less: Cash Outflows 10,00,000 20,20,000*

N.P.V. = 1,91,320 - 2,72,070

*Acquisition Cost (including installation cost) Rs. 20,50,000

Less: Salvage Value of existing machine net of Tax Rs. 30,000

Rs. 20,20,000

As the NPV in both the new (alternative) proposals is negative, the company should

continue with the existing old Machine.

7. (a) Many investment managers and stock market analysts believe that stock market

prices can never be predicted because they are not a result of any underlying

factors but are mere statistical ups and downs. This hypothesis is known as

Random Walk hypothesis which states that the behaviour of stock market prices is

unpredictable and that there is no relationship between the present prices of the

Page 49: Test Series: August, 2017 MOCK TEST PAPER 1 FINAL COURSE ... · The company incurred losses of Rs. 40,000 and Rs. 60,000 in 2015-2016 and 2016-2017 after charging depreciation of

13

shares and their future prices. Proponents of this hypothesis argue that stock

market prices are independent. A British statistician, M. G. Kendell, found that

changes in security prices behave nearly as if they are generated by a suitably

designed roulette wheel for which each outcome is statistically independent of the

past history. In other words, the fact that there are peaks and troughs in stock

exchange prices is a mere statistical happening – successive peaks and troughs are

unconnected. In the layman's language it may be said that prices on the stock

exchange behave exactly the way a drunk would behave while walking in a blind

lane, i.e., up and down, with an unsteady way going in any direction he likes,

bending on the side once and on the other side the second time.

The supporters of this theory put out a simple argument. It follows that:

(a) Prices of shares in stock market can never be predicted. The reason is that the

price trends are not the result of any underlying factors, but that they represent

a statistical expression of past data.

(c) There may be periodical ups or downs in share prices, but no connection can

be established between two successive peaks (high price of stocks) and

troughs (low price of stocks).

(b) Some of the techniques used for economic analysis are:

(i) Anticipatory Surveys: They help investors to form an opinion about the future

state of the economy. It incorporates expert opinion on construction activities,

expenditure on plant and machinery, levels of inventory – all having a definite

bearing on economic activities. Also future spending habits of consumers are

taken into account.

(ii) Barometer/Indicator Approach: Various indicators are used to find out how

the economy shall perform in the future. The indicators have been classified as

under:

(1) Leading Indicators: They lead the economic activity in terms of their

outcome. They relate to the time series data of the variables that reach

high/low points in advance of economic activity.

(2) Roughly Coincidental Indicators: They reach their peaks and troughs at

approximately the same in the economy.

(3) Lagging Indicators: They are time series data of variables that lag behind

in their consequences vis-a-vis the economy. They reach their turning

points after the economy has reached its own already.

All these approaches suggest direction of change in the aggregate economic

activity but nothing about its magnitude.

(iii) Economic Model Building Approach: In this approach, a precise and clear

relationship between dependent and independent variables is determined.

Page 50: Test Series: August, 2017 MOCK TEST PAPER 1 FINAL COURSE ... · The company incurred losses of Rs. 40,000 and Rs. 60,000 in 2015-2016 and 2016-2017 after charging depreciation of

14

GNP model building or sectoral analysis is used in practice through the use of

national accounting framework.

(c) A very important phenomenon witnessed in the Mergers and Acquisitions scene, in

recent times is one of buy - outs. A buy-out happens when a person or group of

persons gain control of a company by buying all or a majority of its shares. A buyou t

involves two entities, the acquirer and the target company. The acquirer seeks to

gain controlling interest in the company being acquired normally through purchase

of shares. There are two common types of buy-outs: Leveraged Buyouts (LBO) and

Management Buy-outs (MBO). LBO is the purchase of assets or the equity of a

company where the buyer uses a significant amount of debt and very little equity

capital of his own for payment of the consideration for acquisition. MBO is the

purchase of a business by its management, who when threatened with the sale of

its business to third parties or frustrated by the slow growth of the company, step -in

and acquire the business from the owners, and run the business for themselves.

The majority of buy-outs is management buy-outs and involves the acquisition by

incumbent management of the business where they are employed. Typically, the

purchase price is met by a small amount of their own funds and the rest from a mix

of venture capital and bank debt.

Internationally, the two most common sources of buy-out operations are divestment

of parts of larger groups and family companies facing succession problems.

Corporate groups may seek to sell subsidiaries as part of a planned strategic

disposal programme or more forced reorganisation in the face of parental financing

problems. Public companies have, however, increasingly sought to dispose of

subsidiaries through an auction process partly to satisfy shareholder pressure for

value maximisation.

In recessionary periods, buy-outs play a big part in the restructuring of a failed or

failing businesses and in an environment of generally weakened corporate

performance often represent the only viable purchasers when parents wish to

dispose of subsidiaries.

Buy-outs are one of the most common forms of privatisation, offering opportunities

for enhancing the performances of parts of the public sector, widening employee

ownership and giving managers and employees incentives to make best use of their

expertise in particular sectors.

(d) In interbank transactions, foreign exchange is transferred from one account to

another account and from one centre to another centre. Therefore, the banks

maintain three types of current accounts in order to facilitate quick transfer of funds

in different currencies. These accounts are Nostro, Vostro and Loro accounts

meaning “our”, “your” and “their”. A bank’s foreign currency account maintained by

the bank in a foreign country and in the home currency of that country is known as

Nostro Account or “our account with you”. For example, An Indian bank’s Swiss

Page 51: Test Series: August, 2017 MOCK TEST PAPER 1 FINAL COURSE ... · The company incurred losses of Rs. 40,000 and Rs. 60,000 in 2015-2016 and 2016-2017 after charging depreciation of

15

franc account with a bank in Switzerland. Vostro account is the local currency

account maintained by a foreign bank/branch. It is also called “your account with

us”. For example, Indian rupee account maintained by a bank in Switzerland with a

bank in India. The Loro account is an account wherein a bank remits funds in

foreign currency to another bank for credit to an account of a third bank.

(e) Call Money: The Call Money is a part of the money market where, day to day

surplus funds, mostly of banks, are traded. Moreover, the call money market is most

liquid of all short-term money market segments.

The maturity period of call loans vary from 1 to 14 days. The money that is lent for

one day in call money market is also known as ‘overnight money’. The interest paid

on call loans are known as the call rates. The call rate is expected to freely reflect

the day-to-day lack of funds. These rates vary from day-to-day and within the day,

often from hour-to-hour. High rates indicate the tightness of liquidity in the financial

system while low rates indicate an easy liquidity position in the market.

In India, call money is lent mainly to even out the short -term mismatches of assets

and liabilities and to meet CRR requirement of banks. The short -term mismatches

arise due to variation in maturities i.e. the deposits mobilized are deployed by the

bank at a longer maturity to earn more returns and duration of withdrawal of

deposits by customers vary. Thus, the banks borrow from call money markets to

meet short-term maturity mismatches.

Moreover, the banks borrow from call money market to meet the cash Reserve

Ratio (CRR) requirements that they should maintain with RBI every fortnight and is

computed as a percentage of Net Demand and Time Liabilities (NDTL).

Page 52: Test Series: August, 2017 MOCK TEST PAPER 1 FINAL COURSE ... · The company incurred losses of Rs. 40,000 and Rs. 60,000 in 2015-2016 and 2016-2017 after charging depreciation of

1

Test Series: August, 2017

MOCK TEST PAPER -1

FINAL COURSE: GROUP – I

PAPER – 3: ADVANCED AUDITING AND PROFESSIONAL ETHICS

Question No. 1 is compulsory

Answer any five from the rest

Time Allowed- 3 Hours Maximum Marks -100

1. (a) S N Projects Limited, a manufacturing company in the Steel industry was allegedly

involved in some irregularity relating to allotment of coal blocks for which a

complaint was lodged against the company by the government. The financial

institutions stopped additional working capital finance which caused a financial

crisis resulting in stoppage of production. The company incurred a massive loss

during the year 2015-16. There were delays in salary and other payments. Certain

key managerial personnel including GM Finance and certain other employees left

the company. The company has no sound action plan to mitigate these situations.

Guide the statutory auditor on how he should deal with this situation.

(b) Y Ltd. has its entire operations including accounting computerized. As the audi t

partner you are concerned about inherent and control risk for material financial

statement assertions. What could be the areas you look forward for deficiencies

and risk identification?

(c) Mr. H who is appointed as auditor of EXL Co. Ltd. wants to use confirmation request

as audit evidence during the course of audit. What are the factors to be considered

by Mr. H when designing a confirmation request? Also state the effects of using

positive external confirmation request by Mr. H.

(d) The management of Honey Ltd. has prepared its summary financial statements for

the year 2015-16 to be provided to its investors. Consequently the company wants

to appoint you for conducting audit of summary financial statements. What are the

procedures that you will perform and consider necessary as the basis for forming an

opinion on the summary financial statements? (5 4 = 20 Marks)

2. Comment on the following with reference to the Chartered Accountants Act, 1949 and

schedules thereto:

(a) CA Ciya, a practicing Chartered Accountant gave 60% of the audit fees received by

her to Mr. Suhas, who was not a Chartered Accountant, under the nomenclature of

office allowances.

(b) CA Raj, a Chartered Accountant in practice approached Manager of a Nationa lised

Bank for a loan of Rs. 36 lakhs. He has also informed the Manager that if the loan is

Page 53: Test Series: August, 2017 MOCK TEST PAPER 1 FINAL COURSE ... · The company incurred losses of Rs. 40,000 and Rs. 60,000 in 2015-2016 and 2016-2017 after charging depreciation of

2

sanctioned, the Income Tax return of the Manager and Manager’s wife will be filed

for free of cost, as quid Pro quo for the loan sanctioned.

(c) CA Naman, a Chartered Accountant in practice is a partner in 4 firms. On the

personal Letter Heads, he mentioned the names and addresses of all the 4 firms in

small fonts.

(d) CA Dev, a practicing Chartered Accountant, did not complete his work relating to

the audit of the accounts of a company and had not submitted his audit report in

due time to enable the company to comply with the statutory requirements .

(4 4 = 16 Marks)

3. (a) State the key requirements of auditing standards of Public Company Accounting

over sight board. (4 Marks)

(b) Modest Hospital is a multi-speciality hospital which has been facing a lot of pilferage

and troubles regarding their inventory maintenance and control. On investigation

into the matter it was found that the person in charge of inventory inflow and outflow

from the store house is also responsible for purchases and maintaining inventory

records.

According to you, which basis system of control has been violated? Also list down

the other general conditions pertaining to such system which needs to be

maintained and checked by the management. (6 Marks)

(c) As an auditor of garment manufacturing company for the last five years you have

observed that new venture of online shopping has been added by the company

during current year. As an auditor what factors would be considered by you in

formulating the audit strategy of the company? (6 Marks)

4. (a) Under CARO, 2016, as a statutory auditor, how would you report:

(i) JKL Ltd. has entered into non-cash transactions with Mr. Rahul, son of

director, which is an arrangement by which the JKL Ltd. is in process to

acquire assets for consideration other than cash.

(ii) STP Limited has its factory building, appearing as fixed assets in its financial

statements in the name of one of its director who was overlooking the

manufacturing activities. (4 2 = 8 Marks)

(b) PING Limited is a listed Company in which no code of conduct is laid down for its

board members and senior members.

As an auditor of PING Limited:

(i) Briefly explain the compliance requirements with respect to Code of Conduct

as per Listing Order Disclosure Requirement (LODR) Regulations.

(ii) What will be your role in compliance of above-mentioned Code of Conduct as

per LODR Regulations? (4 2 = 8 Marks)

Page 54: Test Series: August, 2017 MOCK TEST PAPER 1 FINAL COURSE ... · The company incurred losses of Rs. 40,000 and Rs. 60,000 in 2015-2016 and 2016-2017 after charging depreciation of

3

5. (a) You have been appointed as a statutory central auditor of APNA Bank, a

Nationalized bank. What special points would be borne in mind while conducting the

audit of compliance with "Statutory Liquidity Ratio" (SLR) requirements?

(4 Marks)

(b) The RBI restrict companies from carrying on the business of a non-banking financial

institution without obtaining the certificate of registration, therefore, obtaining

registration under section 45-IA of the Reserve Bank of India Act, 1934 is

necessary. Additionally, new clause has been inserted under CARO, 2016 for

commenting whether the registration has been obtained, if required.

Being an expert in latest provisions under CARO, 2016, you are required to state in

brief the audit procedure and reporting to be followed under above mentioned

circumstances. (6 Marks)

(c) A leading manufacturing concern valued its inventory following a method not in line

with the provisions of Income Computation and Disclosure Standard (ICDS)- 2

‘Valuation of Inventories’.

In such a situation, discuss the relevant clause of Form No. 3CD under which the

tax auditor is required to report? (6 Marks)

6. (a) CARO, 2016 requires the auditor of the company to report whether maintenance of

cost records has been specified by the Central Government under section 148 of

the Companies Act, 2013 and whether such accounts and records have been so

prepared and maintained.

You are required to briefly explain the audit procedure to be followed by the auditor

and suggest the reporting pattern. (6 Marks)

(b) Vita Ltd. is anticipating taking over a manufacturing concern and appoints you for

due diligence review. While reviewing, it requests you to look specifically for any

hidden liabilities. State in brief the major areas you would examine for hidden

liabilities. (4 Marks)

(c) Mr. K is appointed as statutory auditor of NMO Ltd. NMO Ltd is required to appoint

internal auditor as per statutory provisions given in the Companies Act, 2013 and

appointed Mr. L as its internal auditor. The external auditor Mr. K asked internal

auditor to provide direct assistance to him regarding evaluating significant

accounting estimates by the management and assessing the risk of material

misstatements.

(i) Discuss whether Mr. K, statutory auditor, can ask direct assistance from Mr. L,

internal auditor as stated above in view of auditing standards.

(ii) Will your answer be different, if Mr. K asks direct assistance from Mr. L,

internal auditor with respect to external confirmation requests and evaluation of

the results of external confirmation procedures? (6 Marks)

Page 55: Test Series: August, 2017 MOCK TEST PAPER 1 FINAL COURSE ... · The company incurred losses of Rs. 40,000 and Rs. 60,000 in 2015-2016 and 2016-2017 after charging depreciation of

4

7. Write short notes on any four of the following:

(a) Mark to Market margins

(b) Advantages of Cost Audit to Government

(c) Special features involved in the audit of a Co-operative Society.

(d) Technical, professional and ethical standards as per statement on peer review

(e) Environmental Audit. (4 4 = 16 Marks)

Page 56: Test Series: August, 2017 MOCK TEST PAPER 1 FINAL COURSE ... · The company incurred losses of Rs. 40,000 and Rs. 60,000 in 2015-2016 and 2016-2017 after charging depreciation of

1

Test Series: August, 2017

MOCK TEST PAPER -1

FINAL COURSE: GROUP – I

PAPER – 3: ADVANCED AUDITING AND PROFESSIONAL ETHICS

SUGGESTED ANSWERS/ HINTS

1. (a) Inability to Continue as a Going Concern: As per SA 570 on “Going Concern”, it

is the responsibility of the auditor to obtain sufficient appropriate audit evidence

about the appropriateness of management’s use of the going concern assumption in

the preparation and presentation of the financial statements and to conclude

whether there is a material uncerta inty about the entity’s ability to continue as a

going concern. The auditor shall evaluate management’s assessment of the entity’s

ability to continue as a going concern. In evaluating management’s assessment, the

auditor shall consider whether management’s assessment includes all relevant

information of which the auditor is aware as a result of the audit.

In the instant case, S N Projects Limited has incurred massive loss during the year

2015-16 as the financial institutions have stopped financing additional working

capital to the company because of a complaint which was lodged against the

company by government for involvement in some irregularity relating to allotment of

coal blocks. There were delays in salary and other payments. Besides this, certai n

key managerial personnel, GM Finance and certain other employees have also left

the company. The company, in addition, has no sound action plan to mitigate these

situations.

Thus, there are clear indications that there is danger to entity’s ability to continue in

future. Considering the fact that there is no sound plan of action to mitigate these

factors, the going concern assumption does not seem appropriate.

Therefore, the auditor should ask the management for its adequate disclosure in the

financial statement and include the same in his report. However, if the management

fails to make adequate disclosure, the auditor should express Express a qualified

opinion or adverse opinion

(b) Risk Assessment: The auditor in accordance with SA 315 “Identifying and

Assessing the Risks of Material Misstatement through Understanding the Entity and

its Environment”, should make an assessment of inherent and control risk for

material financial statement assertions.

In a CIS environment the risk of a Material financial statement ascertain being

erroneously stated could arise from the deficiencies in the following case as -

(i) Program Development and maintenance.

(ii) System software support.

Page 57: Test Series: August, 2017 MOCK TEST PAPER 1 FINAL COURSE ... · The company incurred losses of Rs. 40,000 and Rs. 60,000 in 2015-2016 and 2016-2017 after charging depreciation of

2

(iii) Operations including processing of data.

(iv) Physical CIS security.

(v) Control over access to specialized utility program.

These deficiencies would tend to have a negative impact on all application systems

that are processed through the computer.

(c) Positive External Confirmation: As per SA 505, “External Confirmation”, factors to

be considered when designing confirmation requests include:

(i) The assertions being addressed.

(ii) Specific identified risks of material misstatement, including fraud risks.

(iii) The layout and presentation of the confirmation request.

(iv) Prior experience on the audit or similar engagements.

(v) The method of communication (for example, in paper form, or by electronic or

other medium).

(vi) Management’s authorisation or encouragement to the confirming parties to

respond to the auditor. Confirming parties may only be willing to respond to a

confirmation request containing management’s authorisation.

(vii) The ability of the intended confirming party to confirm or provide the requested

information (for example, individual invoice amount versus total balance).

A positive external confirmation request asks the confirming party to reply to the

auditor in all cases, either by indicating the confirming party’s agreement wi th the

given information, or by asking the confirming party to provide information. A

response to a positive confirmation request ordinarily is expected to provide reliable

audit evidence. There is a risk, however, that a confirming party may reply to the

confirmation request without verifying that the information is correct. The auditor

may reduce this risk by using positive confirmation requests that do not state the

amount (or other information) on the confirmation request, and ask the confirming

party to fill in the amount or furnish other information. On the other hand, use of this

type of “blank” confirmation request may result in lower response rates because

additional effort is required of the confirming parties.

(d) Basis for the Auditor’s Opinion on the Summary Financial Statements: As per

SA 810, “Engagement to Report on Summary Financial Statements”, the auditor

shall perform the following procedures, and any other procedures that the auditor

may consider necessary, as the basis for the auditor’s opinion on the summary

financial statements:

(i) Evaluate whether the summary financial statements adequately disclose their

summarised nature and identify the audited financial statements.

Page 58: Test Series: August, 2017 MOCK TEST PAPER 1 FINAL COURSE ... · The company incurred losses of Rs. 40,000 and Rs. 60,000 in 2015-2016 and 2016-2017 after charging depreciation of

3

(ii) When summary financial statements are not accompanied by the audited

financial statements, evaluate whether they describe clearly:

(1) From whom or where the audited financial statements are available; or

(2) The law or regulation that specifies that the audited financial statements

need not be made available to the intended users of the summary

financial statements and establishes the criteria for the preparation of the

summary financial statements.

(iii) Evaluate whether the summary financial statements adequately disclose the

applied criteria.

(iv) Compare the summary financial statements with the related information in the

audited financial statements to determine whether the summary financial

statements agree with or can be re-calculated from the related information in

the audited financial statements.

(v) Evaluate whether the summary financial statements are prepared in

accordance with the applied criteria.

(vi) Evaluate, in view of the purpose of the summary financial statements, whether

the summary financial statements contain the information necessary, and are

at an appropriate level of aggregation, so as not to be misleading in the

circumstances.

(vii) Evaluate whether the audited financial statements are available to the intended

users of the summary financial statements without undue difficulty, unless law

or regulation provides that they need not be made available and establishes

the criteria for the preparation of the summary financial statements.

2. (a) Sharing of Audit Fees with non-member: As per Clause (2) of Part I of First

Schedule of the Chartered Accountants Act, 1949, a Chartered Accountant in

practice is deemed to be guilty of professional misconduct if he pays or allows or

agrees to pay or allow, directly or indirectly, any share, commission or broke rage in

the fees or profits of his professional business, to any person other than a member

of the Institute or a partner or a retired partner or the legal representative of a

deceased partner, or a member of any other professional body or with such other

persons having such qualification as may be prescribed, for the purpose of

rendering such professional services to time in or outside India.

In the instant case, CA Ciya, a practicing Chartered Accountant gave 60% of the

audit fees received by her to Mr. Suhas, who was not a Chartered Accountant,

under the nomenclature of office allowance. In this case, it is not the nomenclature

to a transaction that is material but it is the substance of the transaction, which has

to be looked into.

Page 59: Test Series: August, 2017 MOCK TEST PAPER 1 FINAL COURSE ... · The company incurred losses of Rs. 40,000 and Rs. 60,000 in 2015-2016 and 2016-2017 after charging depreciation of

4

Therefore, CA Ciya will be held guilty of professional misconduct under clause (2) of

Part I of First Schedule of Chartered Accountants Act, 1949.

(b) Disrepute to the Profession: As per Clause (2) of Part IV of First Schedule to the

Chartered Accountants Act, 1949, a member of the Institute, whether in practice or

not, shall be deemed to be guilty of other misconduct, if he, in the opinion of the

Council, brings disrepute to the profession or to the Institute as a result of his action

whether or not related to his professional work.

Accordingly, a Chartered Accountant is also expected to maintain the highest

standards and integrity even in his personal affairs and any deviation from these

standards calls for disciplinary action.

In the given case, CA Raj, a Chartered Accountant in practice approached Manager

of a Nationalised Bank for a loan and offered for filing Income tax Returns without

any fees. This approach of CA Raj brings disrepute to the profession of a Chartered

Accountant.

Hence, CA Raj will be held guilty of other misconduct under Clause (2) of Part IV of

the First Schedule of the Chartered Accountants Act, 1949.

(c) Advertisement: As per Clause (7) of Part I of First Schedule of the Chartered

Accountants Act, 1949, a Chartered Accountant in practice is deemed to be guilty of

professional misconduct if he (i) advertises his professional attainments or services

or (ii) uses any designation or expressions other than “Chartered Accountant” on

professional documents, visiting cards, letter heads or sign boards unless it be a

degree of a university established by law in India or recognized by the Central

Government or a title indicating membership of the ICAI or of any other institution

that has been recognized by the Central Government or may be recognized by the

council.

Here there is no prohibition for printing names on the personal letter heads in which

a member holding certificate of practice is a partner.

In the given case, CA Naman mentioned the names and addresses of all the 4 firms

in which he is a partner on the personal letter heads. Therefore, CA Naman will not

be held guilty of any professional misconduct in pursuant to Clause (7) of Part I of

First Schedule of the Chartered Accountants Act, 1949.

(d) Not exercising due diligence: According to Clause (7) of Part I of Second

Schedule of Chartered Accountants Act, 1949, a Chartered Accountant in practice

shall be deemed to be guilty of professional misconduct if he does not exercise due

diligence, or is grossly negligent in the conduct of his professional duties.

It is a vital clause which unusually gets attracted whenever it is necessary to judge

whether the accountant has honestly and reasonably discharged his duties. The

expression negligence covers a wide field and extends from the frontiers of fraud to

collateral minor negligence.

Page 60: Test Series: August, 2017 MOCK TEST PAPER 1 FINAL COURSE ... · The company incurred losses of Rs. 40,000 and Rs. 60,000 in 2015-2016 and 2016-2017 after charging depreciation of

5

Where a Chartered Accountant had not completed his work relating to the audit of

the accounts a company and had not submitted his audit report in due time to

enable the company to comply with the statutory requirement in this regard, it was

held, under a case, that he was guilty of professional misconduct under Clause (7).

In the given case, CA. Dev has not completed his audit work in time and

consequently could not submit audit report in due time and consequently, company

could not comply with the statutory requirements.

Therefore, CA. Dev will be held guilty of professional misconduct under Clause (7)

of Part I of the Second Schedule of the Chartered Accountants Act, 1949 .

3. (a) Auditing Standards of the Public Company Accounting Oversight Board

(PCAOB) has the following key requirements:

(i) The design of controls-relevant assertions related to all significant accounts

and disclosures in the financial statements;

(ii) Information about how significant transactions are initiated, authorized,

supported, processed, and reported;

(iii) Enough information about the flow of transactions to identify where material

misstatements due to error or fraud could occur;

(iv) Controls designed to prevent or detect fraud, including who performs the

controls and the regulated segregation of duties;

(v) Controls over the period-end financial reporting process;

(vi) Controls over safeguarding of assets;

(vii) The results of management's testing and evaluation.

(b) Basic system of Control: Internal Checks and Internal Audit are important

constituents of Accounting Controls. Internal check system implies organization of

the overall system of book-keeping and arrangement of Staff duties in such a way

that no one person can carry through a transaction and record every aspect

thereof.

In the given case of Modest Hospital, the person-in-charge of inventory inflow and

outflow from the store house is also responsible for purchases and maintaining

inventory records. Thus, one of the basic system of control i.e. internal check which

includes segregation of duties or maker and checker has been violated where

transaction processing are allocated to different persons in such a manner that no

one person can carry through the completion of a transaction from start to finish or

the work of one person is made complimentary to the work of another person.

The general condition pertaining to the internal check system may be

summarized as under-

(i) No single person should have complete control over any important aspect of

Page 61: Test Series: August, 2017 MOCK TEST PAPER 1 FINAL COURSE ... · The company incurred losses of Rs. 40,000 and Rs. 60,000 in 2015-2016 and 2016-2017 after charging depreciation of

6

the business operation. Every employee’s action should come under the

review of another person.

(ii) Staff duties should be rotated from time to time so that members do not

perform the same function for a considerable length of time.

(iii) Every member of the staff should be encouraged to go on leave at least once a

year.

(iv) Persons having physical custody of assets must not be permitted to have

access to the books of accounts.

(v) There should exist an accounting control in respect of each class of assets, in

addition, there should be periodical inspection so as to establish their

physical condition.

(vi) Mechanical devices should be used, where ever practicable to prevent loss or

misappropriation of cash.

(vii) Budgetary control should be exercised and wide deviations observed should

be reconciled.

(viii) For inventory taking, at the close of the year, trading activities should, if

possible be suspended, and it should be done by staff belonging to several

sections of the organization.

(ix) The financial and administrative powers should be distributed very judiciously

among different officers and the manner in which those are actually exercised

should be reviewed periodically.

(x) Procedures should be laid down for periodical verification and testing of

different sections of accounting records to ensure that they are accurate.

(c) Formulation of Audit Strategy: While formulating the audit strategy for a company,

following factors may be considered -

General Factors:

(i) The engagement objectives.

(ii) The results of the business review, including major developments in the

client’s business and industry, significant operating results and financial

arrangements.

(iii) Preliminary judgements as to materiality.

(iv) Identified inherent risks. The team should also consider the risk of fraud and,

in particular, any evidence of a high level of risk to the firm. They should take

into account the results of procedures for the acceptance and continuation of

clients.

(v) The degree to which the team should carry out further assessment of controls

Page 62: Test Series: August, 2017 MOCK TEST PAPER 1 FINAL COURSE ... · The company incurred losses of Rs. 40,000 and Rs. 60,000 in 2015-2016 and 2016-2017 after charging depreciation of

7

as a means of reducing substantive tests.

(vi) The broad nature, extent and timing of substantive tests, or changes to the

previous year’s strategy for substantive testing.

(vii) Main points relating to planning and controlling the audit or comments on the

adequacy of the existing arrangements.

Specific Factors for Online Shopping:

The auditor shall also obtain an understanding of the information system including

the related business processes due to new venture of online shopping in the

following areas:

(i) The classes of transactions in the entity’s operations tha t are significant to the

financial statements;

(ii) The procedures, within both information technology (IT) and manual systems,

by which those transactions are initiated, recorded, processed, corrected as

necessary, transferred to the general ledger and reported in the financial

statements;

(iii) The related accounting records, supporting information and specific accounts

in the financial statements that are used to initiate, record, process and report

transactions; this includes the correction of incorrect information and how

information is transferred to the general ledger. The records may be in either

manual or electronic form;

(iv) How the information system captures events and conditions, other than

transactions, that are significant to the financial s tatements;

(v) Controls surrounding journal entries, including non-standard journal entries

used to record non-recurring, unusual transactions or adjustments.

4. (a) (i) Non-cash Transactions with Relative of Director: As per Clause (xv) of

paragraph 3 of CARO, 2016, the auditor is required to report “whether the

company has entered into any non-cash transactions with directors or persons

connected with him and if so, whether the provisions of section 192 of

Companies Act, 2013 have been complied with”.

Section 192 of the said Act deals with restriction on non-cash transactions

involving directors or persons connected with them. The section prohibits the

company from entering into such types of arrangements unless it is an

arrangement by which the company acquires or is to acquire assets for

consideration other than cash, from such director or person so connected.

In the instant case, JKL Ltd. has entered into non-cash transactions with Mr.

Rahul, son of director which is an arrangement by which JKL Ltd. is in process

to acquire assets for consideration other than cash. In the above situation the

Page 63: Test Series: August, 2017 MOCK TEST PAPER 1 FINAL COURSE ... · The company incurred losses of Rs. 40,000 and Rs. 60,000 in 2015-2016 and 2016-2017 after charging depreciation of

8

provisions of section 192 of Companies Act, 2013 have been complied with.

However, the reporting requirements under this clause are given in two parts.

The first part requires the auditor to report on whether the company has

entered into any non-cash transactions with the directors or any persons

connected with such director/s. The second part of the clause requires the

auditor to report whether the provisions of section 192 of the Act have been

complied with. Therefore, the second part of the clause becomes reportable

only if the answer to the first part is in affirmative.

In the given situation, JKL Ltd. has entered into non-cash transactions with Mr.

Rahul, son of director which is affirmative answer to the first part of the Clause

(xv) of Paragraph 3 of CARO, 2016, thus, reporting is required for the same.

Draft report is given below.

According to the information and explanations given to us, the Company has

entered into non-cash transactions with Mr. Rahul, son of one of the directors

during the year, for the acquisition of assets, which in our opinion is covered

under the provisions of Section 192 of the Companies Act, 2013.

(ii) Title deeds of Immovable Property in the name of Director: As per Clause

(i)(c) of Paragraph 3 of the CARO, 2016, the auditor is required to report on

whether the title deeds of immovable properties are held in the name of the

company. If not, provide the details thereof.

The auditor should verify the title deeds available and reconcile the same with

the fixed assets register. The scrutiny of the title deeds of the immovable

property may reveal a number of discrepancies between the details in the fixed

assets register and the details available in the title deeds. This may be due to

various reasons which needs to be examined.

In the given case, STP Limited has its factory building, appearing as fixed

assets in its financial statements in the name of director. Thus, the auditor

shall report on the same under Clause (i)(c) of Paragraph 3 of the CARO,

2016.

The reporting under this clause, where the title deeds of the immovable

property are not held in the name of the Company, may be made incorporating

following details, in the form of a table or otherwise:

A In case of land:-

total number of cases,

whether leasehold / freehold,

gross block and net block, (as at Balance Sheet date), and

remarks, if any.

Page 64: Test Series: August, 2017 MOCK TEST PAPER 1 FINAL COURSE ... · The company incurred losses of Rs. 40,000 and Rs. 60,000 in 2015-2016 and 2016-2017 after charging depreciation of

9

B In case of Buildings:-

total number of cases,

gross block & net block, (as at Balance Sheet date) and

remarks, if any.

(b) (i) Compliance Requirements for Code of Conduct as per Listing Order

Disclosure Requirement (LODR) Regulations:

(1) The Board shall lay down a code of conduct for all Board members and

senior management of the listed entity.

(2) All Board members and senior management personnel shall affirm

compliance with the code on an annual basis.

(3) The Code of Conduct shall suitably incorporate the duties of Independent

Directors as laid down in the Companies Act, 2013.

(4) The code of conduct shall be posted on the website of the company.

(5) The Annual Report of the company shall contain a declaration to this

effect signed by the CEO.

The term “senior management” shall mean personnel of the listed entity who

are members of its core management team excluding Board of Directors.

Normally, this would comprise all members of management one level below

the executive directors, including all functional heads.

(ii) Role of Auditor: The auditor should ascertain-

(1) whether the Board of Directors of the company has laid down a Code of

Conduct for all Board members and senior personnel of the company and

obtain a copy of the same.

(2) He should also verify whether all Board members and senior

management personnel have affirmed compliance with the code on an

annual basis and whether the code has been posted on company’s

website.

5. (a) Statutory Liquidity Ratio (SLR) Requirements: The Reserve Bank of India

requires statutory central auditors of banks to verify the compliance with SLR

requirements of 12 odd dates in different months of a financial year not being

Fridays. The resultant report is to be sent to the top management of the bank and to

the Reserve Bank. The report of the statutory auditors in relation to compliance with

SLR requirements has to cover two aspects-

(i) correctness of the compilation of DTL (Demand and Time Liabilities) position;

and

(ii) maintenance of liquid assets.

Page 65: Test Series: August, 2017 MOCK TEST PAPER 1 FINAL COURSE ... · The company incurred losses of Rs. 40,000 and Rs. 60,000 in 2015-2016 and 2016-2017 after charging depreciation of

10

Audit Approach and Procedure:

(i) Obtain an understanding of the relevant circulars of the RBI, particularly

regarding composition of items of DTL.

(ii) Require the branch auditors to send their weekly trial balance as on Friday and

these are consolidated at the head office. Based on this consolidation, the DTL

position is determined for every reporting Friday. The statutory central auditor

should request the branch auditors to verify the correctness of the trial

balances relevant to the dates selected by him. The branch auditors should

also be specifically requested to examine the cash balance at the branch on

the selected dates.

(iii) Examine, on a test basis, the consolidations regarding DTL position prepared

by the bank with reference to the related returns received from branches. The

auditor should examine whether the valuation of securities done by the bank is

in accordance with the guidelines prescribed by the RBI.

(iv) While examining the computation of DTL, specifically examine that the

following items have been excluded from liabilities-

(a) Part amounts of recoveries from the borrowers in respect of debts

considered bad and doubtful of recovery.

(b) Amounts received in Indian currency against import bills and held in

sundry deposits pending receipts of final rates.

(c) Un-adjusted deposits/balances lying in link branches for agency business

like dividend warrants, interest warrants, refund of application money,

etc., in respect of shares/debentures to the extent of payment made by

other branches but not adjusted by the link branches.

(d) Margins held and kept in sundry deposits for funded facilities-

(v) Similarly, specifically examine that the following items have been included in

liabilities-

(a) Net credit balance in branch adjustment accounts including these relating

to foreign branches.

(b) Interest on deposit as at the end of the firm half year reversed in the

beginning of the next half-year.

(c) Borrowings from abroad by banks in India needs to be considered as

‘liabilities to other’ and thus, needs to be considered at gross level unlike

‘liabilities towards banking system in India’, which are permitted to be

netted off against ‘assets towards banking system in India’. Thus, the

adverse balances in Nostro Mirror Account needs to be considered as

‘Liabilities to other’

Page 66: Test Series: August, 2017 MOCK TEST PAPER 1 FINAL COURSE ... · The company incurred losses of Rs. 40,000 and Rs. 60,000 in 2015-2016 and 2016-2017 after charging depreciation of

11

(d) The reconciliation of Nostro accounts (with Nostro Mirror Accounts) needs

to be scrutinized carefully to analyze and ascertain if any inwards

remittances are received on behalf of the customers / constituents of the

bank and have remained unaccounted and / or any other debit (inward)

entries have remained unaccounted and are pertaining to any liabilities

for the bank.

(vi) Examine whether the consolidations prepared by the bank include the relevant

information in respect of all the branches.

(vii) It may be noted that, even though interest accrues on a daily basis, it is

recorded in the books only at periodic intervals. Thus, examine whether such

interest accrued but not accounted for in books is included in the computation

of DTL.

(viii) The auditor at the central level should apply the audit procedures listed above

to the overall consolidation prepared for the bank as a whole. Where such

procedure is followed, the central auditor should adequately describe the same

in his report.

(viii) While reporting on compliance with SLR requirements, the auditor should

specify the number of unaudited branches and state that he has relied on the

returns received from the unaudited branches in forming his opinion. Recently,

there has been introduction of Automated Data Flow (ADF) for CRR & SLR

reporting and the auditors should develop necessary audit procedures around

this.

(b) Reporting under CARO, 2016 for Registration under section 45-IA of the RBI

Act, 1934: As per Clause (xvi) of paragraph 3 of the CARO, 2016, the auditor is

required to report whether the company is required to be registered under section

45-IA of the Reserve Bank of India Act, 1934. If so, whether the registration has

been obtained.

Audit Procedures and Reporting-

(a) The auditor should examine the transactions of the company with relation to

the activities covered under the RBI Act and directions related to the Non-

Banking Financial Companies.

(b) The financial statements should be examined to ascertain whether company’s

financial assets constitute more than 50 per cent of the total assets and

income from financial assets constitute more than 50 per cent of the gross

income.

(c) Whether the company has net owned funds as required for the registration as

NBFC.

(d) Whether the company has obtained the registration as NBFC, if not, the

Page 67: Test Series: August, 2017 MOCK TEST PAPER 1 FINAL COURSE ... · The company incurred losses of Rs. 40,000 and Rs. 60,000 in 2015-2016 and 2016-2017 after charging depreciation of

12

reasons should be sought from the management and documented.

(e) The auditor should report incorporating the following:-

(i) Whether the registration is required under section 45-IA of the RBI Act,

1934.

(ii) If so, whether it has obtained the registration.

(iii) If the registration not obtained, reasons thereof.

(c) Reporting for Adjustment to be made to the Profits or Loss for complying with

ICDSs: Clause 13(d) of Form No. 3CD of the tax audit report requires the tax

auditor to state whether any adjustment is required to be made to the profits or

loss for complying with the provisions of income computation and disclosure

standards notified under section 145(2) of the Income Tax Act, 1961.

Further, the tax auditor is also required to report under Clause 13(e ), if answer to

Clause 13(d) above is in the affirmative i.e. the auditor is required to give details of

such adjustments as follows:

Increase in profit (Rs.)

Decrease in profit (Rs.)

Net effect

(Rs.)

ICDS I Accounting Policies

ICDS II Valuation of Inventories

ICDS III Construction Contracts

ICDS IV Revenue Recognition

ICDS V Tangible Fixed Assets

ICDS VI Changes in Foreign Exchange Rates

ICDS VII Governments Grants

ICDS VIII Securities

ICDS IX Borrowing Costs

ICDS X Provisions, Contingent Liabilities and Contingent Assets

Total

Furthermore, Clause 13(f) also requires the tax auditor for disclosure of the

following as per ICDS:

(i) ICDS I-Accounting Policies

Page 68: Test Series: August, 2017 MOCK TEST PAPER 1 FINAL COURSE ... · The company incurred losses of Rs. 40,000 and Rs. 60,000 in 2015-2016 and 2016-2017 after charging depreciation of

13

(ii) ICDS II-Valuation of Inventories

(iii) ICDS III-Construction Contracts

(iv) ICDS IV-Revenue Recognition

(v) ICDS V-Tangible Fixed Assets

(vi) ICDS VII-Governments Grants

(vii) ICDS IX Borrowing Costs

(viii) ICDS X-Provisions, Contingent Liabilities and Contingent Assets”.

6. (a) Reporting of Cost Records under CARO, 2016: The auditor is required to report

as per clause (vi) to Paragraph 3 of the CARO, 2016, “Whether maintenance of cost

records has been specified by the Central Government under sub-section (1) of

Section 148 of the Companies Act, 2013 and whether such accounts and records

have been so made and maintained.”

Audit Procedures and Reporting-

The Order requires the auditor to report whether cost accounts and records have

been made and maintained. The word “made” applies in respect of cost accounts

(or cost statements) and the word “maintained” applies in respect of cost records

relating to materials, labour, overheads, etc. The auditor has to report under the

clause irrespective of whether a cost audit has been ordered by the central

government.

The auditor should obtain a written representation from the management stating:

(a) whether cost records are required to be maintained for any product(s) or

services of the company under section 148 of the Act, and the Companies

(Cost Records and Audit) Rules, 2014; and

(b) whether cost accounts and records are being made and maintained regularly.

The auditor should also obtain a list of books/records made and maintained in this

regard. The Order does not require a detailed examination of such records. The

auditor should, therefore, conduct a general review of the cost records to ensure

that the records as prescribed are made and maintained. He should, of course,

make such reference to the records as is necessary for the purposes of his audit.

It is necessary that the extent of the examination made by the auditor is clearly

brought out in his report. The following wording is, therefore, suggested:

“We have broadly reviewed the books of account maintained by the company

pursuant to the Rules made by the Central Government for the maintenance of cost

records under section 148 of the Act, and are of the opinion that prima facie, the

prescribed accounts and records have been made and maintained.”

Page 69: Test Series: August, 2017 MOCK TEST PAPER 1 FINAL COURSE ... · The company incurred losses of Rs. 40,000 and Rs. 60,000 in 2015-2016 and 2016-2017 after charging depreciation of

14

Where the auditor finds that the records have not been written or are not prima f acie

complete, it will be necessary for the auditor to make a suitable comment in his

report.

(b) Major areas to examine in course of Due Diligence Review: 'Due Diligence' is a

term that is often heard in the corporate world these days in relation to corporate

restructuring. The purpose of due diligence is to assist the purchaser or the investor

in finding out all he can, reasonably about the business he is acquiring or investing

in prior to completion of the transaction including its critical success facto rs as well

as its strength and weaknesses.

Due diligence is an all pervasive exercise to review all important aspects like

financial, legal, commercial, etc. before taking any final decision in the matter. As

far as any hidden liabilities or overvalued assets are concerned, this shall form part

of such a review of Financial Statements. Normally, cases of hidden liabilities and

overvalued assets are not apparent from books of accounts and financial

statements. Review of financial statements does not involve examination from the

view point of extraordinary items, analysis of significant deviations, etc.

However, in order to investigate hidden liabilities, the auditor should pay his

attention to the following areas:

The company may not show any show cause notices which have not matured

into demands, as contingent liabilities. These may be material and important.

The company may have given “Letters of Comfort” to banks and Financial

Institutions. Since these are not “guarantees”, these may not be disclosed in

the Balance sheet of the target company.

The Company may have sold some subsidiaries/businesses and may have

agreed to take over and indemnify all liabilities and contingent liabilities of the

same prior to the date of transfer. These may not be reflected in the books of

accounts of the company.

Product and other liability claims; warranty liabilities; product returns/discounts;

liquidated damages for late deliveries etc. and all litigation.

Tax liabilities under direct and indirect taxes.

Long pending sales tax assessments.

Pending final assessments of customs duty where provisional assessment only

has been completed.

Agreement to buy back shares sold at a stated price.

Future lease liabilities.

Environmental problems/claims/third party claims.

Page 70: Test Series: August, 2017 MOCK TEST PAPER 1 FINAL COURSE ... · The company incurred losses of Rs. 40,000 and Rs. 60,000 in 2015-2016 and 2016-2017 after charging depreciation of

15

Unfunded gratuity/superannuation/leave salary liabilities; incorrect gratuity

valuations.

Huge labour claims under negotiation when the labour wage agreement has

already expired.

Contingent liabilities not shown in books.

(c) (i) Direct Assistance from Internal Auditor: As per SA 610 “Using the Work of

Internal Auditor”, the external auditor shall not use internal auditors to provide

direct assistance to perform procedures that Involve making significant

judgments in the audit.

Since the external auditor has sole responsibility for the audit opinion

expressed, the external auditor needs to make the significant judgments in the

audit engagement.

Significant judgments include the following:

Assessing the risks of material misstatement;

Evaluating the sufficiency of tests performed;

Evaluating the appropriateness of management’s use of the going

concern assumption;

Evaluating significant accounting estimates; and

Evaluating the adequacy of disclosures in the financial statements, and

other matters affecting the auditor’s report.

In view of above, Mr. K cannot ask direct assistance from internal auditors

regarding evaluating significant accounting estimates and assessing the risk of

material misstatements.

(ii) Direct Assistance from Internal Auditor in case of External Confirmation

Procedures: SA 610 “Using the Work of Internal Auditor”, provide relevant

guidance in determining the nature and extent of work that may be assigned to

internal auditors. In determining the nature of work that may be assigned to

internal auditors, the external auditor is careful to limit such work to those

areas that would be appropriate to be assigned.

Further, in accordance with SA 505, “External Confirmation” the external

auditor is required to maintain control over external confirmation requests and

evaluate the results of external confirmation procedures, it would not be

appropriate to assign these responsibilities to internal auditors. However,

internal auditors may assist in assembling information necessary for the

external auditor to resolve exceptions in confirmation responses.

Page 71: Test Series: August, 2017 MOCK TEST PAPER 1 FINAL COURSE ... · The company incurred losses of Rs. 40,000 and Rs. 60,000 in 2015-2016 and 2016-2017 after charging depreciation of

16

7. (a) Mark to Market Margin (MTM): MTM margin is the notional loss, which a stock

member or his client would incur, if the net cumulative outstanding positions in all

securities were closed out at the closing price of the relevant trading day, which is

different from the price at which the transaction had been entered into. For each

security, this is worked out by multiplying the difference between the closing price

and the price at which the trade was executed by the cumulative buy and sell open

position (for buy position the close price being lower than actual trade price and for

sell position the close price being higher than actual trade price). The aggregate

amount computed across all securities is MTM margin payable by a member. The

mark-to-market margin is payable with reference to net position at client’s level.

(b) Advantages of Cost Audit to Government: Cost auditor’s approach is to ensure

that the cost accounting plan is in consonance with the objectives set by the

organisation and the system of accounting is geared towards the attainment of the

objectives. A cost accounting system designed to exercise control over cost may be

different from the one if the objective is to fix price. Accordingly, over a period of

time particularly in view of administered pricing system the cost accounting

becomes quite important. Some of the specific advantages which can be reaped by

the Government are:

(i) Where the Government enters into a cost-plus contract, cost audit helps

government to fix the price of the contract at a reasonable level.

(ii) Cost audit helps in the fixation of selling prices of essential commodities and

thus undue profiteering is checked.

(iii) Cost audit enables the government to focus its attention on inefficient units.

(iv) Cost audit enables the government to decide in favour of giving protection to

certain industries.

(v) Cost audit facilitates settlement of trade disputes brought to the government.

(vi) Cost audit and consequent management action can create a healthy

competition among the various units in an industry. This imposes an automatic

check on inflation.

(c) Audit of a Co-operative Society: Special features to be borne in mind in general

while conducting the audit of a Co-operative Society are as follows-

(i) Examination of overdue debts.

(ii) Special treatment of overdue interest in the determination of profit.

(iii) Certification of bad debts before being written off.

(iv) Valuation of Assets and Liabilities.

(v) Adherence to co-operative Principles.

(vi) Observations of the Provisions of the Act and Rules.

Page 72: Test Series: August, 2017 MOCK TEST PAPER 1 FINAL COURSE ... · The company incurred losses of Rs. 40,000 and Rs. 60,000 in 2015-2016 and 2016-2017 after charging depreciation of

17

(vii) Verification of members’ Register and examination of their pass books .

(viii) Special report to the registrar on fraud, mismanagement and personal

profiteering.

(ix) Audit classification of society based on the audit findings.

(x) Discussion of draft audit report with the managing committee.

(d) Technical, Professional and Ethical Standards as per Statement on Peer

Review: As per the Statement on Peer Review, Technical, Professional and Ethical

Standards means-

(i) Accounting Standards issued by ICAI and /or prescribed and notified by the

Central Government of India;

(ii) Standards issued by the Institute of Chartered Accountants of India including

(a) Engagement standards

(b) Statements

(c) Guidance notes

(d) Standards on Internal Audit

(e) Statements on Quality Control

(f) Notifications/ Directions/ Announcements/ Guidelines/ Pronouncements/

Professional standards issued from time to time by the Council or any of

its committees.

(iii) Framework for the Preparation and presentation of financial statements,

framework of statements and Standard on Auditing, Standard on Assurance

Engagements, Standards on Quality Control and Guidance Notes on related

services issued, from time to time, by the Institute of Chartered Accountants of

India and framework for assurance engagements;

(iv) Provisions of the various relevant statutes and / or regulations which are

applicable in the context of the specific engagements being Reviewed

including instructions, guidelines, notifications, directions issued by regulatory

bodies as covered in the scope of assurance engagements.

(e) Environmental Audit: Environmental reporting deals with the disclosure by an

entity of environmentally related data, regarding environmental risks, environmental

impacts, policies, strategies, targets, costs, liabilities or environmental performance

to those who have an interest in such information as an aid to enabling/enriching

their relationship with the reporting entity via either the annual report; a stand -alone

corporate environmental performance report; or some other medium (e.g. staff

newsletter, video, CD ROM, internet site). The reports that are generated after such

Page 73: Test Series: August, 2017 MOCK TEST PAPER 1 FINAL COURSE ... · The company incurred losses of Rs. 40,000 and Rs. 60,000 in 2015-2016 and 2016-2017 after charging depreciation of

18

audits can be either compliance-based reporting or impact-based performance

reporting.

Environmental audit deals with verif ication of information contained in such reports

with a view to expressing an opinion thereon. Environmental audit can be performed

by external agencies or internal experts (including internal auditors). In practice,

environmental audit is not done by a single agency but by various agencies who are

experts in the field. Since the subject matter of environmental audit involves multi -

disciplinary knowledge and skill, it is preferable to form a team of persons drawn

from different disciplines who may assist the chartered accountant in performing the

task in an effective manner, generally environmental audits are not required by any

statute but are sometimes done at the request of the management to address

issues like compliance with environmental laws and regulations, etc.

Page 74: Test Series: August, 2017 MOCK TEST PAPER 1 FINAL COURSE ... · The company incurred losses of Rs. 40,000 and Rs. 60,000 in 2015-2016 and 2016-2017 after charging depreciation of

Test Series: August, 2017

MOCK TEST PAPER 1

FINAL COURSE: GROUP –I

PAPER – 4: CORPORATE AND ALLIED LAWS

Question No.1 is compulsory.

Attempt any five questions from the remaining six Questions.

Time Allowed – 3 Hours Maximum Marks – 100

1. (a) Construction Ltd. took up a project of the National Highways Authority of India on the

basis of a guarantee provided by the Central Government regarding payment of

dividends to the shareholders of the company. Can the company pay dividends out

of the above guaranteed money? Referring to the applicable provisions of the

Companies Act, 2013, advise the company regarding the sources out of which the

company can pay dividends. (5 Marks)

(b) Mr. Competent is an auditor of Capable Ltd. While auditing the accounts of the

Capable Ltd. for 2016-2017, he finds manipulation of funds around 2 crore committed

by the officers of the company against the Capable Ltd. Examine in the light of the

Companies Act, 2013 the frauds required to be reported by Mr. Competent .

(5 Marks)

(c) Mr. Z was appointed as the presiding officer of the Securities Appellate Tribunal (SAT)

for a term of five years. Considering his good performances during the tenure of his

service, it was proposed by the competent authority to re-appoint him for another

term. During reappointment, he has completed good 69 years of age. Examine in the

light of the SEBI Act, 1992, the eligibility of his appointment/ reappointment as a

presiding officer in the Securities Appellate Tribunal. (5 Marks)

(d) Mr. Azad, an independent director of X company, was appointed in the AGM for a

period of three years. After the expiry of 3 years he was re-appointed for a period of

5 years. Considering that though Mr. Azad has completed two tenures but hasn’t

completed ten years in total, therefore he may be appointed in the upcoming AGM for

another 2 years to complete his total term of 10 years. Conferring in the light of the

Companies Act, 2013, state the validity of reappointment of Mr. Azad for further term

in the company. (5 Marks)

Page 75: Test Series: August, 2017 MOCK TEST PAPER 1 FINAL COURSE ... · The company incurred losses of Rs. 40,000 and Rs. 60,000 in 2015-2016 and 2016-2017 after charging depreciation of

2

2. (a) (i) ABC Limited is a wholly owned subsidiary company of XYZ Limited. The

Company wants to make application for merger of Holding and Subsidiary

Companies under Section 232. The Company Secretary of the XYZ Limited is of

the opinion that company cannot apply for merger as per section 232. The

company shall have to apply for merger as per section 233 i.e. Fast Track

Merger. Is the contention of Company Secretary being valid as per law?

(5 Marks)

(ii) Provide various grounds on which the investigation is assigned to Serious Fraud

Investigation Office? (3 Marks)

(b) State the legal position in the given situations:

(i) Mr. Khan appointed as an Independent Director on the Board of a company, be

appointed in its subsidiary or its holding or its associate company.

(ii) Mr. D, proposes his candidature as a director in X Ltd. along with the deposit of

1 lac rupees. Later Mr. D failed to be appointed but received 30 % of total votes.

D asked X Ltd. to refund the deposit but the company denied to pay as he failed

to be elected. (8 Marks)

3. (a) (i) While counting the limit of 20 for holding directorship under section 165 should

section 8 companies and foreign body corporate (whether holding or subsidiary

or otherwise) be also included?

(ii) Xyz, a small company held its Board meetings on 1st January, 2016,

31st December 2016 and 1st January, 2017 then what is the due date of next

board meeting? (8 Marks)

(b) Advise the Board of Director of Spectra Papers Ltd. regarding validity and extent of

their powers, under the provisions of the Companies Act, 2013 in relation to the

following matters:

(i) Buy-back of the shares of the Company, for the first time, upto 10% of the paid

up equity share capital without passing a special resolution.

(ii) Delegation of Power to the Managing Director of the company to invest surplus

funds of the company in the shares of some companies. (8 Marks)

4. (a) (i) XYZ Ltd is a listed company having turnover of ` 1200 crores during the financial

year 2015-16. The CSR committee of the Board formulated and recommended

a CSR project which was approved by the Board. Company finalised the project

under its CSR initiatives which require funds @ 5 % of average net profi t of the

company for last three financial years. Will such excess expense be counted in

subsequent financial years as a part of CSR expenditure? Advise.

Page 76: Test Series: August, 2017 MOCK TEST PAPER 1 FINAL COURSE ... · The company incurred losses of Rs. 40,000 and Rs. 60,000 in 2015-2016 and 2016-2017 after charging depreciation of

3

(ii) A Ltd. has one subsidiary i.e. B Ltd.& one associate company i.e. C Ltd. B Ltd.

is the entity incorporated outside India and C Ltd, is the entity incorporated in

India. State the law with respect to the consolidation of financial statements in

the given case. (8 Marks)

(b) What is meant by oppression’? State whether the aggrieved party would succeed in

obtaining relief from Tribunal on the ground of oppression in the following cases:

(i) The majority of the Board of directors override the minority directors and the

minority directors apply to Tribunal complaining oppression by majority di rectors.

(ii) A petition by majority shareholders complaining oppression by minority

shareholders.

Give your answer according to the provisions of the Companies Act, 2013 . (8 Marks)

5. (a) Apex Limited failed to repay the amount borrowed from the bankers, ACE Bank

Limited, which is holding a charge on all the assets of the company. The Bank took

over management of the company in accordance with the provisions of the

Securitization and Reconstruction of Financial Assets and Enforcement of Security

Interest Act, 2002 by appointing four persons as directors. The company is managed

by a Managing Director, Mr. X. Referring to the provisions of the said Act, examine

whether Mr. X is entitled to compensation for loss of office and also explain the effect

of such takeover on certain rights of the shareholders of the company. (8 Marks)

(b) State the law with respect to the Establishment of Special Court? Mr. A is judicial

magistrate in a lower court. He was appointed to hold the office of the special court

for the speedy disposal of the pending cases under the Act. Decide in the light of the

Companies Act, 2013, whether the appointment of Mr. A is tenable. (8 Marks)

6. (a) Mr. X was appointed as company liquidator by tribunal during the liquidation

proceedings in the sale of assets. Later it was discovered that Mr. X is relative of one

of the director of the company. Certain creditors of the company filed an application

before the tribunal that there is a lack of independence during the term of his

appointment due to his relation with director. State the legal position of holding of Mr.

X as a company liquidator during the liquidation proceedings . (8 Marks)

(b) What is the procedure for replacement of insolvency resolution professional? What

are the key tasks to be performed by a Resolution Professional? (8 Marks)

Page 77: Test Series: August, 2017 MOCK TEST PAPER 1 FINAL COURSE ... · The company incurred losses of Rs. 40,000 and Rs. 60,000 in 2015-2016 and 2016-2017 after charging depreciation of

4

7. Attempt any four:

(a) M/s Samrat is a company engaged in the providing of services of supplying goods all

over the world through aircrafts. The aircrafts of the said company is registered and

insured in India with the reputed insurance company. Company found that the

insurance policy of one of aircraft which is in Europe had expired. Company said to

his officer to get new Insurance policy of that aircraft in Europe. State the validity of

such an act of registration of aircraft in Europe. (4 Marks)

(b) Universal, a foreign company, incorporated in Australia was carrying on its business

in Delhi related to manufacturing of automobile parts. Due to failure of its compliance

with the respective law of the country under which it was incorporated, it was ceased

to exist. Decide in the light of the Companies Act, 2013 the status of the company

and the effect on the conduct of business in India. (4 Marks)

(c) State the circumstances when persons are not entitled to make an application to

initiate corporate insolvency resolution process.

Suppose a corporate debtor has committed a default and is undergoing a corporate

insolvency resolution process. A corporate applicant Mr. X thereof files an application

for initiating corporate insolvency resolution process with the Adjudicating Authority,

State whether he is entitled to make an application to initiate corporate insolvency

resolution process? (4 Marks)

(d) Explain the effect of usage developed by contemporary opinion through practice

under any statute with an example. (4 Marks)

(e) What is the effect of order of moratorium in the Insolvency and Bankruptcy Code,

2016? (4 Marks)

Page 78: Test Series: August, 2017 MOCK TEST PAPER 1 FINAL COURSE ... · The company incurred losses of Rs. 40,000 and Rs. 60,000 in 2015-2016 and 2016-2017 after charging depreciation of

Test Series: August, 2017

MOCK TEST PAPER -1

FINAL COURSE: GROUP – I

PAPER – 4: CORPORATE AND ALLIED LAWS

SUGGESTED ANSWERS/HINTS

1. (a) Section 123 of the Companies Act, 2013 provides for declaration of dividend.

According to this section, Dividend shall be declared or paid by a company for any

financial year only—

out of the profits of the company for that year arrived at after providing for

depreciation in accordance with the provisions of section 123(2), or

out of the profits of the company for any previous financial year or years arrived

at after providing for depreciation in accordance with the provisions of that sub -

section and remaining undistributed, or

out of both; or

out of money provided by the Central Government or a State Government for

the payment of dividend by the company in pursuance of a guarantee given by

that Government.

In the light of the above provisions we can conclude that Construction limited can pay

dividends out of the money provided by the Central Government.

(b) Reporting of frauds by auditor and other matters: As per section 143 read with

rule 13 of the Companies (Audit and Auditors) Rules, 2014, if an auditor of a company,

in the course of the performance of his duties as auditor, has reason to believe that

an offence of fraud, which involves or is expected to involve individually an amount

of rupees one crore or above, is being or has been committed against the company

by its officers or employees, the auditor shall report the matter to the Central

Government.

The auditor shall report the matter to the Central Government as under: -

(i) the auditor shall report the matter to the Board or the Audit Committee , as

the case may be, immediately but not later than two days of his knowledge of

the fraud, seeking their reply or observations within forty-five days;

(ii) on receipt of such reply or observations, the auditor shall forward his report

and the reply or observations of the Board or the Audit Committee along with his

comments (on such reply or observations of the Board or the Audit Committee)

to the Central Government within fifteen days from the date of receipt of such

reply or observations;

Page 79: Test Series: August, 2017 MOCK TEST PAPER 1 FINAL COURSE ... · The company incurred losses of Rs. 40,000 and Rs. 60,000 in 2015-2016 and 2016-2017 after charging depreciation of

2

(iii) in case the auditor fails to get any reply or observations from the Board or

the Audit Committee within the stipulated period of forty-five days, he shall

forward his report to the Central Government along with a note containing the

details of his report that was earlier forwarded to the Board or the Audit

Committee for which he has not received any reply or observations;

(iv) the report shall be sent to the Secretary, Ministry of Corporate Affairs in a

sealed cover by Registered Post with Acknowledgement Due or by Speed Post

followed by an e-mail in confirmation of the same;

(v) the report shall be on the letter-head of the auditor containing postal

address, e-mail address and contact telephone number or mobile number and

be signed by the auditor with his seal and shall indicate his Membership Number;

and

(vi) The report shall be in the form of a statement as specified in Form ADT-4.

Details of each of the fraud reported to the Audit Committee or the Board during the

year shall be disclosed in the Board’s Report:-

(a) Nature of Fraud with description;

(b) Approximate Amount involved;

(c) Parties involved, if remedial action not taken; and

(d) Remedial actions taken.

(c) Tenure of office of Presiding Officer and other Members of Securities Appellate

Tribunal (Section 15N)- According to the provision, the Presiding Officer or every

Judicial or Technical Member of the Securities Appellate Tribunal(SAT) shall hold

office for a term of five years from the date on which he enters upon his office, and

shall be eligible for reappointment for another term of maximum five years.

Provided that no Presiding Officer or the Judicial or Technical Member shall hold

office after he has attained the age of seventy years.

As per the given provision, during re-appointment, Mr. Z was of 69 years. As per the

section he shall be eligible for reappointment for another term of maximum five years

but as per the proviso no Presiding Officer or the Judicial or Technical Member shall

hold office after he has attained the age of seventy years.

Accordingly, in the given case Mr. Z can be re-appointed but only for one year.

(d) Tenure [Section 149(10) & (11)]:

Subject to the provisions of section 152 (Appointment of directors), an independent

director shall hold office for a term up to five consecutive years on the Board of a

company. He shall be eligible for re-appointment on passing of a special resolution

by the company and disclosure of such appointment in the Board's report.

Page 80: Test Series: August, 2017 MOCK TEST PAPER 1 FINAL COURSE ... · The company incurred losses of Rs. 40,000 and Rs. 60,000 in 2015-2016 and 2016-2017 after charging depreciation of

3

No independent director shall hold office for more than two consecutive terms.

However, such independent director shall be eligible for appointment after the

expiration of three years of ceasing to be an independent director:

Provided that during the said period of three years, such independent director shall

not, be appointed in or be associated with the company in any other capacity, either

directly or indirectly.

As per the given fact, Mr. Azad, an independent director, was appointed in the AGM

for a period of three years. After the expiry of 3 years he was re-appointed for a period

of 5 years. Since he has completed two tenures but hasn’t completed ten years in

total, therefore X company thought to appoint Mr. Azad as independent director in the

upcoming AGM for another 2 years to complete his total term of 10 years.

As per the above provision, since the sections 149(10) provides for a term upto 5

years for an independent director which means that an independent director can be

appointed for a term less than 5 years. Further section 149(11) states that no

independent director shall hold office for more than two consecutive terms but he

shall be eligible for appointment after expiration of 3 years of ceasing to become an

independent director. Therefore as Mr. Azad has already completed two consecutive

terms, so he cannot be re-appointed for another 2 years so as to complete his total

term of 10 years. So, reappointment of Mr. Azad for further term in the X Company is

not valid.

2. (a) (i) As per section 233 (1) Notwithstanding the provisions of section 230 and section

232, a scheme of merger or amalgamation may be entered between,

2 or more small companies

a holding company and its wholly-owned subsidiary company. If 100% of

its share capital is held by the holding company, except the shares held by

the nominee or nominees to ensure that the number of members of

subsidiary company is not reduced below the statutory limit as provided in

section 187

such other class or classes of companies as may be prescribed.

The provisions given for fast track merger in the section 233 are in the optional

nature and not a compulsion to the company. If a company wants to make

application for merger as per section 232, it can do so.

Hence, here the Company Secretary of the XYZ limited has erred in the law and

his contention is not valid as per law. The company shall have an option to

choose between normal process of merger and fast track merger.

(ii) As per section 212 of the Companies Act, 2013, the Central Government may

assign the investigation into affairs of a company to the Serious Frauds

Investigation Office on the basis of an opinion formed from the following:

Page 81: Test Series: August, 2017 MOCK TEST PAPER 1 FINAL COURSE ... · The company incurred losses of Rs. 40,000 and Rs. 60,000 in 2015-2016 and 2016-2017 after charging depreciation of

4

1. After the inspection of books of account or papers or inquiry the Registrar

shall submit a written report to the Central Government. The report may

recommend the need for further investigation along with reasons in

support. The Central Government on receipt of such report can order an

investigation under Serious Frauds Investigation Office.

2. The company may pass a special resolution and can request Central

Government to investigate into the affairs of the company.

3. The Central Government can order investigation under Serious Frauds

Investigation Office, in public interest.

4. The departments Central Government and State Governments can request

for investigation under Serious Frauds Investigation Office.

(b) (i) As per the definition of the independent director (ID) given under section 149 of

the Companies Act, 2013, an independent director means a director other than

a managing director or a whole time director or a nominee director and who

fulfills the criteria laid out u/s 149(6).

Therefore, if ID meets the above qualifications, then the same person can be

appointed as an ID in the company or its holding, subsidiary or associate

company. So Mr. Khan can be appointed in the Board of a Company, in its

subsidiary or its holding or associate company.

(ii) As per section 160, a company shall refund amount deposited along with the

candidature of a person who has been elected as a director or who has received

more than 25% of total votes. Since in the given case, Mr. D though failed to be

appointed but acquired 30% of the total votes, so he is eligible for the refund of

the deposit amount of 1lac rupees.

3. (a) (i) As per Section 165(1) of the Companies Act no person shall hold office as a

director, including any alternate directorship, in more than 20 companies at the

same time.

As the section uses the term “company” therefore while counting the limit of 20,

all companies will be included. However, it may be inferred that foreign body

corporates, whether a holding, subsidiary or otherwise, will not be included in

the said limit of holding directorship in 20 companies. Further, as per the

notification dated 5th June 2015, section 165(1) shall not apply to section 8

companies. Therefore, the directorships in section 8 companies will be excluded

while calculating the limit of 20 companies

(ii) As per section 173(5), OPC, small company and dormant company shall be

deemed to have complied with the provisions of this section if at least one

meeting of the board of ‘directors’ has been conducted in each half of a calendar

year and the gap between the two meetings is not less than 90 days.

Page 82: Test Series: August, 2017 MOCK TEST PAPER 1 FINAL COURSE ... · The company incurred losses of Rs. 40,000 and Rs. 60,000 in 2015-2016 and 2016-2017 after charging depreciation of

5

Therefore, the meetings held on 1st January, 2016 and 31st December 2016 have

complied with the above provision and will be deemed to be proper board

meetings. However, the meeting held on 1st January 2017 will not be considered

as board meeting required for the purpose of the meeting the criteria of holding

minimum 2 Board meetings with a gap of 90 days. Hence, while the Board

meeting held on 1st January 2017 will be valid, however, the next board meeting

must be held between April-June, 2016 i.e., after a gap of 90 days from 31st

December, 2016.

(b) (i) According to clause (b) of section 179(3), The Board of Directors of a company

shall exercise the power to authorise buy-back of securities under section 68,

on behalf of the company by means of resolutions passed at meetings of the

Board.

According to section 68(2), No company shall purchase its own shares or other

specified securities, unless—

(a) the buy-back is authorised by its articles;

(b) a special resolution has been passed at a general meeting of the company

authorising the buy-back:

However, nothing contained in this clause shall apply to a case where—

(1) the buy-back is, 10% or less of the total paid-up equity capital and free

reserves of the company; and

(2) such buy-back has been authorised by the Board by means of a resolution

passed at its meeting,

Thus, we can say that in the case of buy-back of shares of the Company, for the

first time, upto 10% of the paid up share capital, a special resolution will not be

required if such buy-back has been authorised by the Board by means of a

resolution passed at its meeting.

(ii) According to clause (e) of section 179(3), the Board of Directors of a company

shall exercise the power to invest the funds of the company, on behalf of the

company by means of resolutions passed at meetings of the Board.

The board may under the proviso to section 179(3) of the Companies Act, 2013

delegate the power to invest the funds of the company by a Board Resolution

passed at a duly convened Board Meeting. However, the investment in shares

of other companies will be governed by the applicable provisions of the

Companies Act, 2013 (i.e. section 186 of the Companies Act, 2013). Since the

investment of funds is governed by section of the Companies Act, 2013, thus,

specific provisions of section 186 will be applicable for such investment.

According to section 186(5), No investment shall be made or loan or guarantee

or security given by the company unless the resolution sanctioning it is passed

Page 83: Test Series: August, 2017 MOCK TEST PAPER 1 FINAL COURSE ... · The company incurred losses of Rs. 40,000 and Rs. 60,000 in 2015-2016 and 2016-2017 after charging depreciation of

6

at a meeting of the Board with the consent of all the directors present at the

meeting and the prior approval of the public financial institution concerned where

any term loan is subsisting, is obtained. Thus, a unanimous resolution of the

Board is required. Section 186 does not provide for delegation. Hence, the

proposed delegation of power to the Managing Director to invest surplus funds

of the company in the shares of some other companies, is not in order.

4. (a) (i) In terms of Section 135(5) of the Companies Act, 2013, the Board of every

company to which section 135 is applicable, shall ensure that the company

spends, in every Financial year at least 2 per cent of average net profits of the

company made during the three immediately preceding financial years, in

pursuance of its CSR policy. There is no provision for carry forward of excess

expenditure to the next year(s). The words used in the section are ‘at least’.

Therefore, any expenditure over 2% would be considered as voluntary higher

spending.

(ii) Here in the given case, as per section 129 of the Companies Act, 2013, A Ltd.

will consolidate financial statement of both B and C Ltd. A Ltd. shall also attach

the financial statement of B Ltd. along with filing of consolidated financials of A

Ltd. Also a statement containing the salient features of the financial statement

of a companys’ subsidiary, associate or joint venture shall be filed as an

annexure to the board’s report. The word ‘Subsidiary’ includes Associate

Company and joint venture.

(b) Oppression: Oppression, according to the Dictionary meaning of the word, is any act

exercised in a manner burdensome, harsh and wrongful. The meaning of the term

‘oppression’ was explained by Lord Cooper in the Scottish case of Elder v. Elder and

Watson Ltd, as given below:

“The conduct complained of should be at the lowest involve a feasible departure from

the standards of fair dealing and the violation of the conditions of fair play on which

every shareholder entrusting his money to the company is entitled to rely.

(i) Oppression of a member as a director: The oppression dealt with by section

241 of the Companies Act, 2013, is only oppression of members in their

character as such; and it is only in that character they can involve section 241.

The harsh treatment, for instance, of a member who is a director or other officer

or employee, by the Board of directors or management does not come within

section 241. It has been held in Re. Bellador Silk Ltd. that if the majority of the

Board of directors override the minority directors the latter cannot resort to

section 241 and hence the minority directors will not succeed in getting relief

from Tribunal on the ground of oppression.

(ii) Right not confined to minority: According to section 244, the right to apply for

relief under section 241/242 is given to 100 members or 1/10 th of the total

number of members or any member or members holding not less than 1/10 th of

Page 84: Test Series: August, 2017 MOCK TEST PAPER 1 FINAL COURSE ... · The company incurred losses of Rs. 40,000 and Rs. 60,000 in 2015-2016 and 2016-2017 after charging depreciation of

7

the issued share capital of the company. There is nothing in this section which

suggests even indirectly that unless the application is made by minority

shareholders, it is not maintainable. The right to apply is, therefore, not confined

to oppressed minority of the shareholders alone. It was held by Calcutta High

Court in Re. Sindhri Iron Foundry (P) Ltd. that the oppressed majority also might

apply for relief under section 241. Therefore, the petitioners are likely to succeed

in getting relief provided the other condition laid down in section 242 ( i.e. that to

wind up the company would unfairly prejudice such members, but that otherwise

the facts would justify the making of a winding-up order on just and equitable

ground) is satisfied, even though the Delhi High Court held a contrary view in

Suresh Kumar Sanghi v. Supreme Motors Ltd.

5. (a) Apex Limited failed to repay the amount borrowed from the bankers, ACE Bank

Limited, which is holding a charge on all the assets of the company. The bank took

over management of the company in accordance with the provisions of the

Securitisation and Reconstruction of Financial Assets and Enforcement of Security

Interest Act, 2002 by appointing four persons as directors. The company is managed

by a Managing Director, Mr. X.

Here, Apex Limited is a borrower and ACE Bank Limited is a secured creditor.

Compensation to Managing director (Mr. X) for loss of office:

According to section 16 of the Securitisation and Reconstruction of Financial Assets

and Enforcement of Security Interest Act, 2002, irrespective of anything contained in

any contract or in any other law for the time being in force, no managing director or

any other director or a manager or any person in charge of management of the

business of the borrower shall be entitled to any compensation for the loss of o ffice

or for the premature termination under this Act. However any such managing director

or any other director or manager or any such person in charge of management has

the right to recover from the business of the borrower, moneys recoverable otherwise

than by way of such compensation.

Effect of takeover on rights of the shareholders:

Where the management of the business of a borrower, being a company as defined

in the Companies Act is taken over by the secured creditor, then, notwithstanding

anything contained, such borrower in the said Act or in the memorandum or articles

of association of such company -

(1) it shall not be lawful for the shareholders of such company or any other person

to nominate or appoint any person to be a director of the company;

(2) no resolution passed at any meeting of the shareholders of such company shall

be given effect to unless approved by the secured creditor;

Page 85: Test Series: August, 2017 MOCK TEST PAPER 1 FINAL COURSE ... · The company incurred losses of Rs. 40,000 and Rs. 60,000 in 2015-2016 and 2016-2017 after charging depreciation of

8

(3) no proceeding for the winding up of such company or for the appointment of a

receiver in respect thereof shall lie in any court, except with the consent of the

secured creditor.

The secured creditor is under an obligation to restore the management of the

business of the borrower, on realisation of his debt in full, in case of takeover of the

management of the business of a borrower by such secured creditor.

"Provided that if any secured creditor jointly with other secured creditors or any asset

reconstruction company or financial institution or any other assignee has converted

part of its debt into shares of a borrower company and thereby acquired controlling

interest in the borrower company, such secured creditors shall not be liable to restore

the management of the business to such borrower."

(b) Establishment of special court: As per section 435 of the Companies Act, 2013,

the Central Government may, for the purpose of providing speedy trial of offences

punishable under this Act with imprisonment of two years or more, by notification,

establish or designate as many Special Courts as may be necessary.

Provided that all other offences shall be tried, as the case may be, by a Metropolitan

Magistrate or a Judicial Magistrate of the First Class having jurisdiction to try any

offence under this Act or under any previous company law.

Appointment of judge: A Special Court shall consist of a single judge who shall be

appointed by the Central Government with the concurrence of the Chief Justice of the

High Court within whose jurisdiction the judge to be appointed is working. A person

shall not be qualified for appointment as a judge of a Special Court unless he is,

immediately before such appointment, holding office of a Sess ions Judge or an

Additional Sessions Judge.

Since in the given case, Mr. A who is a judicial magistrate in a lower court, was

appointed to hold the office of the special court for the speedy disposal of the pending

cases under the Act. As per the above provision, person shall be qualified for

appointment as a judge of a Special Court if he, immediately before such

appointment, holding office of a Sessions Judge or an Additional Sessions Judge.

Here Mr A. was not complying with the eligibility criteria, so his appointment as a

judge of special court is not tenable.

6. (a) As per section 276 of Insolvency & Bankruptcy Code, 2016, the Tribunal may, on a

reasonable cause being shown and for reasons to be recorded in writing, remove the

provisional liquidator or the Company Liquidator, as the case may be, as liquidator of

the company on any of the following grounds, namely:—

(a) misconduct;

(b) fraud or misfeasance;

Page 86: Test Series: August, 2017 MOCK TEST PAPER 1 FINAL COURSE ... · The company incurred losses of Rs. 40,000 and Rs. 60,000 in 2015-2016 and 2016-2017 after charging depreciation of

9

(c) professional incompetence or failure to exercise due care and diligence in

performance of the powers and functions;

(d) inability to act as provisional liquidator or as the case may be, Company

Liquidator;

(e) conflict of interest or lack of independence during the term of his appointment

that would justify removal.

On removal, the Tribunal may transfer the work assigned to him or it to another

Company Liquidator for reasons to be recorded in writing. In case of loss or damage,

the Tribunal may recover or cause to be recovered such loss or damage from the

liquidator and pass such other orders as it may think fit. The Tribunal shall, before

passing any order under this section, provide a reasonable opportunity of being heard

to the Company Liquidator.

In the given case, there seems to be lack of independence during the term of his

appointment as of existence of a relationship of Mr. X the company liquidator and the

company due to being the relative of the director of the company. However as per

section 275 of the IB Code, on appointment as Company Liquidator, as the case may

be, such liquidator shall file a declaration within seven days from the date of

appointment disclosing conflict of interest or lack of independence in respect of his

appointment, if any, with the Tribunal and such obligation shall continue throughou t

the term of his appointment.

(b) As per Section 27 of the Insolvency & Bankruptcy Code, the Committee of Creditors

may at a meeting, by a vote of 75% of voting shares propose to replace the insolvency

resolution professional with another resolution professional.

The Committee of Creditors shall forward the name of the insolvency professional

proposed by them to the Adjudicating Authority and after the confirmation of the

proposed insolvency resolution professional by the Board he shall be appointed in

the same manner as laid down in Section 16.

Where any disciplinary proceedings are pending against the proposed resolution

professional then the resolution professional appointed under section 22 shall

continue till the appointment of another resolution professional under this section.

key tasks to be performed by a Resolution Professional

The following are the key tasks to be performed by a resolution professional: -

(a) Obtaining Valuation of the entity

(b) Preparation of Information Memorandum

(c) Preparation of Resolution plan

(d) Obtaining consent of the Committee of Creditors for the Resolution plan

(e) Periodic reporting to the Board

Page 87: Test Series: August, 2017 MOCK TEST PAPER 1 FINAL COURSE ... · The company incurred losses of Rs. 40,000 and Rs. 60,000 in 2015-2016 and 2016-2017 after charging depreciation of

10

7. (a) Given problem is based on the section 2CB of the Insurance Act, 1938. Said section

deals with the Indian properties not to be insured with foreign insurers. According to

the section, no person shall take out or renew any policy of insurance in respect of

any property in India or any ship or other vessel or aircraft registered in India with an

insurer whose principal place of business is outside India, without the permission of

the IRDAI.

In the given case, act of registration of aircraft of M/s Samrat which is an Indian

property, with an insurer in Europe, is an invalid act.

(b) Section 376 of the Companies Act, 2013 provides the law related to the power of wind

up Foreign Companies, although dissolved. Provision states that where a body

corporate incorporated outside India which has been carrying on business in India,

ceases to carry on business in India, it may be wound up as an unregistered company

under this Part (i.e., Part I of the Chapter 21 which deals with the companies

authorized to register under this Act), notwithstanding that the body corporate has

been dissolved or otherwise ceased to exist as such under or by virtue of the laws of

the country under which it was incorporated.

As per the facts given in the question, Universal, a foreign company, incorporated in

Australia ceased to exist as per the law of the country, also ceased to carry on

business in Delhi. Accordingly, Universal Company may be wound up as an

unregistered company although it ceased to exist in Australia.

(c) Persons not entitled to make application.

The following persons shall not be entitled to make an application to initiate corporate

insolvency resolution process -

(a) a corporate debtor undergoing a corporate insolvency resolution process; or

(b) a corporate debtor having completed corporate insolvency resolution process

twelve months preceding the date of making of the application; or

(c) a corporate debtor or a financial creditor who has violated any of the terms of

resolution plan which was approved twelve months before the date of making of

an application under this Chapter; or

(d) a corporate debtor in respect of whom a liquidation order has been made.

In this section, a corporate debtor includes a corporate applicant in respect of such

corporate debtor. [Section 11]

As per the facts corporate applicant Mr. X seems to be a independent individual and

not a corporate applicant in relation to such corporate debtor who is undergoing a

corporate insolvency resolution process. So, he shall be entitled to make an

application to initiate corporate insolvency resolution process.

Page 88: Test Series: August, 2017 MOCK TEST PAPER 1 FINAL COURSE ... · The company incurred losses of Rs. 40,000 and Rs. 60,000 in 2015-2016 and 2016-2017 after charging depreciation of

11

(d) Effect of usage: Usage or practice developed under the statute is indicative of the

meaning recognized to its words by contemporary opinion. A uniform notorious

practice continued under an old statute and inaction of the Legislature to amend the

same are important factors to show that the practice so followed was based on correct

understanding of the law. When the usage or practice receives judicial or legislative

approval it gains additional weight.

In this connection, we have to bear in mind two Latin maxims:

(i) ‘Optima Legum interpresest consuetudo’ (the custom is the best interpreter of

the law); and

(ii) ‘Contempranea expositoest optima et fortissima in lege’ (the best way to

interpret a document is to read it as it would have been read when made).

Therefore, the best interpretation/construction of a statute or any other

document is that which has been made by the contemporary authority. Simply

stated, old statutes and documents should be interpreted as they would have

been at the time when they were enacted/written.

Contemporary official statements throwing light on the construction of a statute

and statutory instruments made under it have been used as contemporanea

exposition to interpret not only ancient but even recent statutes in India.

Example: Documents issued by the Government simultaneously with the

notification under section 16(1) of the Securities Contracts (Regulation) Act,

1956 were used as contemporanea expositio of the notification . [Desh Bandhu

Gupta & Co. v. Delhi Stock Exchange Association Ltd., AIR 1979 SC]

(e) Moratorium has been explained in Section 14 of the Code, during the moratorium

period the following acts shall be prohibited:

The institution of suits or continuation of any pending suits or proceedings

against the corporate debtor including execution of any judgment, decree or

order in any court of law, tribunal, arbitration panel or other authority;

Transferring, encumbering, alienating or disposing of by the corporate debtor

any of its assets or any legal right or beneficial interest therein;

Any action to foreclose, recover or enforce any security interest created by the

corporate debtor in respect of its property including any action under the

SARFAESI Act, 2002

The recovery of any property by an owner or lessor where such property is

occupied by or in the possession of the corporate debtor.

Page 89: Test Series: August, 2017 MOCK TEST PAPER 1 FINAL COURSE ... · The company incurred losses of Rs. 40,000 and Rs. 60,000 in 2015-2016 and 2016-2017 after charging depreciation of

Test Series: August, 2017

MOCK TEST PAPER – 1

FINAL COURSE: GROUP – II

PAPER – 5: ADVANCED MANAGEMENT ACCOUNTING

Question No. 1 is compulsory

Answer any five questions from the remaining six questions

Time Allowed – 3 Hours Maximum Marks – 100

1. (a) Mini SIAM, located in Himachal Pradesh, manufactures high quality industrial

products. NIA Industries have asked Mini SIAM for a special job that must be

completed within one week.

Raw material R1 (highly toxic) will be needed to complete the NIA Industries’ special

job. Mini SIAM purchased the R1 two weeks ago for Rs. 7,500 for a job ‘A’ that

recently was completed. The R1 currently in stock is the excess from that job and

Mini SIAM had been planning to dispose of it. Mini SIAM estimates that it would cost

them Rs. 1,250 to dispose of the R1. Current replacement cost of R1 is Rs. 6,000.

Special job will require 250 hours of labour G1 and 100 hours of labour G2. Mini

SIAM pays their G1 and G2 employees Rs.630 and Rs.336 respectively for 42 hours

of work per week. Mini SIAM anticipates having excess capacity of 150 [G1] and 200

[G2] labour hours in the coming week. Mini SIAM can also hire additional G1 and G2

labour on an hourly basis; these part-time employees are paid an hourly wage

based on the wages paid to current employees.

Suppose that material and labour comprise Mini SIAMs only costs for completing

the special job. What is the minimum price that Mini SIAM should bid on this job?

(5 Marks)

(b) Following information are extracted from monthly budgets of V ictoria Ltd. You are

required to find out missing figures.

November December

Beginning WIP Inventory 36,000 ???

Beginning Finished Goods Inventory 44,000 ???

Variable Cost of Goods Sold 1,23,000 ???

Direct Material Usage 50,000 56,000

Direct Labour 53,100 69,000

Variable Overhead 25,000 29,000

Page 90: Test Series: August, 2017 MOCK TEST PAPER 1 FINAL COURSE ... · The company incurred losses of Rs. 40,000 and Rs. 60,000 in 2015-2016 and 2016-2017 after charging depreciation of

2

Variable Cost of Goods Manufactured 1,09,000 1,14,800

Ending WIP Inventory ??? ???

Ending Finished Goods Inventory ??? 45,000

(5 Marks)

(c) XY Ltd. manufactures two types of mobile phones, X and Y. Due to severe

competition, it has to reduce the prices for the next production period. The following

information is provided:

X Y

Current Period:

Selling price (Rs. / unit) 10,000 12,000

P/V ratio 25% 30%

Product-specific fixed cost (Rs.) 10,00,000 15,00,000

Next Period:

Selling price (Rs. /unit) 8,000 9,000

For the next period the company wants the present P/V ratio to be maintained and

achieve a break-even for both the products at 400 units.

What is the cost reduction programme envisaged? (5 Marks)

(d) The following 3 × 3 matrix arises after the Row minimum and Column minimum

operation of a minimization assignment problem:

Columns Rows

C1 C2 C3

R1 0 0 0

R2 0 a b

R3 0 c d

Given that a b c d 0, (i.e. a, b, c, d are non zero and unequal), will R1 C1

(cell at the intersection of Row 1 and Column 1) appear in the final solution under

the assignment algorithm? Why? Interpret this part of the algorithm. (5 Marks)

2. (a) A Company has two production departments KTS and KTW. Standards for the

forthcoming year is as follows:

Particulars Department

KTS

Department

KTW

Direct Labour Hours available per period 12,000 8,000

Page 91: Test Series: August, 2017 MOCK TEST PAPER 1 FINAL COURSE ... · The company incurred losses of Rs. 40,000 and Rs. 60,000 in 2015-2016 and 2016-2017 after charging depreciation of

3

Standard Wage Rate per hour Rs. 3 Rs. 2.5

Expected Learning Curve 80% 70%

Standard Variable Overheads per hour Rs. 10 Rs. 6

Standard Fixed Overheads per hour Rs. 11 Rs. 7

Direct Labour Hours required for first 100 units 36 per unit 18 per unit

The direct materials are introduced in Department KTS. The company has following

quote for purchase of direct material:

Level of Outputs (Units) Price of Direct Material per unit of output

100 Rs. 36.00

200 Rs. 32.40

Overtime, if required is paid at the time and half. The overhead rates as given

above do not include overtime premium.

It is the policy of the company to add profit margin as under in quoting the prices:

Department Percentage

KTS 10% of Labour & 15% Overhead Cost

KTW

The company has received a special order. Special tooling costs of the order

amount to Rs. 5,500. If this order is for 200 units or less, it will be executed in the

period which has a workload of 7,680 direct labour hours in Department KTS and

4,200 direct labour hours in Department KTW.

Required:

If the company decides to get the work executed entirely within the company, what

price, on cost-plus basis, should be quoted for the order, if it consists of 100 Units,

200 units. (12 Marks)

(b) State whether the learning curve theory can be applied to the following independent

situations briefly justifying your decision:

(i) A labour intensive sculpted product is carved from the metal provided to the

staff. The metal is sourced from different suppliers since it is scarce. The alloy

composition of the input metal is quite different among the suppliers.

(ii) Pieces of hand-made furniture are assembled by the company in a far off

location. The labourers do not know anything about the final product which

utilizes their work. As a matter of further precaution, rotation of labour is done

frequently.

(iii) Skilled workers have been employed for a long time. The company has

Page 92: Test Series: August, 2017 MOCK TEST PAPER 1 FINAL COURSE ... · The company incurred losses of Rs. 40,000 and Rs. 60,000 in 2015-2016 and 2016-2017 after charging depreciation of

4

adequate market for the craft pieces done by these experts.

(iv) A company funds that it always has an adverse usage of indirect material. It

wants to apply learning curve theory to improve the way standards have been

set. (4 Marks)

3. (a) Given below is an iteration in a simplex table for a maximization objective linear

programming product mix problem for products X1, X2 and X3.

Cj 6 4 10 0 0 0

Basic

Variable Quantity X1 X2 X3 S1 S2 S3

0 S1 400 0 4/3 0 1 -1/3 0

6 X1 400 1 2/3 2 0 1/3 0

0 S3 400 0 5/3 0 0 -2/3 1

Zj 2,400 6 4 12 0 2 0

Cj - Zj 0 0 -2 0 -2 0

Answer the following questions:

(i) Is the above solution feasible?

(ii) Perform one more iteration with X2 entering the solution to get a solution with

the same value for the objective function.

(iii) Indicate the shadow prices.

(iv) If customer is prepared to pay higher price for product X3 then by how much

should the price be increased so that the company's profit remains

unchanged?

(v) From the given table, derive any one original constraint inequality with the

coefficients of variables in their simplest whole number forms. (8 Marks)

(b) A company following standard marginal costing system has the following interim

trading statement for the quarter ending 30th June, 2013, which reveals a loss of

Rs. 17,000, detailed below:

Rs.

Sales 4,99,200

Closing Stock (at prime cost) 18,000

Direct Material 1,68,000

Direct Labour 1,05,000

Variable Overhead 42,000

Page 93: Test Series: August, 2017 MOCK TEST PAPER 1 FINAL COURSE ... · The company incurred losses of Rs. 40,000 and Rs. 60,000 in 2015-2016 and 2016-2017 after charging depreciation of

5

Fixed Overhead 1,20,000

Fixed Administration Overhead 40,000

Variable Distribution Overhead 19,200

Fixed Selling Overhead 40,000

Loss 17,000

Additional information is as follows:

(i) Sales for the quarter were 1,200 units. Production was 1,400 units, of which

100 units were scrapped after complete manufacture. The factory capacity is

estimated at 2,000 units.

(ii) Because of low production, labour efficiency during the quarter is estimated to

be 20% below normal level.

You are required to analyse the above and report to the management giving the

reasons for the loss. (8 Marks)

4. (a) Venus Medical Care Co. (VMCC) is operating its entire business through its four

customers V1, V2, V3, and V4. V1 and V2 are small pharmaceutical stores while V3

and V4 are large discount stores with attached pharmacies. VMCC prices its

products at 25% above variable cost, although all four customers demand and

receive a sizable discount off the list price.

The Finance Officer Mr. Albert has been asked to undertake a customer profitability

analysis that shows the profit from each customer and each customer channel,

stand-alone pharmaceuticals, and large pharmaceuticals attached to discount

stores.

Mr. Albert identifies Rs. 20,250 of general administration costs to small

pharmaceuticals stores and Rs.48,375 of general administration costs to the large

discount stores.

You are required to assist Mr. Albert in preparing a customer profitability report as

desired. Also suggest some points to improve VMCC’s profit.

Item Small Pharmaceuticals Large

Pharmaceuticals

Activity Rate

V1 V2 V3 V4

Number of Orders

4 9 6 3 Rs.750

Order Size Rs.40,000 Rs.20,000 Rs.4,25,000 Rs.4,00,000 n/a

Average Discount

5% 10% 18% 12% n/a

Page 94: Test Series: August, 2017 MOCK TEST PAPER 1 FINAL COURSE ... · The company incurred losses of Rs. 40,000 and Rs. 60,000 in 2015-2016 and 2016-2017 after charging depreciation of

6

Regular Deliveries

4 9 6 3 Rs.375

Expedited Deliveries

2 0 2 0 Rs. 1,250

(8 Marks)

(b) The following table gives the activities in a construction project and the time

duration of each activity:

Activity Predecessors Normal Time (Days)

A --- 16

B --- 20

C A 8

D A 10

E B,C 6

F D,E 12

Required

(i) Draw the activity network of the project

(ii) Find critical path

(iii) Find the total float and free-float for each activity (8 Marks)

5. (a) Natural Spices manufactures and distributes high-quality spices to gourmet food

shops and top quality restaurants. Gourmet and high-end restaurants pride

themselves on using the freshest, highest-quality ingredients.

Natural Spices has set up five state of the art plants for meeting the ever growing

demand. The firm procures raw material directly from the centers of produce to

maintain uniform taste and quality. The raw material is first cleaned, dried and

tested with the help of special machines. It is then carefully grounded into the

finished product passing through various stages and packaged at the firm’s

ultraclean factory before being dispatched to customers.

The following variances pertain to last week of operations, arose as a consequence

of management’s decision to lower prices to increase volume.

Sales Volume Variance 18,000 (F)

Sales Price Variance 14,000 (A)

Purchase Price Variance 10,000 (F)

Labour Efficiency Variance 11,200 (F)

Fixed Cost Expenditure Variance 4,400 (F)

Page 95: Test Series: August, 2017 MOCK TEST PAPER 1 FINAL COURSE ... · The company incurred losses of Rs. 40,000 and Rs. 60,000 in 2015-2016 and 2016-2017 after charging depreciation of

7

Required

(i) Identify the ‘Critical Success Factors’ for Natural Spices.

(ii) Evaluate the management’s decision with the ‘Overall Corporate Strategy’ and

‘Critical Success Factors’. (8 Marks)

(b) Catalyst Ltd. Makes a single product with the following details:

Description Current Situation Proposed Change

Selling Price (Rs./unit) 10

Direct Costs (Rs./unit) 5

Present number of setups per production period, (before each production run, setup is done)

42

Cost per set up (Rs.) 450 Decrease by Rs.90

Production units per run 960 1,008

Engineering hours for production period 500 422

Cost per engineering hour (Rs.) 10

The company has begun Activity Based Costing of fixed costs and has presently

identified two cost drivers, viz. production runs and engineering hours. Of the total

fixed costs presently at Rs. 96,000, after the above, Rs. 72,100 remains to be

analyzed. There are changes as proposed above for the next production period for

the same volume of output.

Required

(i) How many units and in how many production runs should Catalyst Ltd.

produce in the changed scenario in order to break-even?

(ii) Should Catalyst Ltd. continue to break up the remaining fixed costs into activity

based costs? Why? (8 Marks)

6. (a) Duke Ltd. has two Divisions ‘Dx’ and ‘Dz’ with full profit responsibility. The Division

‘Dx’ produces Component ‘X’ which it sells to ‘outside’ customers only. The Division

‘Dz’ produces a product called the ‘Z’ which incorporates Component ‘X’ in its

design. ‘Dz’ Division is currently purchasing required units of Component ‘X’ per

year from an outside supplier at market price.

New CEO for Indian Operations has explored that ‘Dx’ Division has enough capacity

to meet entire requirements of Division ‘Dz’ and accordingly he requires internal

transfer between the divisions at marginal cost from the overall company’s

perspective.

Page 96: Test Series: August, 2017 MOCK TEST PAPER 1 FINAL COURSE ... · The company incurred losses of Rs. 40,000 and Rs. 60,000 in 2015-2016 and 2016-2017 after charging depreciation of

8

Manager of Division ‘Dx’ claims that transfer at marginal cost are unsuitable for

performance evaluation since they don’t provide an incentive to the division to

transfer goods internally. He stressed that transfer price should be ‘Cost plus a

Mark-Up’.

New CEO worries that transfer price suggested by the manager of Division ‘Dx’ will

not induce managers of both Divisions to make optimum decisions. You are

requested to help him out of the problem. (6 Marks)

(b) “Cost is not the only criterion for deciding in favour of shut down” – Briefly explain.

(4 Marks)

(c) Arnav Automobiles Ltd. is a leading manufacturer of commercial vehicles in India. It

uses Standard Costing and Variance Analysis to measure its operating

performance. Now management of Arnav Automobiles Ltd. wants to compete

globally and changed its performance measurement system to Kaizen Costing.

State why Kaizen Costing could be more useful for performance measurement than

the Standard Costing and Variance Analysis. (6 Marks)

7. Answer any four of the following questions:

(a) What is target costing? It is said that target costing fosters team work within the

organisation. Explain how target costing creates an environment in which team work

fosters. (4 Marks)

(b) The following information is given about the type of defects during a production

period and the frequencies of their occurrence in a spectacle manufacturing

company:

Defect No. of items

End Frame not equidistant from the centre 10

Non-uniform grinding of lenses 60

Power mismatches 20

Scratches on the surface 110

Spots / Stains on lenses 5

Rough edges of lenses 70

Frame colours-shade differences 25

Construct a frequency table so that a Pareto Chart can be constructed for the

defect type. Which areas should the company focus on? (4 Marks)

(c) "Sunk cost is irrelevant in decision making, but all irrelevant costs are not sunk

costs." Explain with examples. (4 Marks)

(d) Write a short note on the characteristics of the dual problem. (4 Marks)

(e) Brief the principles associate with synchronous manufacturing. (4 Marks)

Page 97: Test Series: August, 2017 MOCK TEST PAPER 1 FINAL COURSE ... · The company incurred losses of Rs. 40,000 and Rs. 60,000 in 2015-2016 and 2016-2017 after charging depreciation of

Test Series: August, 2017

MOCK TEST PAPER – 1

FINAL COURSE: GROUP – II

PAPER – 5: ADVANCED MANAGEMENT ACCOUNTING

SUGGESTED ANSWERS/HINTS

1. (a) Opportunity Cost of Labour - The G2 labour has zero opportunity cost as there is no

other use for the time already paid for and is available. However, Mini SIAM needs

to pay an additional amount for G1 labour. This amount can be save if the special

job were not there.

G1 labour:

Hours Required 250

Hours Available 150

Extra Hours Needed 100

Cost per hour (Rs.630/42hrs) Rs. 15

Opportunity Cost Rs.1,500

Thus, the ‘Opportunity Cost of Labour’ for completing the special job is Rs. 1,500.

Opportunity Cost of Material - Mini SIAM has no alternative use for the R1, they

must dispose of it at a cost of Rs. 1,250. Thus, Mini SIAM actually saves Rs. 1,250

by using the materials for the NIA Industries’ special job. Consequently, the

‘Opportunity Cost of Material’ is - Rs. 1,250 (i.e., the opportunity cost of this

resource is negative).

The minimum price is the price at which Mini SIAM just recovers its ‘Opportunity

Cost. Mini SIAM’s ‘Total Opportunity Cost’ is Rs.250 (Rs. 1,500 − Rs. 1,250).

Accordingly, minimum Price for the Special Job is Rs.250.

(b) Analysis of WIP Account

November December

Opening WIP 36,000 55,100

Add: Direct Materials Usage 50,000 56,000

Add: Direct Labor 53,100 69,000

Add: Variable Overhead 25,000 29,000

Total Inflow into WIP 1,64,100 2,09,100

Less: Variable Cost of Goods Manufactured 1,09,000 1,14,800

Ending WIP 55,100 94,300

Page 98: Test Series: August, 2017 MOCK TEST PAPER 1 FINAL COURSE ... · The company incurred losses of Rs. 40,000 and Rs. 60,000 in 2015-2016 and 2016-2017 after charging depreciation of

2

Analysis of Finished Goods Inventory Account

November December

Opening Finished Goods 44,000 30,000

Add: Cost of Goods Manufactured 1,09,000 1,14,800

Cost of Goods Available for Sale 1,53,000 1,44,800

Less: Cost of Goods Sold 1,23,000 99,800

Ending Finished Goods Inventory 30,000 45,000

(c) Statement Showing “Cost Reduction Programme”

X (Rs.) Y (Rs.)

(i) Current Period Selling Price per unit 10,000 12,000

(ii) Variable Cost for X (75%) & Y (70%) of (i) 7,500 8,400

(iii) Next Period Selling Price per unit 8,000 9,000

(iv) Variable Cost for X (75%) & Y (70%) of (iii) 6,000 6,300

(v) Cost Reduction in Variable Cost per unit (ii)- (iv) 1,500 2,100

(vi) Next Period Contribution per unit (iii)- (iv) 2,000 2,700

(vii) Desired BEP in units 400 units 400 units

(viii) Fixed Cost for next period (vi × vii) 8,00,000 10,80,000

(ix) Current Period’s Fixed Cost 10,00,000 15,00,000

(x) Cost Reduction in Fixed Cost 2,00,000 4,20,000

(d) R1C1 appears at the intersection of R1 and C1. Hence, it will have its zero replaced

by minimum of a, b, c, or d in the next operation since the number of lines to cover

zeros is less than 3.

In the next step, a or b or c or d will have one zero. Then, number of lines will be 3,

the order of the matrix. Assignments will be made to the Zeros. Hence, R 1C1 cannot

figure in this.

Interpretation

An assignment of R1C1 will eliminate the use of other costs available on R1 and C1

entirely. The left over will be a, b, c, or d combinations which are more than zero.

Hence, R1C1 taking on assignment will be non-optimal.

2. (a) Statement Showing Recommended Selling Price

100 Units (per Unit) 200 units (per Unit)

Department KTS:

Direct Labour 36 hrs. × Rs.3 108.00 36 Hrs. × 0.80 × Rs.3 86.40

Page 99: Test Series: August, 2017 MOCK TEST PAPER 1 FINAL COURSE ... · The company incurred losses of Rs. 40,000 and Rs. 60,000 in 2015-2016 and 2016-2017 after charging depreciation of

3

Overtime Premium# 0.00 10.80

Total Labour Cost (A) 108.00 97.20

Variable Overheads 36hrs. × Rs. 10 360.00 36 Hrs. × 0.80 × Rs. 10 288.00

Fixed Overheads 36hrs. × Rs. 11 396.00 36 Hrs. × 0.80 × Rs.11 316.80

Total Overheads (B) 756.00 604.80

Department KTW:

Direct Labour 18hrs. × Rs. 2.5 45.00 18 Hrs. × 0.70 × Rs.2.5 31.50

Overtime Premium 0.00 0.00

Total Labour Cost (C) 45.00 31.50

Variable Overheads 18hrs. × Rs. 6 108.00 18 Hrs. × 0.70 × Rs.6 75.60

Fixed Overheads 18hrs. × Rs. 7 126.00 18 Hrs. × 0.70 × Rs.7 88.20

Total Overheads (D) 234.00 163.80

Special Tool (E) Rs. 5,500 / 100 55.00 Rs. 5,500 / 200 27.50

Direct Material (F) 36.00 32.40

Profit on Labour (10%) Rs. (108 + 45) × 10%

15.30 Rs.(97.20 + 31.50) × 10%

12.87

Profit on Overheads (15%) Rs. (756+234) × 15%

148.50 Rs.(604.80 +163.80) × 15%

115.29

Total Profit (G) 163.80 128.16

Recommended Selling Price [(A)+ (B)+(C) + +(D)+(E)+(F)+(G)]

1,397.80 1,085.36

(#) Statement Showing Overtime Premium

Department

KTS

Department

KTW

Direct Labour Hours Available 12,000 8,000

Present workload 7,680 4,200

Balance Direct Labour Hours 4,320 3,800

Hours Required to produce 100 units: 3,600 1,800

Hours Required to produce 200 units: 5,760

(200 x 36 x 0.80)

2,520

(200 x 18 x 0.70)

Overtime Required to produce 100 units --- ---

Overtime Required to produce 200 units 1,440 Hrs ---

Overtime Premium 10.8$ per Unit ---

($) 1,440 Hrs × 3 × 50% / 200 Units

Page 100: Test Series: August, 2017 MOCK TEST PAPER 1 FINAL COURSE ... · The company incurred losses of Rs. 40,000 and Rs. 60,000 in 2015-2016 and 2016-2017 after charging depreciation of

4

(b) (i) ‘Learning Curve Theory’ will not be applicable as alloy combination of the input

metal is quite different among the suppliers hence learning experience with

one type of metal may not be beneficial for the workers to deal with other metal

with separate alloy composition.

(ii) ‘Learning Curve Theory’ will not be applicable as in this situation rotation of

labour is done frequently, labours will not be able to get the benefit of learning

and apply their learning. Hence, learning curve theory can not be applied.

(iii) ‘Learning Curve Theory’ will not be applicable as in this situation as workers

are skilled and employed for a long time , they have already achieved maximum

level of expertise by taking advantage of learning. Hence, at this point of time

learning curve theory can not be applied.

(iv) ‘Learning Curve Theory’ will not be applicable as indirect materials are the

materials which are not used directly in the production (not directly

proportionate with volume of output) and usually used machines (e.g.

lubricants, spares parts etc.) with less human interactions. Adverse usage of

indirect materials can be controlled through proper monitoring and appropriate

standard settings and not from applying learning curve theory.

3. (a)

Cj 6 4 10 0 0 0 Min.

Ratio CB Basic Variable Quantity X1 X2 X3 S1 S2 S3

0 S1 400 0 4/3 0 1 -1/3 0 300

6 X1 400 1 2/3 2 0 1/3 0 600

0 S3 400 0 5/3 0 0 -2/3 1 240

Zj = Bi jC X 6 4 12 0 2 0

Cj − Zj 0 0 -2 0 -2 0

(i) Yes, because the given solution has no artificial variables in the basic column.

(ii) Perform one more iteration with X2:

Cj 6 4 10 0 0 0

CB Basic Variable

Quantity X1 X2 X3 S1 S2 S3

0 S1 80 0 0 0 1 1/5 -4/5

6 X1 240 1 0 2 0 3/5 -2/5

4 X2 240 0 1 0 0 -2/5 3/5

Zj = Bi jC X 6 4 12 0 2 0

Cj − Zj 0 0 -2 0 -2 0

Page 101: Test Series: August, 2017 MOCK TEST PAPER 1 FINAL COURSE ... · The company incurred losses of Rs. 40,000 and Rs. 60,000 in 2015-2016 and 2016-2017 after charging depreciation of

5

(iii) Shadow Price is Rs.0, Rs.2 and Rs.0 (or any other given monetary unit) for

Constraint 1, Constraint 2 and Constraint 3 respectively and same has been

obtained from row Cj − Zj.

(iv) Cj − Zj for X3 being –2, production of each unit of X3 would cause a reduction of

Rs.2 (or any other given monetary unit). Thus, the price for X3 should be

increased by at least two rupee per unit to ensure no reduction of profits.

(v) Original Constraint Inequality with the coefficient of variables:

Let us consider the given iteration is the 2nd one. The first iteration (I1) must

have had S2 instead of X1. Row X1 of I2 has been computed by dividing the S2

row of I1 by 3. S2 of I1 (in Identity Matrix) would have been 1. Now it is 1/3.

Working backwards, we multiply row X1 of I2 by 3 to get Row S2 of I1.

Original Row S2 [X1 of I2 × 3]:

(1X1 + 2/3X2 + 2X3) × 3 ≤ 400 x 3

Or

3X1 + 2X2 + 6X3 ≤ 1,200

Similarly Original Row S1 [S1 of I2 + X1 of I2]:

(0X1 + 4/3X2 + 0X3) + (1X1 + 2/3X2 + 2X3) ≤ 400 + 400

Or

X1 + 2X2 + 2X3 ≤ 800

Similarly Original Row S3 [S3 of I2 + 2 × X1 of I2]:

0X1 + 5/3X2 + 0X3 + (1X1 + 2/3X2 + 2X3) × 2 ≤ 400 + 400 × 2

Or

2X1 + 3X2 + 4X3 ≤ 1,200

Original Constraint Inequality (with the coefficient of variables) can also be

traced through algebraic method by solving through system of equations.

(b) Working Note

Details Working Amount (Rs.)

Selling Price Rs. 4,99,200

1,200 units

416

Raw Materials

Rs.1,68,000

1,400 units

120

Page 102: Test Series: August, 2017 MOCK TEST PAPER 1 FINAL COURSE ... · The company incurred losses of Rs. 40,000 and Rs. 60,000 in 2015-2016 and 2016-2017 after charging depreciation of

6

Labour

Rs.1,05,000

1,750 units *

60

*Equivalent units (1,400 units / 80%)

Variable Overheads

Rs.42,000

1,400 units

30

Manufacturing Cost (Variable)

[Rs.120 + Rs.60 + Rs.30]

210

Distribution Overheads

Rs.19,200

1,200 units

16

Total Variable Cost [Rs.210 + Rs.16] 226

Contribution [Rs.416 - Rs.226] 190

Fixed Cost

Factory Rs. 1,20,000

Administration Rs. 40,000

Selling Rs. 40,000 2,00,000

Standard Profit for 1,200 Units Sold

Details Working Amount (Rs.)

Contribution 1,200 units Rs. 190 2,28,000

Less: Fixed Costs 2,00,000

Profit 28,000

Reconciliation between Budgeted and Actual Profit

Details Working Amount (Rs.)

Budgeted Profit (2,000 units Rs. 190 – Rs. 2,00,000)

1,80,000

Less: Volume variance (800 units Rs.190) 1,52,000

Standard Profit 28,000

Factors causing loss:

Units Scrapped (100 units Rs.210) 21,000

Labour Inefficiency (350 units Rs.60) 21,000

Undervaluation of Closing Stock {100 units (Rs.210 – Rs.180)} 3,000

Actual Profit (-)17,000

Page 103: Test Series: August, 2017 MOCK TEST PAPER 1 FINAL COURSE ... · The company incurred losses of Rs. 40,000 and Rs. 60,000 in 2015-2016 and 2016-2017 after charging depreciation of

7

4. (a) Statement Showing ‘Customer Profitability Analysis’

Particulars V1 V2 Channel

Total

V3 V4 Channel

Total Small Stores Large Stores

Revenue at

List Price

1,60,000 1,80,000 3,40,000 25,50,000 12,00,000 37,50,000

Discount 8,000 18,000 26,000 4,59,000 1,44,000 6,03,000

Net

Revenue

152,000 1,62,000 3,14,000 20,91,000 10,56,000 31,47,000

Variable

Costs

1,28,000 1,44,000 2,72,000 20,40,000 9,60,000 30,00,000

Contribution

Margin

24,000 18,000 42,000 51,000 96,000 1,47,000

Order

Processing

3,000 6,750 9,750 4,500 2,250 6,750

Regular

Deliveries

1,500 3,375 4,875 2,250 1,125 3,375

Expedited

Deliveries

2,500 --- 2,500 2,500 --- 2,500

Customer

Profit

17,000 7,875 24,875 41,750 92,625 1,34,375

Channel Cost 20,250 48,375

Channel Profit 4,625 86,000

Suggestions

VMCC is only just at breakeven point with small pharmaceuticals. To improve profit

VMCC should:

(i) Coordinate with V2 to increase order size and try to negotiate a smaller discount.

(ii) Try to work with V1 to reduce number of expedited deliveries.

VMCC makes substantial profit from the large pharmaceuticals. VMCC may give

little extra attention on V4 as V4 is most favorable customer and its order is for large

quantities. For V3, VMCC may have no options as V3 accounts more than 50% of

Sales.

Page 104: Test Series: August, 2017 MOCK TEST PAPER 1 FINAL COURSE ... · The company incurred losses of Rs. 40,000 and Rs. 60,000 in 2015-2016 and 2016-2017 after charging depreciation of

8

(b) (i) The Network for the given problem:

(ii) Critical Path: 1–2–3–4–5 (A–C–E–F).

(iii) Total Float and Free Float for each activity:

Act

ivity

Du

ratio

n

EST EFT LST LFT Slack of

Tail

Event

Slack of

Head

Event

Total

Float

Free Float

Dij Ei Ei

+

Dij

Lj

Dij

Lj Li

Ei

Lj

Ej

LST

EST

Total Float

Slack of Head Event

A

(1–2)

16 0 16 0 16 0 0 0 0

B

(1–3)

20 0 20 4 24 0 0 4 4

C

(2–3)

8 16 24 16 24 0 0 0 0

D

(2–4)

10 16 26 20 30 0 0 4 4

E

(3–4)

6 24 30 24 30 0 0 0 0

F

(4–5)

12 30 42 30 42 0 0 0 0

Page 105: Test Series: August, 2017 MOCK TEST PAPER 1 FINAL COURSE ... · The company incurred losses of Rs. 40,000 and Rs. 60,000 in 2015-2016 and 2016-2017 after charging depreciation of

9

5. (a) (i) Gourmet and high-end restaurants recognises Natural Spices on the basis of

its high quality of spices. Therefore, quality is most critical success factor of

Natural Spices. There are other factors which cannot be ignore such as price,

delivery options, attractive packing etc. But all are secondary to the quality.

(ii) Deliberate action of cutting price to increase sales volume indicates that firm is

intending to expand its market to retail market and street shops which is price

sensitive.

Purchase Price Variance is clearly indicating that firm has purchased raw

material at lower price which may be due to buying of lower quality of material.

Similarly positive Efficiency Variance is indicating cost cutting and stretching

resources.

It appears that firm is intending to expand its market to retail market and street

shops by not only reducing the price but also compromising its quality which is

opposing its current strategy of high quality.

Management should monitor the trends of variances on regular basis and take

appropriate action in case of evidence of permanent decline in quality. Here,

customer feedback is also very important.

(b) Workings

Statement Showing ‘Non-unit Level Overhead Costs’

Particulars Current Situation Proposed Situation

No. of Production Runs/Setups 42 40

960 runs×42 setup

1,008units

Cost per Setup Rs. 450 Rs. 360

Production Units per run 960 units 1,008 units

Production Units 40,320

(960 units × 42) 40,320

Engineering Hrs. 500 422

Engineering Cost per hour Rs. 10 Rs. 10

Page 106: Test Series: August, 2017 MOCK TEST PAPER 1 FINAL COURSE ... · The company incurred losses of Rs. 40,000 and Rs. 60,000 in 2015-2016 and 2016-2017 after charging depreciation of

10

Requirement of Question

(i) Break Even Point (Changed Scenario)

Break Even Point

Fixed Cost+ (Setup Cost×No.of Setups)+ (Engineering Costs×No.of Engineering Hrs.)

(Price - Unit Variable Cost)

= Rs. 72,100+ (Rs. 360×40Setups)+ (Rs.10×422 hrs.)

(Rs.10 - Rs. 5)

= 18,144 units

Break Even Point (No of Production Runs)

= BreakEven(units)

Production(unitsper run)

= 18,144units

1,008units

= 18 Runs

(ii) A company should adopt Activity Based Costing (ABC) system for accurate

product costing, as traditional volume based costing system does not take into

account the Non-unit Level Overhead Costs such as Setup Cost, Inspection

Cost, and Material Handling Cost etc. Cost Analysis under ABC system

showed that while these costs are largely fixed with respect to sales volume,

but they are not fixed to other appropriate cost drivers. If break up the

remaining Rs. 72,100 fixed costs consist of only a small portion of these costs,

ABC need not be applied.

However, it may also be noted that the primary study has resulted in cost

savings. If the savings in cost are expected to exceed the cost of study and

implementing ABC, it may be justified. Further it is pertinent to mention that

ABC offers no increase in product-costing accuracy for single-product setting.

6. (a) To overcome the optimum decision making and performance evaluation conflicts that can occur with marginal cost-based transfer pricing following methods has been proposed:

Dual Rate Transfer Pricing System

“With a ‘Dual Rate Transfer Pricing System’ the ‘Receiving Division’ is charged with marginal cost of the intermediate product and ‘Supplying Division’ is credited with full cost per unit plus a profit margin”.

Accordingly Division ’Dx’ should be allowed to record the transactions at full cost per unit plus a profit margin. On the other hand Division ‘Dz’ may be charged only

Page 107: Test Series: August, 2017 MOCK TEST PAPER 1 FINAL COURSE ... · The company incurred losses of Rs. 40,000 and Rs. 60,000 in 2015-2016 and 2016-2017 after charging depreciation of

11

marginal cost. Any inter divisional profits can be eliminated by accounting adjustment.

Impact:

− Division ’Dx’ will earn a profit on inter Division transfers.

− Division ’Dz’ can chose the output level at which the marginal cost of the

product ’X’ is equal to the net marginal revenue of the product ’Z’.

Two Part Transfer Pricing System:

“The ‘Two Part Transfer Pricing System’ involves transfers being made at the

marginal cost per unit of output of the supplying Division plus a lump-sum fixed fee

charged by the supplying Division to the receiving Division for the use of the

capacity allocated to the intermediate product.”

Accordingly Division ‘Dx’ can transfer its products to Division ‘Dz’ at marginal cost

per unit and a lump-sum fixed fee.

Impact:

− ‘Two Part Transfer Pricing System’ will inspire the Division ’Dz’ to choose the

optimal output level.

− This pricing system also enable the Division ’Dx’ to obtain a profit on inter

Division transfer.

(b) Cost is not only criterion for deciding in the favour of shut down. Non-cost factors

worthy of consideration in this regard are as follows:

(i) Interest of workers, if the workers are discharged, it may become difficult to get

skilled workers later, on reopening of the factory. Also shut-down may create

problems.

(ii) In the face of competition it may difficult to re-establish the market for the

product.

(iii) Plant may become obsolete or depreciate at a faster rate or get rusted. Thus,

heavy capital expenditure may have to be incurred on re-opening.

(c) Both Standard Costing and Kaizen Costing are helpful and used for measurement of

performance of a company but there are differences in approach between the two

systems.

Under Standard Costing system standards of all important variables like cost and

quantity of materials, labours and overheads are set at the beginning of the year or

activity. These set standards are compared with the actual performance to analyse

the variances. As a step further all variances are classified as planning and

operational variances to distinguish variances that are with in the manager’s control

Page 108: Test Series: August, 2017 MOCK TEST PAPER 1 FINAL COURSE ... · The company incurred losses of Rs. 40,000 and Rs. 60,000 in 2015-2016 and 2016-2017 after charging depreciation of

12

and beyond their effort. In brief Standard Costing and Variance Analysis helps in

determine the variances and take post event measures to stop recurrences.

On the other hand Kaizen Costing emphasises on continual improvement. Targets

once set at the beginning of the year or activities are updated continuously to reflect

the improvement that has already been achieved and that are yet to be achieved.

As a continuous improvement measure Kaizen Costing set new challenges before

the workers and managers and helps to improve and control the situation to achieve

desired target results. Therefore, if Kaizen costing is used in place of Standard

Costing and Variance analysis to measure performance then definitely it will keep

Arnav Automobile Ltd. competent enough to head on with the global automobile

players.

7. (a) Target cost is the difference between the estimated selling price of a proposed

product with specified functionality and quality and target margin. This is a cost

management technique that aims to produce and sell products that will ensure the

target margin. It is an integral part of the product design. While designing the

product the company allocates value and cost to different attributes and quality.

Therefore, they use the technique of value engineering and value analysis. The

target cost is achieved by assigning cost reduction targets to different operations

that are involved in the production process. Eventually, all operations do not

achieve the cost reduction targets, but the overall cost reduction target is achieved

through team work. Therefore, it is said that target costing fosters team work.

(b) Statement Showing “Pareto Analysis of Defects”

Defect Type No. of Items % of Total Items Cumulative Total

Scratches on the surface 110 36.67% 36.67%

Rough edges of lenses 70 23.33% 60.00%

Non-uniform grinding of lenses

60 20.00% 80.00%

Frame colours-shade differences

25 8.33% 88.33%

Power mismatches 20 6.67% 95.00%

End frame not equidistant from the centre

10 3.33% 98.33%

Spots/ Strain on lenses 5 1.67% 100.00%

300 100.00%

The company should focus on eliminating scratches on the surface, rough edges of

lenses and grinding of lenses related defects which constitute 80% portion,

according to Pareto Theory.

Page 109: Test Series: August, 2017 MOCK TEST PAPER 1 FINAL COURSE ... · The company incurred losses of Rs. 40,000 and Rs. 60,000 in 2015-2016 and 2016-2017 after charging depreciation of

13

(c) Sunk costs are costs that have been created by a decision made in the past and

that cannot be changed by any decision that will be made in the future.

Example, the written down value of assets previously purchased are sunk cost.

Sunk costs are not relevant for decision making because they are past cost.

But not all irrelevant costs are sunk cost. For example, a comparison of two

alternative production methods may result in identical material costs for both the

alternatives. In this case, the direct material cost will remain the same whichever

alternative in chosen. In this situation, through direct material cost is the future cost

to be incurred in accordance with the production, it is irrelevant, but it is not a sunk

cost.

Irrelevant is only with respect to alternatives being considered and not for fund flows

whereas for sunk cost there is no further cash flow. Cash flows have already been

incurred.

(d) Characteristic of the dual problem:

(i) For any linear programming model called primal model, there exists a

companion model called the dual model.

(ii) The number of constraints in the primal model equals the number of variables

in the dual model.

(iii) The number of variables in the primal model equals the number of constraints

in the dual model

(iv) If the primal model has a maximization objective then the dual model will have

a minimization objective and vice-versa. Inequalities get reversed.

(v) The solution of the primal model yields the solution of the dual model. Also, an

optimal simplex table for the dual model yields the optimal solution to the

primal model. Further, the objective functions of the two optimal tables will

have identical values.

(vi) Dual of the dual problem is the original primal itself.

(vii) Feasible solutions to a primal and dual problem are both optimal if the

complementary slackness conditions hold. If this relationship does not hold

either the primal solution or the dual solution or both are not optimal.

(viii) If the primal problem has no optimal solution due to infeasibility, then the dual

problem will have no optimal solution due to unboundedness.

(ix) If primal has no optimal solution due to unboundedness, than the dual will

have no optimal solution due to infeasibility.

Page 110: Test Series: August, 2017 MOCK TEST PAPER 1 FINAL COURSE ... · The company incurred losses of Rs. 40,000 and Rs. 60,000 in 2015-2016 and 2016-2017 after charging depreciation of

14

(e) Synchronous Manufacturing: In an all-encompassing management philosophy

which includes a set of principles, procedures and techniques where every action is

evaluated in terms of common goals of the organization.

The seven principles are:

(i) Focus on synchronizing the production flow than on idle capacities.

(ii) Value of time at a bottleneck resource is equal to the throughput rate of

products processed by the bottleneck.

(iii) Value of time at a non-bottleneck resource is negligible.

(iv) Level of utilization of a non-bottleneck resource is controlled by other

constraints within the system.

(v) Resources must be utilized, not simply activated.

(vi) Transfer batch should not be equal to process batch.

(vii) A process batch should be variable both along its route and overtime.

Page 111: Test Series: August, 2017 MOCK TEST PAPER 1 FINAL COURSE ... · The company incurred losses of Rs. 40,000 and Rs. 60,000 in 2015-2016 and 2016-2017 after charging depreciation of

1

Test Series: August, 2017

MOCK TEST PAPER- 1

FINAL COURSE GROUP-II

PAPER-6: INFORMATION SYSTEMS CONTROL & AUDIT

Question No.1 is compulsory.

Attempt any five questions from the remaining six questions.

Total Time: 3 hours Max. Marks: 100

1. ABC Industries Ltd. is a company engaged in a business of manufacturing and supply of electronic equipments to various companies in India. The company implemented an integrated E-governance system at all of its departments. The company’s new business models and new methods presume that the information required by the business managers is available all the time, it is accurate and reliable. The company is relying on Information Technology for information and transaction processing. It is also presumed that the company is up and running all time 24x7 basis. Also, the company intends to keep all its records in a digitized form.

(a) The company appoints an auditor to conduct audit of the existing Information

System. What are the steps an auditor would follow to conduct the audit of

Information Systems? (8 Marks)

(b) List down some user – related issues because of which an enterprise at times fails

to achieve the objectives of the system development. (6 Marks)

(c) What are the provisions given in Information Technology (Amendment) Act, 2008 for

the retention and audit of documents etc. in electronic form? (6 Marks)

2. (a) Discuss some examples of Decision Support Systems (DSS) in Accounting.

(6 Marks)

(b) Discuss the role of Auditor during verification of Environmental Controls. (6 Marks)

(c) Explain two primary methods, which are used for the analysis of the scope of a

project in SDLC. (4 Marks)

3. (a) Describe contents of a Disaster Recovery and Planning Document. (6 Marks)

(b) What are the major objectives of Systems Requirements Analysis phase in SDLC to

generate Systems Requirements Specification (SRS)? (6 Marks)

(c) Explain major advantages and limitations of Public Cloud. (4 Marks)

4. (a) Mr. 'X' has opened a new departmental store and all the activities are computerized.

He uses Personal Computers (PCs) for carrying out the business activities. As an IS

Page 112: Test Series: August, 2017 MOCK TEST PAPER 1 FINAL COURSE ... · The company incurred losses of Rs. 40,000 and Rs. 60,000 in 2015-2016 and 2016-2017 after charging depreciation of

2

auditor, list the risks related to the use of PCs in the business of Mr. 'X' and suggest

some security measures to be exercised to overcome them. (6 Marks)

(b) You are appointed by a leading enterprise to assess and to evaluate its system of IT

internal controls. What are the key management practices to be followed to carry

out the assignment complying with COBIT 5? (6 Marks)

(c) What are Detective Controls? (4 Marks)

5. (a) Discuss the seven enablers of COBIT 5. (8 Marks)

(b) Briefly discuss the components of ERP Model. (4 Marks)

(c) Describe major advantages of continuous audit techniques. (4 Marks)

6. (a) Discuss different ways of protecting the installation of Information Systems

Resources against fire damage. (6 Marks)

(b) Discuss the characteristics of an effective Management Information Systems (MIS).

(6 Marks)

(c) Discuss the “Data integrity Policies”. (4 Marks)

7. Write short notes on any four of the following:

(a) Key Governance practices of GEIT.

(b) Components of Information Security Policy.

(c) [Section 3] Authentication of Electronic Records under Information Technology

(Amendment) Act, 2008.

(d) Benefits of Mobile Computing.

(e) Strengths of Waterfall Model. (4 × 4 = 16 Marks)

Page 113: Test Series: August, 2017 MOCK TEST PAPER 1 FINAL COURSE ... · The company incurred losses of Rs. 40,000 and Rs. 60,000 in 2015-2016 and 2016-2017 after charging depreciation of

1

Test Series: August, 2017

MOCK TEST PAPER- 1

FINAL COURSE GROUP-II

PAPER-6: INFORMATION SYSTEMS CONTROL & AUDIT

SUGGESTED ANSWER/HINTS

1. (a) Steps that an Auditor follows during Information System Audit are as follows:

Steps in Information System Audit process

(i) Scoping and pre-audit survey: Auditors determine the main area/s of focus

and any areas that are explicitly out-of-scope, based on the scope-definitions

agreed with management. Information sources at this stage include

background reading and web browsing, previous audit reports, pre-audit

interview, observations and, sometimes, subjective impressions that simply

deserve further investigation.

(ii) Planning and preparation: During this step, the scope is broken down into

greater levels of detail, usually involving the generation of an audit work plan

or risk-control-matrix.

(iii) Fieldwork: This step involves gathering of evidence by interviewing staff and

managers, reviewing documents, and observing processes etc.

(iv) Analysis: This step involves desperately sorting out, reviewing and trying to

make sense of all that evidence gathered earlier. SWOT (Strengths,

Weaknesses, Opportunities, and Threats) or PEST (Political, Economic,

Social, and Technological) techniques can be used for analysis.

(v) Reporting: Reporting to the management is done after analysis of evidence is

gathered and analyzed.

(vi) Closure: Closure involves preparing notes for future audits and follow up with

management to complete the actions they promised after previous audits.

Scoping

Planning

Fieldwork

Analysis

Reporting

Close

Page 114: Test Series: August, 2017 MOCK TEST PAPER 1 FINAL COURSE ... · The company incurred losses of Rs. 40,000 and Rs. 60,000 in 2015-2016 and 2016-2017 after charging depreciation of

2

Analysis and reporting may involve the use of automated data analysis tools

such as ACL or IDEA, if not Excel, Access and hand-crafted SQL queries.

Automated system security analysis, configuration or vulnerability

management and security benchmarking tools are also used for reviewing

security parameters, and the basic security management functions that are

built-in to modern systems can help with log analysis, reviewing user access

rights etc.

(b) Some of the most notable user related issues because of which an enterprise at

times fails to achieve the objectives of the system development are described briefly

as follows:

Shifting User Needs: User requirements for IT are constantly changing. As

these changes accelerate, there will be more requests for Information systems

development and more development projects. When these changes occur

during a development process, the development team faces the challenge of

developing systems whose very purpose might change since the development

process began.

Resistance to Change: People have a natural tendency to resist change, and

information systems development projects signal changes - often radical - in

the workplace. When personnel perceive that the project will result in

personnel cutbacks, threatened personnel will dig in their heels, and the

development project is doomed to failure.

Lack of Users’ Participation: Users must participate in the development

efforts to define their requirements, feel ownership for project success, and

work to resolve development problems. User participation also helps to reduce

user resistance to change.

Inadequate Testing and User Training: New systems must be tested before

installation to determine that they operate correctly. Users must be trained to

effectively utilize the new system.

(c) In Information Technology (Amendment) Act, 2008; Section 7 is related to the

“Retention of Electronic Records” and Section 7A is related to the “Audit of

documents in electronic form”.

[Section 7] Retention of Electronic Records

(1) Where any law provides that documents, records or information shall be

retained for any specific period, then, that requirement shall be deemed to

have been satisfied if such documents, records or information are retained in

the electronic form, -

(a) the information contained therein remains accessible so as to be usable

for a subsequent reference;

Page 115: Test Series: August, 2017 MOCK TEST PAPER 1 FINAL COURSE ... · The company incurred losses of Rs. 40,000 and Rs. 60,000 in 2015-2016 and 2016-2017 after charging depreciation of

3

(b) the electronic record is retained in the format in which it was originally

generated, sent or received or in a format which can be demonstrated to

represent accurately the information originally generated, sent or

received;

(c) the details which will facilitate the identification of the origin, destination,

date and time of dispatch or receipt of such electronic record are

available in the electronic record.

However, this clause does not apply to any information which is automatically

generated solely for the purpose of enabling an electronic record to be

dispatched or received.

(2) Nothing in this section shall apply to any law that expressly provides for the

retention of documents, records or information in the form of electronic

records, publication of rules, regulation, etc. in Electronic Gazette.

[Section 7A] Audit of Documents etc. in Electronic form

Where in any law for the time being in force, there is a provision for audit of

documents, records or information, that provision shall also be applicable for

audit of documents, records or information processed and maintained in

electronic form.

2. (a) Examples of Decision Support Systems in Accounting – Many DSS are

developed in-house using either a general type of decision support program or a

spreadsheet program to solve specific problems. Below are several illustrations of

these systems:

o Cost Accounting System - The health care industry is well known for its cost complexity. Managing costs in this industry require controlling costs of supplies, expensive machinery, technology, and a variety of personnel. Cost accounting applications help health care enterprises calculate product costs for individual procedures or services. One health care organization, for example, combines a variety of DSS applications in productivity, cost accounting, case mix, and nursing staff scheduling to improve its management decision making.

o Capital Budgeting System - Companies require new tools to evaluate high-technology investment decisions. Decision makers need to supplement analytical techniques, such as net present value and internal rate of return, with decision support tools that consider some benefits of new technology not captured in strict financial analysis. One DSS designed to support decisions about investments in automated manufacturing technology is Auto Man, which allows decision makers to consider financial, non financial, quantitative, and qualitative factors in their decision-making processes. Using this decision support system, accountants, managers, and engineers identify and prioritize these factors. Then they can evaluate up to seven investment alternatives at once.

Page 116: Test Series: August, 2017 MOCK TEST PAPER 1 FINAL COURSE ... · The company incurred losses of Rs. 40,000 and Rs. 60,000 in 2015-2016 and 2016-2017 after charging depreciation of

4

o Budget Variance Analysis System - Financial institutions rely heavily on their budgeting systems for controlling costs and evaluating managerial performance. One institution uses a computerized DSS to generate monthly variance reports for division comptrollers. The system allows these comptrollers to graph, view, analyze, and annotate budget variances, as well as create additional one-and five-year budget projections using the forecasting tools provided in the system. The decision support system thus helps the comptrollers create and control budgets for the cost-center managers reporting to them.

o General Decision Support System - As mentioned earlier, some planning languages used in Decision Support Systems are general purpose and therefore have the ability to analyze many different types of problems. In a sense, these types of decision support systems are a decision-maker’s tools. The user needs to input data and answer questions about a specific problem domain to make use of this type of decision support system. An example is a program called Expert Choice which supports a variety of problems requiring decisions. The user works interactively with the computer to develop a hierarchical model of the decision problem. The DSS then asks the user to compare decision variables with each other. For instance, the system might ask the user how important cash inflows are versus initial investment amount to a capital budgeting decision. The decision maker also makes judgments about which investment is best with respect to these cash flows and which requires the smallest initial investment. Expert choice analyzes these judgments and presents the decision maker with the best alternative .

(b) Audit of environmental controls requires the IS auditor to conduct physical inspections

and observe practices. The Auditor should verify:

The IPF (Infrastructure Planning and Facilities) and the construction with regard to the type of materials used for construction;

The presence of water and smoke detectors, power supply arrangements to such devices, and testing logs;

The location of fire extinguishers, firefighting equipment and refilling date of fire extinguishers;

Emergency procedures, evacuation plans and marking of fire exists. There should be half-yearly Fire drill to test the preparedness;

Documents for compliance with legal and regulatory requirements with regards to fire safety equipment, external inspection certificate and shortcomings pointed out by other inspectors/auditors;

Power sources and conduct tests to assure the quality of power, effectiveness of the power conditioning equipment, and generators. Also the power supply interruptions must be checked to test the effectiveness of the back-up power;

Page 117: Test Series: August, 2017 MOCK TEST PAPER 1 FINAL COURSE ... · The company incurred losses of Rs. 40,000 and Rs. 60,000 in 2015-2016 and 2016-2017 after charging depreciation of

5

Environmental control equipment such as air-conditioning, dehumidifiers, heaters, ionizers etc.;

Compliant logs and maintenance logs to assess if MTBF and MTTR are within acceptable levels; and

Identify undesired activities such as smoking, consumption of eatables etc.

(c) Two primary methods with the help of which the scope of the project can be

analyzed are given as follows:

Reviewing Internal Documents: The analysts conducting the investigation

first try to learn about the organization involved in, or affected by, the project.

For example, to review an inventory system proposal, an analyst may try to

know how does the inventory department operates and who are the managers

and supervisors. Analysts can usually learn these details by examining

organization charts and studying written operating procedures.

Conducting Interviews: Written documents tell the analyst how the systems

should operate, but they may not include enough details to allow a decision to

be made about the merits of a systems proposal, nor do they present users'

views about current operations. To learn these details, analysts use interviews.

Interviews allow analysts to know more about the nature of the project request

and the reasons for submitting it. Usually, preliminary investigation interviews

involve only management and supervisory personnel.

3. (a) The Disaster Recovery Planning document may include the following areas:

The conditions for activating the plans, which describe the process to be followed before each plan, are activated.

Emergency procedures, which describe the actions to be taken following an incident which jeopardizes business operations and/or human life. This should include arrangements for public relations management and for effective liaisoning with appropriate public authorities e.g. police, fire, services and local government.

Fallback procedures, which describe the actions to be taken to move essential business activities or support services to alternate temporary locations, to bring business process back into operation in the required time-scale.

Resumption procedures, which describe the actions to be taken to return to normal business operations.

A maintenance schedule, which specifies ‘how and when the plan will be tested’, and the process for maintaining the plan.

Awareness and education activities, which are designed to create an understanding of the business continuity, process and ensure that the business continues to be effective.

Page 118: Test Series: August, 2017 MOCK TEST PAPER 1 FINAL COURSE ... · The company incurred losses of Rs. 40,000 and Rs. 60,000 in 2015-2016 and 2016-2017 after charging depreciation of

6

The responsibilities of individuals describing who is responsible for executing which component of the plan. Alternatives should be nominated as required.

Contingency plan document distribution list.

Detailed description of the purpose and scope of the plan.

Contingency plan testing and recovery procedure.

List of vendors doing business with the organization, their contact numbers and address for emergency purposes.

Checklist for inventory taking and updating the contingency plan on a regular basis.

List of phone numbers of employees in the event of an emergency.

Emergency phone list for fire, police, hardware, software, suppliers, customers, back-up location, etc.

Medical procedure to be followed in case of injury.

Back-up location contractual agreement, correspondences.

Insurance papers and claim forms.

Primary computer center hardware, software, peripheral equipment and software configuration.

Location of data and program files, data dictionary, documentation manuals, source and object codes and back-up media.

Alternate manual procedures to be followed such as preparation of invoices.

Names of employees trained for emergency situation, first aid and life saving techniques.

Details of airlines, hotels and transport arrangements.

(b) The major objectives of Systems Requirements Analysis phase in SDLC to generate

Systems Requirements Specification (SRS) are as follows:

To identify and consult the stake owners to determine their expectations and resolve their conflicts;

To analyze requirements to detect and correct conflicts and determine priorities;

To gather data or find facts using tools like - interviewing, research/document collection, questionnaires, observation;

To verify that the requirements are complete, consistent, unambiguous, verifiable, modifiable, testable and traceable;

To model activities such as developing models to document Data Flow Diagrams, E-R Diagrams; and

Page 119: Test Series: August, 2017 MOCK TEST PAPER 1 FINAL COURSE ... · The company incurred losses of Rs. 40,000 and Rs. 60,000 in 2015-2016 and 2016-2017 after charging depreciation of

7

To document activities such as interview, questionnaires, reports etc. and development of a system (data) dictionary to document the modeling activities.

(c) The Advantages of Public Cloud include the following:

It is widely used in the development, deployment and management of enterprise applications, at affordable costs.

It allows the organizations to deliver highly scalable and reliable applications rapidly and at more affordable costs.

There is no need for establishing infrastructure for setting up and maintaining the cloud.

Strict SLAs are followed.

There is no limit for the number of users.

Limitations of Public Cloud are as follows:

One of the limitation of Public Cloud is security assurance and thereby building trust among the clients is far from desired but slowly liable to happen.

Further, privacy and organizational autonomy are not possible.

4. (a) Risks related to the use of Personal Computers (PCs) in the business are as

follows:

Personal computers are small and easy to connect and disconnect, they are likely to be shifted from one location to another or even taken outside the organization for theft of information.

Pen drives can be very conveniently transported from one place to another, as a result of which data theft may occur. Even hard disks can be ported easily these days.

PC is basically a single user oriented machine and hence, does not p rovide inherent data safeguards. Problems can be caused by computer viruses and pirated software, namely, data corruption, slow operations and system break down etc.

Segregation of duty is not possible, owing to limited number of staff.

Due to vast number of installations, the staff mobility is higher and hence becomes a source of leakage of information.

The operating staff may not be adequately trained.

Weak access control: Most of the log-on procedures become active at the booting of the computer from the hard drive.

The Security Measures that could be exercised to overcome these aforementioned

risks are given as follows:

Physically locking the system;

Page 120: Test Series: August, 2017 MOCK TEST PAPER 1 FINAL COURSE ... · The company incurred losses of Rs. 40,000 and Rs. 60,000 in 2015-2016 and 2016-2017 after charging depreciation of

8

Proper logging of equipment shifting must be done;

Centralized purchase of hardware and software;

Standards set for developing, testing and documenting;

Uses of antimalware software; and

The use of personal computer and their peripheral must have controls.

Use of disc locks that prevent unauthorized access to floppy disk or pen drive of a

computer.

(b) The key management practices complying with COBIT 5 for assessing and

evaluating the system of IT internal controls in an enterprise are given as follows:

Monitor Internal Controls: Continuously monitor, benchmark and improve the

IT control environment and control framework to meet organizational

objectives.

Review Business Process Controls Effectiveness: Review the operation of

controls, including a review of monitoring and test evidence to ensure that

controls within business processes operate effectively. It also includes

activities to maintain evidence of the effective operation of controls through

mechanisms such as periodic testing of controls, continuous controls

monitoring, independent assessments, command and control centers, and

network operations centers.

Perform Control Self-assessments: Encourage management and process

owners to take positive ownership of control improvement through a continuing

program of self- assessment to evaluate the completeness and effectiveness

of management’s control over processes, policies and contracts.

Identify and Report Control Deficiencies: Identify control deficiencies and

analyze and identify their underlying root causes. Escalate control deficiencies

and report to stakeholders.

Ensure that assurance providers are independent and qualified: Ensure

that the entities performing assurance are independent from the function,

groups or organizations in scope. The entities performing assurance should

demonstrate an appropriate attitude and appearance, competence in the skills

and knowledge necessary to perform assurance, and adherence to codes of

ethics and professional standards.

Plan Assurance Initiatives: Plan assurance initiatives based on enterprise

objectives and conformance objectives, assurance objectives and strategic

priorities, inherent risk resource constraints, and sufficient knowledge of the

enterprise.

Page 121: Test Series: August, 2017 MOCK TEST PAPER 1 FINAL COURSE ... · The company incurred losses of Rs. 40,000 and Rs. 60,000 in 2015-2016 and 2016-2017 after charging depreciation of

9

Scope assurance initiatives: Define and agree with management on the

scope of the assurance initiative, based on the assurance objectives.

Execute assurance initiatives: Execute the planned assurance initiative.

Report on identified findings. Provide positive assurance opinions, where

appropriate, and recommendations for improvement relating to identified

operational performance, external compliance and internal con trol system

residual risks.

(c) Detective Controls: These controls are designed to detect errors, omissions or

malicious acts that occur and report the occurrence. An example of a detective

control would be a use of automatic expenditure profiling where management gets

regular reports of spend to date against profiled spend. The main characteristics of

such controls are given as follows:

Clear understanding of lawful activities so that anything which deviates from these is reported as unlawful, malicious, etc.;

An established mechanism to refer the reported unlawful activities to the appropriate person or group;

Interaction with the preventive control to prevent such acts from occurring; and

Surprise checks by supervisor.

Examples of detective controls include Hash totals; Check points in production jobs;

Echo control in telecommunications; Error message over tape labels; Duplicate

checking of calculations; Periodic performance reporting with variances; Past-due

accounts report; The internal audit functions; Intrusion detection system; Cash counts

and bank reconciliation, and monitoring expenditures against budgeted amount.

5. (a) Enablers are factors that, individually and collectively, influence whether something

will work; in this case, governance and management over enterprise IT. Enablers

are driven by the goals cascade, i.e., higher-level IT related goals defining ‘what the

different enablers should achieve’. The COBIT 5 framework describes seven

categories of enablers, which are discussed as follows:

Principles, Policies and Frameworks are the vehicle to translate the desired behaviour into practical guidance for day-to-day management.

Processes describe an organized set of practices and activities to achieve certain objectives and produce a set of outputs in support of achieving overall IT-related goals.

Organizational structures are the key decision-making entities in an enterprise.

Culture, Ethics and Behaviour of individuals and of the enterprise are very often underestimated as a success factor in governance and management activities.

Page 122: Test Series: August, 2017 MOCK TEST PAPER 1 FINAL COURSE ... · The company incurred losses of Rs. 40,000 and Rs. 60,000 in 2015-2016 and 2016-2017 after charging depreciation of

10

Information is pervasive throughout any organization and includes all information produced and used by the enterprise. Information is required for keeping the organization running and well governed, but at the operational level, information is very often the key product of the enterprise itself.

Services, Infrastructure and Applications include the infrastructure, technology and applications that provide the enterprise with information technology processing and services.

People, Skills and Competencies are linked to people and are required for successful completion of all activities and for making correct decisions and taking corrective actions.

(b) Components of ERP model are as follows:

ERP model is consists of four components which are implemented through a

methodology. All four components are as follows:

(i) Software Component: The software component is the component that is most visible part and consists of several modules such as Finance, Human Resource, Supply Chain Management, Supplier Relationship Management, Customer Relationship, and Business Intelligent.

(ii) Process Flow: It is the model that illustrates the way how information flows among the different modules within an ERP system. By creating this model makes it easier to understand how ERP work.

(iii) Customer mindset: By implementing ERP system, the old ways for working which user understand and comfortable with have to be changed and may lead to users’ resistance. For example, some users may say that they have spent many years doing an excellence job without help from ERP system. In order to lead ERP implementation to succeed, the company needs to eliminate negative value or belief that users may carry toward utilizing new system.

(iv) Change Management: In ERP implementation, change needs to be managed at several levels - User attitude; resistance to change; and Business process changes.

(c) Some of the advantages of continuous audit techniques are given as under:

Timely, Comprehensive and Detailed Auditing – Evidence would be available more timely and in a comprehensive manner. The entire processing can be evaluated and analyzed rather than examining the inputs and the outputs only.

Surprise test capability – As evidences are collected from the system itself by using continuous audit techniques, auditors can gather evidence without the systems staff and application system users being aware that evidence is being collected at that particular moment. This brings in the surprise test advantages.

Page 123: Test Series: August, 2017 MOCK TEST PAPER 1 FINAL COURSE ... · The company incurred losses of Rs. 40,000 and Rs. 60,000 in 2015-2016 and 2016-2017 after charging depreciation of

11

Information to system staff on meeting of objectives - Continuous audit techniques provides information to systems staff regarding the test vehicle to be used in evaluating whether an application system meets the objectives of asset safeguarding, data integrity, effectiveness, and efficiency.

Training for new users – Using the ITFs, new users can submit data to the application system, and obtain feedback on any mistakes they make via the system’s error reports.

6. (a) Some of the major ways of protecting the installation against fire damage are as

follows:

o Both automatic and manual fire alarms may be placed at strategic locations and a control panel may be installed to clearly indicate this.

o Besides the control panel, master switches may be installed for power and automatic fire suppression system. Different fire suppression techniques like Dry-pipe sprinkling systems, water based systems, halon etc., depending upon the situation may be used.

o Manual fire extinguishers can be placed at strategic locations.

o Fireproof Walls; Floors and Ceilings surrounding the Computer Room and Fire Resistant Office Materials such as wastebaskets, curtains, desks, and cabinets should be used.

o Fire exits should be clearly marked. When a fire alarm is activated, a signal may be sent automatically to permanently manned station.

o All staff members should know how to use the system. The procedures to be followed during an emergency should be properly documented are Fire Alarms, Extinguishers, Sprinklers, Instructions / Fire Brigade Nos., Smoke detectors, and Carbon dioxide based fire extinguishers.

o Less Wood and plastic should be in computer rooms.

o Use a gas based fire suppression system;

o To reduce the risk of firing, the location of the computer room should be strategically planned and should not be located in the basement or ground floor of a multi-storey building.

o Regular Inspection by Fire Department should be conducted.

o Fire repression systems should be supplemented and not replaced by smoke detectors.

o Documented and Tested Emergency Evacuation Plans: Relocation plans should emphasize human safety, but should not leave information processing facilities physically unsecured. Procedures should exist for a controlled shutdown of the computer in an emergency situation. In all circumstances saving human life should be given paramount importance.

Page 124: Test Series: August, 2017 MOCK TEST PAPER 1 FINAL COURSE ... · The company incurred losses of Rs. 40,000 and Rs. 60,000 in 2015-2016 and 2016-2017 after charging depreciation of

12

o Smoke Detectors: Smoke detectors are positioned at places above and below the ceiling tiles. Upon activation, these detectors should produce an audible alarm and must be linked to a monitored station (for example, a fire station).

o Wiring Placed in Electrical Panels and Conduit: Electrical fires are always a risk. To reduce the risk of such a fire occurring and spreading, wiring should be placed in the fire-resistant panels and conduit. This conduit generally lies under the fire-resistant raised floor in the computer room.

(b) Characteristics of an effective MIS: Major characteristic of an effective MIS are

given as follows:

o Management Oriented – It means that efforts for the development of the information system should start from an appraisal of management needs and overall business objectives. Such a system is not necessarily for top management only but may also meet the information requirements of middle level or operating levels of management as well.

o Management Directed – Because of management orientation of MIS, it is necessary that management should actively direct the system’s development efforts. For system’s effectiveness, it is necessary for management to devote their sufficient time not only at the stage of designing the system but for its review as well to ensure that the implemented system meets the specifications of the designed system.

o Integrated – The best approach for developing information systems is the integrated approach as all the functional and operational information sub-systems are tied together into one entity. An integrated Information system has the capability of generating more meaningful information to management as it takes a comprehensive view or a complete look at the interlocking sub-systems that operate within a company.

o Common Data Flows – It means the use of common input, processing and output procedures and media whenever required. Data is captured by the system analysts only once and as close to its original source as possible. Afterwards, they try to utilize a minimum of data processing procedures and sub-systems to process the data and strive to minimize the number of output documents and reports produced by the system. This eliminates duplication in data collections, simplifies operations and produces an efficient information system.

o Heavy Planning Element – An MIS usually takes one to three years and sometimes even longer period to get established firmly within a company. Therefore, a MIS designer must be present in MIS development and should consider future enterprise objectives and requirements of information as per the organization structure of the enterprise as per requirements.

o Sub System Concept – Even though the information system is viewed as a single entity, it must be broken down into digestible sub-systems, which can be implemented one at a time by developing a phased plan. The breaking down of

Page 125: Test Series: August, 2017 MOCK TEST PAPER 1 FINAL COURSE ... · The company incurred losses of Rs. 40,000 and Rs. 60,000 in 2015-2016 and 2016-2017 after charging depreciation of

13

MIS into meaningful sub-systems sets the stage for this phasing plan.

o Common Database – Database is the mortar that holds the functional systems together. It is defined as a "super-file", which consolidates and integrates data records formerly stored in many separate data files. The organization of a database allows it to be accessed by several information sub-systems and thus, eliminates the necessity of duplication in data storage, updating, deletion and protection.

o Computerized - Though MIS can be implemented without using a computer; the use of computers increases the effectiveness of the system. In fact, its use equips the system to handle a wide variety of applications by providing their information requirements quickly. Other necessary attributes of the computer to MIS are accuracy and consistency in processing data and reduction in clerical staff. These attributes make computer a prime requirement in management information system.

(c) Major Data Integrity policies are given as under:

Virus-Signature Updating: Virus signatures must be updated automatically when they are made available from the vendor through enabling of automatic updates.

Software Testing: All software must be tested in a suitable test environment before installation on production systems.

Division of Environments: The division of environments into Development, Test, and Production is required for critical systems.

Offsite Backup Storage: Backups older than one month must be sent offsite for permanent storage.

Quarter-End and Year-End Backups: Quarter-end and year-end backups must be done separately from the normal schedule, for accounting purposes

Disaster Recovery: A comprehensive disaster-recovery plan must be used to ensure continuity of the corporate business in the event of an outage.

7. (a) The key governance practices required to implement GEIT in enterprises are

highlighted here:

Evaluate the Governance System: Continually identify and engage with the

enterprise's stakeholders, document an understanding of the requirements,

and make judgment on the current and future design of governance of

enterprise IT;

Direct the Governance System: Inform leadership and obtain their support,

buy-in and commitment. Guide the structures, processes and practices for the

governance of IT in line with agreed governance design principles, decision-

making models and authority levels. Define the information required for

informed decision making; and

Page 126: Test Series: August, 2017 MOCK TEST PAPER 1 FINAL COURSE ... · The company incurred losses of Rs. 40,000 and Rs. 60,000 in 2015-2016 and 2016-2017 after charging depreciation of

14

Monitor the Governance System: Monitor the effectiveness and performance

of the enterprise’s governance of IT. Assess whether the governance system

and implemented mechanisms (including structures, principles and processes)

are operating effectively and provide appropriate oversight of IT.

(b) A good security policy should clearly state the following:

Purpose and Scope of the Document and the intended audience;

The Security Infrastructure;

Security policy document maintenance and compliance requirements;

Incident response mechanism and incident reporting;

Security organization Structure;

Inventory and Classification of assets;

Description of technologies and computing structure;

Physical and Environmental Security;

Identity Management and access control;

IT Operations management;

IT Communications;

System Development and Maintenance Controls;

Business Continuity Planning;

Legal Compliances; and

Monitoring and Auditing Requirements.

(c) [Section 3] Authentication of Electronic Records

(1) Subject to the provisions of this section any subscriber may authenticate an

electronic record by affixing his Digital Signature.

(2) The authentication of the electronic record shall be effected by the use of

asymmetric crypto system and hash function which envelop and transform the

initial electronic record into another electronic record.

Explanation -

For the purposes of this sub-section, "Hash function" means an algorithm

mapping or translation of one sequence of bits into another, generally smaller,

set known as "Hash Result" such that an electronic record yields the same

hash result every time the algorithm is executed with the same electronic

record as its input making it computationally infeasible

(a) to derive or reconstruct the original electronic record from the hash result

produced by the algorithm;

Page 127: Test Series: August, 2017 MOCK TEST PAPER 1 FINAL COURSE ... · The company incurred losses of Rs. 40,000 and Rs. 60,000 in 2015-2016 and 2016-2017 after charging depreciation of

15

(b) that two electronic records can produce the same hash result using the

algorithm.

(3) Any person by the use of a public key of the subscriber can verify the

electronic record.

(4) The private key and the public key are unique to the subscriber and constitute

a functioning key pair.

(d) Benefits of Mobile Computing are as follows:

It provides mobile workforce with remote access to work order details, such as work order location, contact information, required completion date, asset history relevant warranties/service contracts.

It enables mobile sales personnel to update work order status in real -time, facilitating excellent communication.

It facilitates access to corporate services and information at any time, from anywhere.

It provides remote access to the corporate Knowledgebase at the job location.

It enables to improve management effectiveness by enhancing information quality, information flow, and ability to control a mobile workforce .

(e) Strengths of Waterfall Model: The fundamental strength of the waterfall model has

made it quite popular and handy among the fraternity. Major strengths are given as

follows:

It is ideal for supporting less experienced project teams and project managers or project teams, whose composition fluctuates.

The orderly sequence of development steps and design reviews help to ensure the quality, reliability, adequacy and maintainability of the developed software.

Progress of system development is measurable.

It enables to conserve resources.

Page 128: Test Series: August, 2017 MOCK TEST PAPER 1 FINAL COURSE ... · The company incurred losses of Rs. 40,000 and Rs. 60,000 in 2015-2016 and 2016-2017 after charging depreciation of

1

Test Series: August, 2017

MOCK TEST PAPER – 1

FINAL: GROUP – II

PAPER – 7: DIRECT TAX LAWS

Question 1 is compulsory

Answer any five questions from the remaining six questions

Time Allowed – 3 Hours Maximum Marks – 100

1. (a) M/s. ABC Corporation, a partnership firm, has earned a gross total income of Rs. 300

lacs for the year ended 31-3-2017. The firm has not undertaken any international

transaction or specified domestic transaction during the said year.

The above includes a profit of Rs. 220 lacs from an industrial undertaking having a

turnover of Rs. 80 crores. This is the fourth year and deduction under section 80-IA

of the Income-tax Act, 1961 is available to the extent of Rs. 200 lacs.

There are some grey areas in the taxation workings and hence, the assessee is

contemplating to file the return of income on 10.11.2017, after seeking clarifications

from tax experts.

Advise the assessee-firm by working out the total income and tax payable, where the

return is filed on 30-09-2017 or when the same is filed on 10-11-2017.

What is the practical solution as regards obtaining clarifications, which might or might

not have an impact on the total income? (10 Marks)

(b) MNO trust, a business trust, registered under SEBI (Real Estate Investment Trusts)

Regulations, 2014, gives particulars of its income for the P.Y.2016-17:

(1) Interest income from High Ltd. – Rs. 4 crore;

(2) Dividend income from High Ltd. – Rs. 2 crore (received on 1st October, 2016);

(3) Short-term capital gains on sale of listed shares of High Ltd. – Rs. 1.5 crore;

(4) Interest received from investments in unlisted debentures of real estate

companies – Rs. 10 lakh;

(5) Rental income from directly owned real estate assets – Rs. 3.50 crore

High Ltd. is an Indian company in which the MNO trust holds controlling interest. The

MNO trust holds 100% of the shareholding of High Ltd.

Discuss the tax consequences of the above income earned by the MNO trust in its

hands and in the hands of the unit holders, assuming that the MNO trust has

distributed Rs. 10 crore to the unit holders in the P.Y.2016-17. (10 Marks)

Page 129: Test Series: August, 2017 MOCK TEST PAPER 1 FINAL COURSE ... · The company incurred losses of Rs. 40,000 and Rs. 60,000 in 2015-2016 and 2016-2017 after charging depreciation of

2

2. Ram, Shyam and HUF of Mohan (represented by Mohan) are partners with equal shares

in profits and losses of a firm, M/s Popular Tele films, which is engaged in the production

of TV serials and telefilms. In the previous year 2015-16, one partner ‘Sunder’ retired, but

his dues have been settled in the previous year 2016-17.

The earlier partnership deed did not authorise payment of remuneration or interest to

partners. The partnership deed was revised by the partners on 1st June, 2016 to authorise

payment of remuneration of Rs. 1 lac per month to each working partner and simple interest

at 15% per annum on partners’ capital. Ram, Shyam and Mohan are actively associated

with the affairs of the firm.

The Profit & Loss Account of the firm for the year ended 31st March, 2017 shows a net

profit of Rs. 10 lacs after debiting/crediting the following:

(a) Interest amounting to Rs.5 lacs each was paid to partners on the balances standing

to their capital accounts from 1stJune, 2016 to 31st March, 2017.

(b) Remuneration to the partners including partner in representative capacity Rs. 30 lacs.

(c) Interest amounting to Rs. 2 lacs paid to Mohan on loan provided by him in his

individual capacity at 16% interest.

(d) Royalty of Rs. 5 lacs paid to partner Ram, who is a professional script writer, for use

of his scripts as per agreement between the firm and Ram. The same is authorized

by partnership deed.

(e) Two separate payments of Rs. 17,000 and Rs. 16,000 made in cash on 1st March,

2017 to Asif, a hairdresser, against his bill for services rendered in February, 2017

and two payments of Rs. 16,000 and Rs. 18,000 made in cash on 1st February and

2nd February, 2017, respectively, to Prakash, an assistant cameraman, against her

bill for services provided in January, 2017.

(f) Amount of Rs. 4.50 lacs provided in the books on 31st March 2017 as liability for

remuneration to Shreya, a film artist and a non-resident. Tax deducted at source

under section 195 from the amount so credited was paid on 5th July, 2017.

(g) Amount of Rs. 7 lacs provided as gratuity for the year on the basis of actuarial

valuation. Gratuity actually paid to one retired employee during the year is Rs. 2.50

lacs.

(h) Interest of Rs. 1.20 lacs received on income-tax refund under section 244(1A) in

respect of A.Y. 2014-15.

The firm has also provided the following additional information:

The amount due to Sunder, an ex-partner, was Rs. 15 lacs which was settled on 30 th

September, 2016 by transferring a plot of land purchased two years back having book

value of Rs. 10 lacs. The difference of Rs. 5 lacs was credited to partners' capital accounts

Page 130: Test Series: August, 2017 MOCK TEST PAPER 1 FINAL COURSE ... · The company incurred losses of Rs. 40,000 and Rs. 60,000 in 2015-2016 and 2016-2017 after charging depreciation of

3

in their profit sharing ratio. The value of plot for stamp duty valuation on the date of transfer

was Rs. 16 lacs.

Compute the total income of the firm for the assessment year 2017-18 stating the reasons

for treatment of each item. (16 Marks)

3. (a) Hyper Private Ltd. went into liquidation on 30.06.2016. The company was seized and

possessed of the following funds prior to the distribution of assets to the shareholders:

Rs.

Share Capital (issued on 01.04.2012) 5,00,000

Reserves prior to 30.6.2016 3,00,000

Excess realization in the course of liquidation 5,00,000

Total 13,00,000

There are 5 shareholders, each of whom received Rs. 2,60,000 from the liquidator in

full settlement. The shareholders desire to invest the resultant element of capital

gains in long-term specified assets as defined in section 54EC. You are required to

examine the various issues and advice the shareholders about their liability to income

tax. (6 Marks)

(b) “Units set up in an International Financial Services Centre are entitled to special tax

concessions”. Discuss. (6 Marks)

(c) Mr. Kashyap held 12% equity shares in MNO Ltd., a private limited company. He

gifted all the shares held by him in MNO Ltd., to his wife Mrs. Kashyap on 23/7/2016.

The transfer was made without adequate consideration. On 5/8/2016, Mrs. Kashyap

obtained a loan of Rs. 75,000 from MNO Ltd., when the company's accumulated profit

was Rs. 50,000. What are the tax implications of the above transactions? (4 Marks)

4. (a) Foodie Ltd., engaged in the business of owning, operating and managing hotels,

allowed the employees to receive tips from the customers, by virtue of their

employment. The tips were also collected directly by the hotel -company from the

customers, when payment was made by them through credit cards. The hotel -

company thereafter disbursed the tips to the employees. The Assessing Officer

treated the receipt of the tips as income under the head “Salary” in the hands of the

various employees and held that the company was liable to deduct tax at source from

such payments under section 192. Since the company had not deducted tax at source

on such payments, the Assessing Officer treated the company as an assessee-in-

default under section 201(1). Discuss the correctness of the action of the Assessing

Officer. (4 Marks)

(b) Vikram, an individual, owned two residential houses which were let out. Besides, he

and his three brothers co-owned a residential house in equal shares. He sold one

residential house owned by him during the previous year relevant to the assessment

year 2017-18. Within two month from the date of such sale, the three brothers

Page 131: Test Series: August, 2017 MOCK TEST PAPER 1 FINAL COURSE ... · The company incurred losses of Rs. 40,000 and Rs. 60,000 in 2015-2016 and 2016-2017 after charging depreciation of

4

executed a release deed in respect of their shares in the co-owned residential house

in favour of Vikram for a monetary consideration.

Vikram utilised the entire long-term capital gain arising out of the sale of the

residential house for payment of the said consideration to his three brothers. Vikram

has let out the house, in respect of which his brothers executed a release deed, to

another person, who is using it for his residential purposes.

Is Vikram eligible for exemption under section 54 of the Income-tax Act, 1961 for the

assessment year 2017-18 in respect of the long-term capital gain arising from the

sale of his residential house, which he utilised for acquiring the shares of his brothers

in the co-owned residential house? Will the non-use of the new house for his own

residential purposes disentitle him to exemption? (4 Marks)

(c) Mr. Deepak is a distributor of lottery tickets. He won Rs. 7,00,000 as prize money on

unsold lottery tickets. It was offered as business income. The Assessing Officer wants

to tax the same as lottery winning at the rate prescribed under section 115BB. Is he

justified? (4 Marks)

(d) Explain the tax treatment of emergency spares (of plant and machinery) acquired

during the year which, even though kept ready for use, have not actually been used

during the relevant previous year. (4 Marks)

5. (a) Mr. Raj has commenced the business of manufacture of computers on 1.4.2016. He

employed 350 new employees during the P.Y.2016-17, the details of whom are as

follows -

No. of employees

Date of employment

Regular/Casual Total monthly emoluments per employee (Rs.)

(i) 75 1.4.2016 Regular 24,000

(ii) 125 1.5.2016 Regular 26,000

(iii) 50 1.8.2016 Casual 25,500

(iv) 100 1.9.2016 Regular 24,000

The regular employees participate in recognized provident fund while the casual

employees do not.

Compute the deduction, if any, available to Mr. Raj for A.Y.2017-18, if the profits and

gains derived from manufacture of computers that year is Rs. 75 lakhs and his total

turnover is Rs. 2.16 crores.

What would be your answer if Mr. Raj has commenced the business of manufacture

of apparel (and not computers) on 1.4.2017 and the above particulars are related to

such business? (6 Marks)

Page 132: Test Series: August, 2017 MOCK TEST PAPER 1 FINAL COURSE ... · The company incurred losses of Rs. 40,000 and Rs. 60,000 in 2015-2016 and 2016-2017 after charging depreciation of

5

(b) Ms. Kavita, a resident individual, is a magician deriving income of Rs. 6,00,000 from

a magic show performed outside India. Tax of Rs. 1,50,000 was deducted at source

in the country where she had performed the show. India does not have any double

tax avoidance agreement with that country. Her income in India amounted to

Rs. 42,00,000. Compute her tax liability for the assessment year 2017-18 assuming

she has deposited Rs. 1,50,000 in Public Provident Fund and paid medical insurance

premium in respect of her mother, resident in India, aged 61 years, Rs. 35,000.

(6 Marks)

(c) In an order of assessment for the A.Y. 2015-16, the assessee noticed a mistake for

which application under section 154 was moved and the order was rectified.

Subsequently, the assessee moved further application for rectification under section

154 which was rejected by the Assessing Officer on the ground that the order once

rectified cannot be rectified again. Is the contention of the Assessing Officer correct?

(4 Marks)

6. (a) State whether the information regarding possession of unexplained assets and

income received from the Central Bureau of Investigation, a Government agency, can

constitute “information” for action under section 132. Discuss. (4 Marks)

(b) Swift Inc., a france company has a subsidiary, ABC Ltd. in India. Swift Inc. sells

Laptops to ABC Ltd. for resale in India. Swift Inc. also sells Laptops to XYZ Ltd.,

another Laptop dealer for resale in India. It sells 30,000 Laptops to ABC Ltd. at

Rs. 21,000 per unit. The price fixed for XYZ Ltd. is Rs. 18,000 per unit. The warranty

in case of sale of Laptops by ABC Ltd. is handled by ABC Ltd. However, for sale of

Laptops by XYZ Ltd., Swift Inc. is responsible for the warranty for 4 months. Both

Swift Inc. and ABC Ltd. offer extended warranty at a standard rate of

Rs. 2,400 per annum. On these facts, how is the assessment of ABC Ltd. going to be

affected? (4 Marks)

(c) A foreign company entered into contracts with several Indian companies for

installation of mobile telephone system and made an application to the Authority for

Advance Rulings for advance ruling on the rate of withholding tax on receipts from

Indian companies. One of the Indian companies had also made an application to the

Assessing Officer for determination of the rate at which tax is deductible on payment

to the said foreign company. The Authority for Advance Rulings rejected the

application of the foreign company on the ground that the question raised in the

application is already pending before an income tax authority. Is the rejection of the

application of the foreign company justified in law? (4 Marks)

(d) The return for A.Y.2015-16 was filed on 28.7.2015. Later on, the assessee, noticed

certain omissions and therefore filed a revised return on 5.4.2017. The Assessing

Officer ignoring the revised return so filed framed the order under section 143(3) on

7.5.2017. Is the action of Assessing Officer correct? (4 Marks)

Page 133: Test Series: August, 2017 MOCK TEST PAPER 1 FINAL COURSE ... · The company incurred losses of Rs. 40,000 and Rs. 60,000 in 2015-2016 and 2016-2017 after charging depreciation of

6

7. (a) (i) Safe Airways Ltd. sold tickets to the travel agents in India at a minimum fixed

commercial price. The agents were permitted to sell the tickets at a higher price

but not exceeding the maximum published price. Commission at the rate of 9%

of minimum fixed commercial price was deducted under section 194H by the

company. The Assessing Officer contended that the liability for tax deduction at

source is attracted on the difference between the minimum fixed commercial

price and the maximum published price by treating it as "additional special

commission" in the hands of the agents.

Is the contention of Assessing Officer tenable in law?

(ii) Subhash, an individual, had let out his building on a monthly rent of Rs. 25,000.

The tenant deducted tax under section 194-I from the rent paid to Subhash, but

did not remit such tax to the credit of the Central Government. Subhash filed his

return of income for the assessment year 2017-18 including therein the rental

income from the said building and paid the balance tax on his total income after

taking credit for tax deducted at source by the tenant. The Assessing Officer has

called upon Subhash to pay the tax to the extent of tax deducted at source. Is

the Assessing Officer justified in doing so? (6 Marks)

(b) Discuss the powers of the Settlement Commission to amend its orders . (4 Marks)

(c) Discuss the correctness or otherwise of the following statements with reference to the

provisions of the Income-tax Act, 1961:

(i) The Commissioner (Appeals) cannot admit an appeal filed beyond 30 days from

the date of receipt of order by an assessee.

(ii) The Appellate Tribunal is empowered to grant indefinite stay for the demand

disputed in appeals before it. (6 Marks)

Page 134: Test Series: August, 2017 MOCK TEST PAPER 1 FINAL COURSE ... · The company incurred losses of Rs. 40,000 and Rs. 60,000 in 2015-2016 and 2016-2017 after charging depreciation of

1

Test Series: August, 2017

MOCK TEST PAPER–1

FINAL – GROUP – II

PAPER – 7: DIRECT TAX LAWS

SUGGESTED ANSWERS/HINTS

1. (a) As per section 80AC, while computing the total income of an assessee of a previous

year (P.Y.2016-17, in this case) relevant to any assessment year (A.Y.2017-18, in

this case), any deduction is admissible, inter alia, under section 80-IA, such

deduction shall not be allowed unless it furnishes a return of income for such

assessment year on or before the ‘due date’ specified in section 139(1).

Since the firm has not entered into any international transaction or specified domestic

transaction and the turnover of the partnership firm has exceeded Rs. 100 lacs in the

previous year 2016-17, it would be subject to audit under section 44AB, in which case

the ‘due date’ of filing its return of income for A.Y.2017 -18 would be 30th September,

2017 as per section 139(1).

Computation of total income and tax liability of M/s. ABC Corporation for

A.Y.2017-18

I. Where the firm files its return of income on 30 th September 2017:

Particulars Rs. in lacs

Gross Total Income 300.00

Less: Deduction under section 80-IA 200.00

Total Income 100.00

Tax liability@ 30% 30.00

Add: Education cess@2% and secondary and higher education cess@1%

0.90

Regular income-tax payable 30.90

Computation of Alternate Minimum Tax payable [Section 115JC]

Total Income 100.00

Add: Deduction under section 80-IA 200.00

Adjusted Total Income 300.00

Page 135: Test Series: August, 2017 MOCK TEST PAPER 1 FINAL COURSE ... · The company incurred losses of Rs. 40,000 and Rs. 60,000 in 2015-2016 and 2016-2017 after charging depreciation of

2

Alternate Minimum Tax (AMT) @ 18.5% on Rs. 300 lacs 55.50

Add: Surcharge@12% (Since adjusted total income > Rs. 1 crore)

6.66

62.16

Add: Education cess@2% and SHEC@1% 1.86

Total tax payable (AMT) 64.02

Since the regular income-tax payable by the firm is less than the alternate

minimum tax payable, the adjusted total income shall be deemed to be the total

income of the firm for P.Y.2016-17 and it shall be liable to pay income-tax on

such total [email protected]% [Section 115JC(1)]. Therefore, the tax payable for

the A.Y.2017-18 would be Rs. 64.02 lacs.

Tax credit for Alternate Minimum Tax [Section 115JD] Rs. in lacs

Total tax payable for A.Y.2017-18 (Alternate Minimum Tax) 64.02

Less: Regular income-tax payable 30.90

To be carried forward for set-off against regular income-tax payable (upto a maximum of ten assessment years).

33.12

II. Where the firm files its return of income on 10th November 2017:

Where the firm files its return on 10-11-2017, it would be a belated return under

section 139(4). Consequently, as per section 80AC, deduction under section

80-IA would not be available. In such circumstances, the gross total income of

Rs. 300 lacs would be the total income of the firm.

Particulars Rs. in lacs

Income-tax@30% of Rs. 300 lacs 90.000

Add: Surcharge@12% (since total income exceeds Rs. 100 lacs)

10.800

Income-tax (plus surcharge) 100.800

Add: Education cess@2% and SHEC@1% 3.024

Total tax liability 103.824

Practical solution regarding obtaining clarifications

The practical solution regarding obtaining clarifications would be to file the return

of income under section 139(1) on or before the ‘due date’ 30.9.2017 and claim

deduction under section 80-IA. In such a case, the firm can claim deduction of

Rs. 200 lacs under section 80-IA. Thereafter, consequent to the clarifications

obtained, if any change is required, it can file a revised return under section

Page 136: Test Series: August, 2017 MOCK TEST PAPER 1 FINAL COURSE ... · The company incurred losses of Rs. 40,000 and Rs. 60,000 in 2015-2016 and 2016-2017 after charging depreciation of

3

139(5) within 31.3.2019 (i.e., within one year from the end of A.Y.2017-18) which

would replace the original return filed under section 139(1).

If the firm files the return of income under section 139(1) on or before 30.9.2017,

its tax liability would stand reduced to Rs. 64.02 lacs, as against Rs. 103.824

lacs to be paid if return is furnished after due date. Further, it would also be

eligible for tax credit for alternate minimum tax under section 115JD to the extent

of Rs. 33.12 lacs. Therefore, the firm is advised to file its return of income on or

before 30.9.2017.

(b) Tax consequences in the hands of the MNO trust and its unit holders

Particulars Tax treatment in the hands of

MNO Trust (REIT) Unit Holders

(i) (ii) (iii) (iv)

(1) Interest income of Rs. 4 crore from High Ltd.

There would be no tax liability in the hands of business trust due to pass-through status enjoyed by it under sub-clause (a) of section 10(23FC) in respect of interest income from High Ltd., being the special purpose vehicle. Therefore, High Ltd. is not required to deduct tax at source on interest payment to the business trust.

However, the business trust has to deduct tax at source under section 194LBA –

@ 10%, on interest component of income distributed to resident unit holders; and

@ 5%, on interest component of income distributed to non-corporate non-resident unit holders and foreign companies.

Interest component of income distributed to unit holders is taxable in the hands of the unit holders – @ 5%, in case of unit holders, being non-corporate non-residents or foreign companies; and at normal rates of tax, in case of resident unit holders.

The interest component of income received from the business trust in the hands of each unit-holder would be determined in the proportion of 4/11.1, by virtue of section 115UA(1).

Page 137: Test Series: August, 2017 MOCK TEST PAPER 1 FINAL COURSE ... · The company incurred losses of Rs. 40,000 and Rs. 60,000 in 2015-2016 and 2016-2017 after charging depreciation of

4

(2) Dividend income of Rs. 2 crore from High Ltd.

The dividend distributed by the SPV to the business trust is exempt by virtue of section 115-O(7), since the SPV is a specified domestic company in which the business trust has become the holder of whole of the nominal value of equity share capital of the company.

Further, there would be no tax liability in the hands of the business trust, due to specific exemption provided under sub-clause (b) of section 10(23FC).

Any distributed income referred to in section 115UA, to the extent it does not comprise of interest [referred to in sub-clause (a) of section 10(23FC)] and rental income from real estate assets owned directly by the business trust [referred to in section 10(23FCA)] received by unit holders, is exempt in their hands under section 10(23FD). Therefore, by virtue of section 10(23FD), there would be no tax liability on the dividend component [referred to in sub-clause (b) of section 10(23FC)] of income distributed to unit holders in their hands.

(3) Short-term capital gains of Rs. 1.5 crore on sale of listed shares of High Ltd.

As per section 115UA(2), the business trust is liable to pay tax@15% under section 111A in respect of short-term capital gains on sale of listed shares of special purpose vehicle.

Any distributed income referred to in section 115UA, to the extent it does not comprise of interest referred to in section 10(23FC)(a) and rental income referred to in section 10(23FCA), received by unit holders is exempt in their hands under section 10(23FD). Hence, the

Page 138: Test Series: August, 2017 MOCK TEST PAPER 1 FINAL COURSE ... · The company incurred losses of Rs. 40,000 and Rs. 60,000 in 2015-2016 and 2016-2017 after charging depreciation of

5

impugned income will not have any tax effect in the hands of the unit holders.

(4) Interest of Rs. 10 lakh received in respect of investment in unlisted debentures of real estate companies

Such interest is [email protected]%, being the maximum marginal rate, in the hands of the business trust, as per section 115UA(2).

No tax liability on interest component of income distributed to unit holders by virtue of section 10(23FD).

(5) Rental income of Rs. 3.50 crores of REIT from directly owned real estate assets

Any income of a business trust, being a REIT, by way of renting or leasing or letting out any real estate asset owned directly by such business trust is exempt in the hands of the trust as per section 10(23FCA).

Where the income by way of rent is credited or paid to a business trust, being a REIT, in respect of any real estate asset held directly by such REIT, no tax is deductible at source under section 194-I.

The business trust has to deduct tax at source@10% under section 194LBA in case of distribution to a resident unit holder and at rates in force in case of distribution to a non-resident unit holder.

The distributed income or any part thereof, received by a unit holder from the REIT, which is in the nature of income by way of renting or leasing or letting out any real estate asset owned directly by such REIT is deemed income of the unit holder as per section 115UA(3).

The rental income component received from the business trust in the hands of each unit-holder would be determined in the proportion of 3.5/11.1, by virtue of section 115UA(1).

2. Computation of Total Income of M/s. Popular Tele films for the A.Y.2017-18

Particulars Rs. Rs.

Profits and Gains from Business or Profession

Net Profit as per Profit & Loss A/c 10,00,000

Page 139: Test Series: August, 2017 MOCK TEST PAPER 1 FINAL COURSE ... · The company incurred losses of Rs. 40,000 and Rs. 60,000 in 2015-2016 and 2016-2017 after charging depreciation of

6

Add: Expenses disallowed or considered separately:

Interest to partners in excess of 12% (Note 1) 3,00,000

Disallowance under section 40A(3A) for aggregate cash payment exceeding Rs. 20,000 in a single day (Note 5)

33,000

Provision for gratuity (Note 8) 4,50,000

Partners’ Remuneration 30,00,000

Royalty paid to Partner Ram (Note 4) 5,00,000 42,83,000

52,83,000

Less: Interest on income-tax refund (Note 9) 1,20,000

Book Profit 51,63,000

Less: Partners’ remuneration allowable under section 40(b)(v)

(i) As per limit prescribed in section 40(b)

On first Rs. 3,00,000 90% 2,70,000

On the balance Rs.48,63,000 60% 29,17,800

31,87,800

(ii) Remuneration actually paid or payable

(Rs. 30,00,000 to 3 partners) + (Royalty Rs. 5 Lacs to Ram)

35,00,000

(i) or (ii) whichever is less, is deductible 31,87,800

19,75,200

Capital Gain

Short-term capital gain on transfer of land (Note 10) 6,00,000

Income from other sources

Interest on income-tax refund 1,20,000

Gross Total Income 26,95,200

Deductions under Chapter VI-A Nil

Total Income 26,95,200

Notes:

1. As per section 40(b), simple interest at 12% p.a. to partners relating to the period

after the date of partnership deed is allowable. Excess interest @ 3% paid from 1 st

June, 2016 to 31st March, 2017 is to be disallowed. Excess interest of 3% being

Rs.15,00,000 x 3/15 = Rs.3,00,000.

2. Even though Mohan is a partner in a representative capacity, he is still a partner.

Therefore, remuneration to Mohan should also be subject to the limits prescribed in

Page 140: Test Series: August, 2017 MOCK TEST PAPER 1 FINAL COURSE ... · The company incurred losses of Rs. 40,000 and Rs. 60,000 in 2015-2016 and 2016-2017 after charging depreciation of

7

section 40(b). This view finds support from the decision of the Supreme Court in the

case of Rashik Lal & Co. vs CIT (1998) 229 ITR 458 (SC).

3. As per Explanation 1 to section 40(b), where an individual is a partner in a firm in

representative capacity, the provisions of section 40(b) shall not apply to any interest

payable by the firm to such individual in his personal capacity. Mohan represents his

HUF in the firm. However, Mohan gave the loan in his individual capacity. Hence,

assuming that the provisions of section 40A(2) do not get attracted in this case, such

interest shall be allowed as deduction in full even though the interest rate is more

than 12% p.a.

4. It may be noted that the limits specified under section 40(b)(v) are applicable in case

of payment of salary, bonus, commission, or remuneration, by whatever name called,

to a working partner. From a plain reading of the section, it is clear that any

remuneration, by whatever name called, paid to a working partner, is subject to th e

limits laid down in section 40(b)(v). Therefore, the royalty of Rs. 5 Lacs paid to partner

Ram would also be subject to the limits laid down in section 40(b)(v). Hence, the same

has to be added back for computing book profits.

5. Section 40A(3A) provides for disallowance of any expenditure for which the aggregate

payment made is otherwise than by an account payee cheque or account payee bank

draft in a single day to a person exceed a sum of Rs. 20,000. Hence, the payments

of Rs. 17,000 and Rs. 16,000 in cash on 1.3.2017 to Asif, a hairdresser, shall be

disallowed, since the aggregate payment of Rs. 33,000 exceeds the limit of

Rs. 20,000.

6. The payment of bill of the assistant cameraman of Rs. 16,000 and Rs. 18,000

respectively on 1st February and 2nd February is not liable for disallowance under

section 40A(3) since the aggregate payment in cash on a single day has not exceeded

Rs. 20,000.

7. As per section 40(a)(i), any sum payable to a non-resident shall not be allowed as

deduction, if tax has not been deducted at source or after deduction, has not been

paid on or before the due date specified under section 139(1). Tax deducted from the

amount of remuneration credited to payee's account on 31st March 2017 has to be

deposited latest by 31st July 2017/ 30th September, 2017 (as the case may be). The

firm has paid the tax on 5th July, 2017 and hence, the remuneration shall be allowed.

Since the same is already debited to profit and loss account, no further adjustment is

made.

8. As per section 40A(7), any provision made for payment of gratuity to employees on

their retirement or on termination of employment for any reason is disallowed.

However, gratuity of Rs. 2.50 lacs paid to retired employees is allowable as deduction.

Hence, the balance provision of Rs. 4.50 lacs (i.e., Rs. 7 lacs – Rs. 2.50 lacs) is to

be disallowed.

Page 141: Test Series: August, 2017 MOCK TEST PAPER 1 FINAL COURSE ... · The company incurred losses of Rs. 40,000 and Rs. 60,000 in 2015-2016 and 2016-2017 after charging depreciation of

8

9. Interest on income-tax refund is assessable under the head "Income from other

sources".

10. Distribution of a capital asset by a firm to its partner on dissolution or otherwise

attracts capital gains tax liability as per the provisions of section 45(4) and the fair

market value of the asset on the date of transfer is deemed to be the full value of

consideration received or accruing as a result of the transfer. The words "or

otherwise" includes within its scope, cases of distribution of capital assets on

retirement of a partner also. [CIT vs. A. N .Naik Associates (2004) 265 ITR 346

(Bom.)]. Therefore, distribution of a plot of land on retirement of a partner would

attract section 45(4). Rs. 16 lacs, being the fair market value of the plot on the date

of transfer, is deemed to be the full value of consideration. Therefore, the capital gain

would be Rs. 6 lacs (i.e., Rs. 16 lacs – Rs. 10 lacs).

3. (a) Under section 46(1), where the assets of a company are distributed to its

shareholders on its liquidation, such distribution shall not be regarded as transfer in

the hands of the company for the purpose of section 45.

However, under section 46(2), where the shareholder, on liquidation o f a company,

receives any money or other assets from the company, he shall be chargeable to

income-tax under the head “capital gains”, in respect of the money so received or the

market value of the other assets on the date of distribution as reduced by the amount

of dividend deemed under section 2(22)(c) and the sum so arrived at shall be deemed

to be the full value of the consideration for the purposes of section 48.

As per section 2(22)(c), dividend includes any distribution made to the shareholders

of a company on its liquidation, to the extent to which the distribution is attributable

to the accumulated profits of the company immediately before its liquidation, whether

capitalized or not.

In this case, the accumulated profits immediately before liquidation is Rs. 3,00,000.

The share of each shareholder is Rs. 60,000 (being one-fifth of Rs. 3,00,000). An

amount of Rs. 60,000 is the deemed dividend under section 2(22)(c). The same is

exempt under section 10(34) in the hands of the shareholder, since the company is

liable to dividend distribution tax in respect of the same.

Therefore, Rs. 2,00,000 [i.e. Rs. 2,60,000 minus Rs. 60,000, being the deemed

dividend under section 2(22)(c)] is the full value of consideration in the hands of each

shareholder as per section 46(2). Against this, the investment of Rs. 1,00,000 by each

shareholder is to be deducted to arrive at the capital gains of Rs. 1,00,000 of each

shareholder. The benefit of indexation is available to the shareholders (since the

shares are held for more than 24 months and hence long-term capital asset), but

could not be computed in the absence of required information. Since the equity shares

are not listed, it would not be liable for securities transaction tax and hence, the capital

gain (long term) is not exempt under section 10(38). The benefit of concessional rate

Page 142: Test Series: August, 2017 MOCK TEST PAPER 1 FINAL COURSE ... · The company incurred losses of Rs. 40,000 and Rs. 60,000 in 2015-2016 and 2016-2017 after charging depreciation of

9

of tax @10% without indexation would also not be available. Hence, such long term

capital gain would be taxable @ 20% with indexation benefit.

Exemption under section 54EC is available only where there is an actual transfer of

capital assets and not in the case of deemed capital gain as per the decision rendered

in the case of CIT v. Ruby Trading Co (P) Ltd (2003) 259 ITR 54 (Raj). Therefore,

exemption under section 54EC will not be available in this case since it is deemed

transfer and not actual transfer.

Note: However, in case a view is taken that on liquidation of a company, the

shareholders right in the said company is relinquished or extinguished and the

consideration received by the shareholder is for such relinquishment or

extinguishment of rights of the shareholder, then, this transaction shall be transfer as

per section 2(47) and the exemption under section 54EC in respect of the capital gain

arising from the said transaction shall be available.

(b) The statement is correct

(1) Exemption from STT and CTT: With effect from 1.6.2016, securities transaction

tax is not leviable in respect of taxable securities transactions entered into by

any person on a recognised stock exchange located in an International Financial

Services Centre (IFSC) where the consideration for such transaction is paid or

payable in foreign currency.

Likewise, commodities transaction tax is not leviable in respect of taxable commodities transactions entered into by any person on a recognised association located in unit of IFSC where the consideration for such transaction is paid or payable in foreign currency.

(2) Exemption of LTCGs on sale of securities, even if STT is not paid : Long-

term capital gains in respect of income arising from transaction undertaken in

foreign currency on a recognised stock exchange located in an International

Financial Services Centre would be exempt under section 10(38) even though

securities transaction tax is not paid in respect of such transaction.

(3) Concessional rate of tax on short-term capital gains, even if STT is not

paid: Short term capital gains arising from transaction undertaken in foreign

currency on a recognised stock exchange located in an International Financial

Services Centre would be taxable at a concessional rate of 15% under section

111A even though securities transaction tax is not paid in respect of such

transaction.

(4) Concessional rate of MAT: In case of a company, being a unit located in

International Financial Services Centre and deriving its income solely in

convertible foreign exchange, the minimum alternate tax under section 115JB

shall be chargeable at the rate of 9% instead of 18.5%.

Page 143: Test Series: August, 2017 MOCK TEST PAPER 1 FINAL COURSE ... · The company incurred losses of Rs. 40,000 and Rs. 60,000 in 2015-2016 and 2016-2017 after charging depreciation of

10

(5) Exemption from tax on distributed profits: In case of a company being a unit

located in International Financial Services Centre, deriving income solely in

convertible foreign exchange, there would be no tax on any amount declared,

distributed or paid by such company, by way of dividends (whether interim or

otherwise) on or after 1st April, 2017 out of its current income, either in the hands

of the company or the person receiving such dividend.

(c) Under section 2(22)(e), any payment by a closely-held company by way of loan or

advance to its shareholder, being a person who is the beneficial owner of shares,

holding not less than 10% of the voting power, is deemed as dividend to the extent to

which the company possesses accumulated profits.

Therefore, in order to attract the deeming provisions under section 2(22)(e), the

recipient of loan should be a registered shareholder as well as the beneficial owner

of shares.

Accordingly, in this case, Rs. 50,000 (i.e., loan to the extent of accumulated profits of

MNO Ltd.) would be deemed as dividend in the hands of Mrs. Kashyap, who holds

12% equity shares in MNO Ltd., under section 2(22)(e).

Thereafter, the clubbing provisions under section 64(1)(iv) would be attracted, as per

which, income as arises, directly or indirectly, from asset transferred to spouse,

otherwise than for adequate consideration, would be included in the hands of the

transferor.

If the assets so transferred are shares in a company, the loan taken from the company

is deemed as dividend income of the shareholder under section 2(22)(e) to the extent

to which the company possesses accumulated profits. Thus, on account of this

deeming provision, such loan is treated as income arising from the shares. It was so

held by the Madras High Court in CIT v. Vimalan (A.) (1975) 98 ITR 529.

Accordingly, as per section 64(1)(iv), such income arising in the hands of the

shareholder, Mrs. Kashyap, by virtue of section 2(22)(e) (i.e., deemed dividend of Rs.

50,000) would be included in the total income of Mr. Kashyap, who had transferred

the said shares to Mrs. Kashyap without consideration.

4. (a) Section 192(1) requires any person responsible for paying any income chargeable

under the head “Salaries” to deduct tax at source. If an employee receives income

chargeable under a head other than “Salaries”, section 192 does not get attracted at

all.

The issue under consideration is whether “tips” received by the hotel -company from

its customers and distributed to the employees fell within the meaning of “Salaries” to

attract tax deduction at source under section 192.

This issue came up before the Supreme Court in ITC Ltd. v. CIT (TDS) (2016) 384

ITR 14 wherein it was observed in respect of tips collected by the company from the

customers and distributed to the employees, the person responsible for paying the

Page 144: Test Series: August, 2017 MOCK TEST PAPER 1 FINAL COURSE ... · The company incurred losses of Rs. 40,000 and Rs. 60,000 in 2015-2016 and 2016-2017 after charging depreciation of

11

employee was not the employer at all, but a third person, namely the customer. As

income from tips would be chargeable in the hands of the employees as “Income from

Other Sources”, on account of such tips being received from customers and not from

the employer, section 192 would not get attracted at all.

The Supreme Court further observed that there was no vested right in the employee

to claim any amount of tip from his employer. Tips are purely voluntary amounts that

may or may not be paid by customers for services rendered to them, and hence,

would not fall within the meaning and scope of section 15. Further, the amount of

tips collected from the customers by the employer and paid to the employees has no

reference to the contract of employment at all. Tips were received by the employer

in a fiduciary capacity as trustee for payments that were received from customers

which they disbursed to their employees for service rendered to the customer. There

was, therefore, no reference to the contract of employment when these amounts were

paid by the employer to the employee. Due to this reason the tips received by the

employees could not be regarded as profits in lieu of salary in terms of section 17 (3).

The payments of collected tips included and paid by way of a credit card by a

customer, would not be payments made “by or on behalf of” an employer. The contract

of employment not being the proximate cause for the receipt of tips by the employee

from a customer, such payments would be outside the scope of sections 15 and 17

of the Act.

Applying the rationale of the above Supreme Court ruling to the case on hand, the

action of the Assessing Officer in treating the Foodie Ltd as an assessee-in-default

for non-deduction of tax at source on the amount of tips collected by it from the

customers and distributed to its employees, is not correct.

(b) The long-term capital gain arising on sale of residential house would be exempt under

section 54 if it is utilized, inter alia, for purchase of one residential house situated in

India within one year before or two years after the date of transfer. Release by the

other co-owners of their share in co-owned property in favour of Vikram would amount

to “purchase” by Vikram for the purpose of claiming exemption under section 54 [CIT

v. T.N. Arvinda Reddy (1979) 120 ITR 46 (SC)]. Since such purchase is within the

stipulated time of two years from the date of transfer of asset, Vikram is eligible for

exemption under section 54. As Vikram has utilised the entire long-term capital gain

arising out of the sale of the residential house for payment of consideration to the

other co-owners who have released their share in his favour, he can claim full

exemption under section 54.

There is no requirement in section 54 that the new house should be used by the

assessee for his own residence. The condition stipulated is that the new house should

be utilised for residential purposes and its income is chargeable under the head

“Income from house property”. This requirement would be satisfied even when the

new house is let out for residential purposes.

Page 145: Test Series: August, 2017 MOCK TEST PAPER 1 FINAL COURSE ... · The company incurred losses of Rs. 40,000 and Rs. 60,000 in 2015-2016 and 2016-2017 after charging depreciation of

12

(c) The issue under consideration is whether winnings of prize money on unsold lottery

tickets held by the distributor of lottery tickets can be subject to tax at the rate of 30%

prescribed under section 115BB.

In CIT v. Manjoo and Co. (2011) 335 ITR 527, the Kerala High Court observed that

the receipt of winnings from lottery by the distributor was not on account of any

physical or intellectual effort made by him and therefore cannot be said to be "income

earned" by him in business.

The unsold lottery tickets cease to be stock-in-trade of the distributor because, after

the draw, those tickets are unsalable and have no value except waste paper value

and the distributor will get nothing on account of the tickets except any prize winning

ticket if held by him, which, if produced will entitle him for the prize money.

Hence, the receipt of the prize money is not in his capacity as a lottery distributor but

as a holder of the lottery ticket which won the prize. The Lottery Department also

does not treat it as business income received by the distributor but instead treats it

as prize money paid on which tax is deducted at source.

Further, winnings from lotteries are assessable under the special provisions of section

115BB, irrespective of the head under which such income falls.

Therefore, the Kerala High Court held that the rate of 30% prescribed under section

115BB would be applicable on prize money winnings received by a distributor on

unsold lottery tickets held by him.

Applying the rationale of the Kerala High Court ruling to the case on hand, the

Assessing Officer’s intention to tax the prize money received by the distributor on

unsold lottery tickets held by him at the rate prescribed under section 115BB is

justified.

(d) As per ICDS V on Tangible Fixed Assets, machinery spares shall be charged to the

revenue as and when consumed. When such spares can be used only in connection

with an item of tangible fixed asset and their use is expected to be irregular, they shall

be capitalised. Where the spares are capitalised as per the above requirement, the

issue as to provision of depreciation arises – whether depreciation can be provided

where such spares are kept ready for use or is it necessary that they are actually put

to use. This issue was dealt with by the Delhi High Court in CIT v. Insilco Ltd (2010)

320ITR 322. The Court observed that the expression “used for the purposes of

business” appearing in section 32 also takes into account emergency spares, which,

even though ready for use, yet are not consumed or used during the relevant period.

This is because these spares are specific to a fixed asset, namely plant and

machinery, and form an integral part of the fixed asset. These spares will, in all

probability, be useless once the asset is discarded and will also have to be disposed

of. In this sense, the concept of passive use which applies to standby machinery will

also apply to emergency spares. Therefore, once the spares are considered as

Page 146: Test Series: August, 2017 MOCK TEST PAPER 1 FINAL COURSE ... · The company incurred losses of Rs. 40,000 and Rs. 60,000 in 2015-2016 and 2016-2017 after charging depreciation of

13

emergency spares required for plant and machinery, the assessee would be entitled

to capitalize the entire cost of such spares and claim depreciation thereon.

Note – One of the conditions for claim of depreciation is that the asset must be “used

for the purpose of business or profession”. In the past, courts have held that, in certain

circumstances, an asset can be said to be in use even when it is “kept ready for use”.

For example, depreciation can be claimed by a transport company on spare engines

kept in store in case of need, though they have not actually been used by the

company. Hence, in such cases, the term “use” embraces both active use and passive

use for business purposes.

5. (a) Mr. Raj is eligible for deduction under section 80JJAA since he is subject to tax audit

under section 44AB for A.Y.2017-18, as his total turnover from business exceeds Rs.

1 crore and he has employed “additional employees” during the P.Y.2016-17. Since

this is the first year of his new business, emoluments paid or payable during this year

shall be deemed to be the additional employee cost.

I If Mr. Raj is engaged in the business of manufacture of computers

Deemed additional employee cost = Rs. 24,000 × 12 × 75 [See Working Note

below] = Rs. 2,16,00,000

Deduction under section 80JJAA = 30% of Rs. 2,16,00,000 = Rs. 64,80,000.

Working Note:

Number of additional employees

Particulars No. of workmen

Total number of employees employed during the year 350

Less: Casual employees employed on 1.8.2016 who do not participate in recognized provident fund

50

Regular employees employed on 1.5.2016, since their total monthly emoluments exceed Rs. 25,000

125

Regular employees employed on 1.9.2016 since they have been employed for less than 240 days in the P.Y.2016-17.

100

275

Number of “additional employees” 75

Note - Since casual employees do not participate in recognized provident fund,

they do not qualify as additional employees. In any case, their total monthly

emoluments exceed Rs.25,00, and hence do not qualify as additional

employees. Further, 125 regular employees employed on 1.5.2016 also do not

qualify as additional employees since their monthly emoluments exceed

Page 147: Test Series: August, 2017 MOCK TEST PAPER 1 FINAL COURSE ... · The company incurred losses of Rs. 40,000 and Rs. 60,000 in 2015-2016 and 2016-2017 after charging depreciation of

14

Rs. 25,000. Also, 100 regular employees employed on 1.9.2016 do not qualify

as additional employees for the P.Y.2016-17, since they are employed for less

than 240 days in that year.

Therefore, only 75 employees employed on 1.4.2016 qualify as additional

employees, and the total emoluments paid or payable to them during the

P.Y.2016-17 is deemed to be the additional employee cost.

II If Mr. Raj is engaged in the business of manufacture of apparel

In the case of an assessee engaged in the business of manufacture of apparel,

the requirement of minimum period of employment of 240 days in the previous

year to qualify as an additional employee for the purpose of deduction under

section 80JJAA has been relaxed due to the seasonal nature of business of

manufacture of apparel. The minimum period of employment required in case of

this industry, to qualify as an additional employee for the purpose of deduction

under section 80JJAA, is 150 days. Therefore, if Mr. Raj is engaged in the

business of manufacture of apparel, then, he would be entitled to deduction

under section 80JJAA in respect of employee cost of regular employees

employed on 1.9.2016, since they have been employed for more than 150 days

in the previous year 2016-17.

Deemed additional employee cost = Rs. 2,16,00,000 + Rs. 24,000 × 7 × 100 =

Rs. 3,84,00,000

Deduction under section 80JJAA = 30% of Rs. 3,84,00,000 = Rs. 1,15,20,000

(b) Computation of tax liability of Ms. Kavita for A.Y.2017-18

Particulars Rs. Rs.

Indian Income 42,00,000

Foreign Income 6,00,000

Gross Total Income 48,00,000

Less: Deduction under section 80C

PPF Contribution 1,50,000

Deduction under section 80D

Medical insurance premium of mother, being a resident senior citizen, restricted to

30,000

1,80,000

Total Income 46,20,000

Tax on total income 12,11,000

Add: Education cess @ 2% 24,220

Secondary and higher education cess @ 1% 12,110

12,47,330

Page 148: Test Series: August, 2017 MOCK TEST PAPER 1 FINAL COURSE ... · The company incurred losses of Rs. 40,000 and Rs. 60,000 in 2015-2016 and 2016-2017 after charging depreciation of

15

Average rate of tax in India [i.e., Rs. 12,47,330/ Rs. 46,20,000 x 100]

27%

Average rate of tax in foreign country

[i.e. Rs. 1,50,000/ Rs. 6,00,000 x 100]

25%

Doubly taxed income 6,00,000

Rebate under section 91 on Rs. 6,00,000 @25%

(lower of average Indian tax rate and foreign tax rate)

_1,50,000

Tax payable in India [Rs. 12,47,330 – Rs. 1,50,000] 10,97,330

Note: An assessee shall be allowed deduction under section 91 provided all the

following conditions are fulfilled:-

(a) The assessee is a resident in India during the relevant previous year.

(b) The income accrues or arises to him outside India during that previous year.

(c) Such income is not deemed to accrue or arise in India during the previous year.

(d) The income in question has been subjected to income-tax in the foreign country

in the hands of the assessee and the assessee has paid tax on such income in

the foreign country.

(e) There is no agreement under section 90 for the relief or avoidance of double

taxation between India and the other country where the income has accrued or

arisen.

In this case, Ms. Kavita is eligible for deduction under section 91 since all the above

conditions are fulfilled.

(c) As per section 154(1) read with section 154(2)(b), an income-tax authority may amend

any order passed by it under the provisions of the Income-tax Act, 1961 with a view

to rectify any mistake apparent from record, which has been bought to its notice, inter

alia, by the assessee. In the case of Hind Wire Industries Ltd. v. CIT (1995) 212 ITR

639, the Apex Court held that the order once amended can also be rectified

subsequently provided the mistake apparent from record is rectifiable under section

154. The Apex Court enlarged the scope of the words used in that section by stating

that it does not necessarily mean the original order. It could be any order including

the amended or rectified order. The contention of the Assessing Officer is therefore,

incorrect.

6. (a) As per section 132(1)(c), authorization for search and seizure can take place if the

authority, in consequence of information in his possession, has reason to believe that

any person is in possession of money, bullion, jewellery or other valuable articl e or

thing and these assets represent, either wholly or partly, income or property which

has not been, or would not be disclosed by such person for the purposes of this Act.

In the absence of such information, a search cannot be validly authorized.

Page 149: Test Series: August, 2017 MOCK TEST PAPER 1 FINAL COURSE ... · The company incurred losses of Rs. 40,000 and Rs. 60,000 in 2015-2016 and 2016-2017 after charging depreciation of

16

The Apex Court in UOl v Ajit Jain [2003] 260 ITR 80 has held that mere intimation by

the CBI that money was found in the possession of the assessee, which according to

the CBI was undisclosed, without something more, does not constitute “information”

within the meaning of section 132, on the basis of which a search warrant could be

issued. Consequently, the Supreme Court held that the search conducted on this

basis and the assessment made pursuant to such search was not valid.

(b) Swift Inc., the foreign company and ABC Ltd., the Indian company are associated

enterprises since Swift Inc. is the holding company of ABC Ltd. Swift Inc. sells

Laptops to ABC Ltd. for resale in India. Swift Inc. also sells identical Laptops to XYZ

Ltd., which is not an associated enterprise. The price charged by Swift Inc. for a

similar product transferred in comparable uncontrolled transaction is, therefore,

identifiable. Therefore, Comparable Uncontrolled Price (CUP) method for determining

arm’s length price can be applied.

While applying CUP method, the price in comparable uncontrolled transaction needs

to be adjusted to account for difference, if any, between the international transaction

(i.e. transaction between Swift Inc. and ABC Ltd.) and uncontrolled transaction (i.e.

transaction between Swift Inc. and XYZ Ltd.) and the price so adjusted shall be the

arm’s length price for the international transaction.

For sale of Laptops by XYZ Ltd., Swift Inc. is responsible for warranty for 4 months.

The price charged by Swift Inc. to XYZ Ltd. includes the charge for warranty for 4

months. Hence, arm's length price for Laptops being sold by Swift Inc. to ABC Ltd.

would be:

Particulars No. Rs.

Sale price charged by Swift Inc. to XYZ Ltd. 18,000

Less: Cost of warranty included in the price charged to XYZ Ltd. (Rs. 2,400 x 4 /12)

800

Arm's length price 17,200

Actual price paid by ABC Ltd. to Swift Inc. 21,000

Difference per unit 3,800

No. of units supplied by Swift Inc. to ABC Ltd. 30,000

Addition required to be made in the computation of total income of ABC Ltd. (3,800 × 30,000)

11,40,00,000

No deduction under chapter VI-A would be allowable in respect of the enhanced

income of Rs. 11.40 crores.

Note: It is assumed that ABC Ltd. has not entered into an advance pricing agreement

or opted to be subject to Safe Harbour Rules.

Page 150: Test Series: August, 2017 MOCK TEST PAPER 1 FINAL COURSE ... · The company incurred losses of Rs. 40,000 and Rs. 60,000 in 2015-2016 and 2016-2017 after charging depreciation of

17

(c) The matter relates to the admission or rejection of the application filed before the

Advance Ruling Authority on the grounds specified in clause (i) o f the first proviso to

sub-section (2) of section 245R of the Income-tax Act, 1961.

Clause (i) of the first proviso of section 245R(2) provides that the Authority shall not

allow the application where the question raised in the application is already pend ing

before any income-tax authority or Appellate Tribunal or any court. (except in case of

resident falling in sub-clause (iii) of clause (b) of section 245N)

In the above case, no application had been filed or contention urged by the applicant

(foreign company) before any income-tax authority/Appellate Tribunal/court, raising

the question raised in the application filed with AAR. One of the Indian companies,

however, had raised the question before the Assessing Officer, not on the applicant’s

behalf or with a view to benefit the applicant, but only to safeguard its own interest,

as it had a statutory duty to deduct the proper amount of tax from payments made to

a non-resident. Although the question raised pertains to one of the payments made

or to be made to the non-resident applicant, it was not one pending determination

before any income-tax authority in the applicant’s case.

Therefore, as held by the AAR in Ericsson Telephone Corporation India AB v. CIT

(1997) 224 ITR 203, the application filed by the Indian company before the Assessing

Officer cannot be treated to have been filed by the non-resident. Hence, it would not

be proper to reject the application of the foreign company relying on clause (i) of the

proviso to sub-section (2) of section 245R of the Income-tax Act, 1961.

(d) The original return was filed in time and the proceedings were already taken up for

assessment under section 143(2). As per the provisions of section 139(5) the

assessee can file the revised return only within a period of one year from the end of

the relevant assessment year to which the return relates or before completion of the

assessment, whichever is earlier. This period expires on 31.3.2017. Since the revised

return was filed by the assessee after this date, i.e., on 5.4.2017, the action of the

Assessing Officer in making the assessment in disregard of the revised return filed

on 5.4.2017 is correct.

7. (a) (i) As per the provisions of section 194H, a person is liable to deduct tax at source at

the time of credit or payment of commission to any resident, whichever is earlier.

In the present case, Safe Airways Ltd. correctly deducted tax at source under

section 194H from the commission@9% of the minimum fixed commercial price

paid to the travel agents, who were allowed to sell the air tickets at any price

higher than the minimum fixed commercial price subject to a maximum published

price. However, the Assessing Officer contented that the airline company was

required to deduct tax at source on the difference between the minimum fixed

commercial price and the maximum published price by treating it as “additional

special commission” in the hands of the agents.

Page 151: Test Series: August, 2017 MOCK TEST PAPER 1 FINAL COURSE ... · The company incurred losses of Rs. 40,000 and Rs. 60,000 in 2015-2016 and 2016-2017 after charging depreciation of

18

The facts of the case are similar to the case of CIT v. Qatar Airways (2011) 332

ITR 253, where the Bombay High Court held that the difference between the

maximum published price and the minimum fixed commercial price cannot be

taken as “additional special commission” in the hands of the agents. This is

because the maximum published price is the maximum price and the airline

company has granted permission to the agents to sell the tickets at a price lower

than the maximum published price. Further, the airline company would have no

information about the exact rate at which the tickets were ultimately sold by its

agents. In order to deduct tax at source on the difference between actual sale

price and minimum fixed commercial price, the exact income in the hands of the

agents must be ascertainable by the airline company. However, it is not so

ascertainable in this case, since the agents are given discretion to sell the tickets

at any rate between the minimum fixed commercial price and the maximum

published price. It would be impracticable and unreasonable to expect the

airline company to get a feedback from its numerous agents in respect of

the price at which the tickets were sold by them.

Applying the rationale of the above case to the case on hand, Safe Airways Ltd.

is not liable to deduct tax at source under section 194H on the difference

between the maximum published price and the minimum fixed commercial price,

even though the amount earned by the agent over and above the minimum fixed

commercial price is taxable as income in their hands.

Therefore, the contention of the Assessing Officer is not tenable in law.

(ii) Section 205 of the Income-tax Act, 1961 provides that where tax is deductible

at source under the provisions of Chapter XVII, the assessee shall not be called

upon to pay the tax himself to the extent to which tax has been deducted from

that income. Section 205, therefore, bars a direct demand being made on an

assessee to the extent of tax deducted from the income. The Income-tax

Department can recover the tax deducted at source from the tenant and not from

Subhash in view of the clear mandate in section 205.

The Karnataka High Court has, in Smt. Anusuya Alva v. DCIT (2005) 278 ITR

206, ruled that tax deducted at source by the tenant from the rent paid but not

remitted to the credit of the Central Government can be recovered only from the

tenant and not from the landlord. Therefore, in view of the clear mandate in

section 205, the Assessing Officer is not justified in law in calling upon Subhash

to pay the said tax.

(b) As per the section 245D(6B), the Settlement Commission may amend any order passed

by it under section 245D(4) to rectify a mistake apparent from the record, within six

months from the end of the month in which the order was passed. Where an application

for rectification is made by the Principal Commissioner or Commissioner or the

applicant within six months from the end of the month in which the order was passed,

Page 152: Test Series: August, 2017 MOCK TEST PAPER 1 FINAL COURSE ... · The company incurred losses of Rs. 40,000 and Rs. 60,000 in 2015-2016 and 2016-2017 after charging depreciation of

19

the Settlement Commission may amend its order within a period of six months from the

end of the month in which such application has been made.

However, an amendment which has the effect of modifying the liability of the applicant

shall not be made unless the Settlement Commission –

(1) has given notice to the applicant and the Principal Commissioner or

Commissioner of its intention to do so; and

(2) has allowed the applicant and the Principal Commissioner or Commissioner an

opportunity of being heard.

(c) (i) The statement is not correct.

As per section 249(3) of the Income-tax Act, 1961, the Commissioner (Appeals)

may admit an appeal after the expiry of the period of 30 days specified in section

249(2), if he is satisfied that the appellant had sufficient cause for not presenting

the appeal within the prescribed time.

(ii) The statement is not correct.

Section 254(2A) provides that the Appellate Tribunal, where it is possible, may

hear and decide an appeal within a period of four years from the end of the

financial year in which such appeal is filed.

The Appellate Tribunal may, on merit, pass an order of stay in any proceedings

relating to an appeal. However, such period of stay cannot exceed 180 days

from the date of such order. The Appellate Tribunal has to dispose of the appeal

within this period of stay.

Where the appeal has not been disposed of within this period and the delay in

disposing the appeal is not attributable to the assessee, the Appellate Tribunal

can further extend the period of stay originally allowed. However, the aggregate

of period originally allowed and the period so extended should not exceed 365

days even if the delay in disposing of the appeal is not attr ibutable to the

assessee. The Appellate Tribunal is required to dispose off the appeal within

this extended period. If the appeal is not disposed of within such period or

periods, the order of stay shall stand vacated after the expiry of such period or

periods.

Page 153: Test Series: August, 2017 MOCK TEST PAPER 1 FINAL COURSE ... · The company incurred losses of Rs. 40,000 and Rs. 60,000 in 2015-2016 and 2016-2017 after charging depreciation of

1

Test Series: August, 2017

MOCK TEST PAPER - 1

FINAL COURSE: GROUP – II

PAPER – 8: INDIRECT TAX LAWS

Question No. 1 is compulsory.

Attempt any five questions from the remaining six Questions.

(Wherever appropriate, suitable assumption should be made and

indicated in the answer by the candidate)

Time Allowed – 3 Hours Maximum Marks – 100

1. (a) From the following particulars for the preceding financial year, find out whether Hard

Manufacturing Co. is eligible for small scale exemption under Notification No. 8/2003-

C.E. dated 01-03-2003 for the current financial year:

Sr. No.

Particulars Rs.

(in lakhs)

1. Clearance of excisable goods exempted from payment of duty under a notification other than Notification No. 8/2003- C.E.

50

2. Clearance of account books bearing brand name of another person, falling under Heading 4820 of First Schedule to the Central Excise Tariff.

150

3. Clearance of excisable goods to United Nations exempted from payment of duty under Notification No. 108/95-C.E.

85

4. Total Exports [including export to Bhutan Rs. 50 lakh]. Other exports are to USA & UK

250

5. Clearance of goods (duty paid based on annual capacity of production under section 3A of the Central Excise Act, 1944)

190

Show your calculations, workings and explanations clearly, wherever required. Hard

Manufacturing Co. is not a jewellery manufacturer. (5 Marks)

(b) (i) Enumerate the activities or transactions which shall be treated neither as a

supply of goods nor a supply of services. (5 Marks)

(ii) What do you mean by composite supply & mixed supply? Briefly discuss the

manner in which the tax liability on composite supplies and mixed supplies is

determined? (5 Marks)

(c) XYZ Industries Ltd. imports an equipment by air. CIF price of the equipment is 6,000

US$, freight paid is 1,200 US$ and insurance cost is 1,800 US$. The banker realizes

the payment from importer at the exchange rate of Rs. 61 per US$. Central Board of

Excise and Customs notifies the exchange rate as Rs. 60 per US$ while rate of

Page 154: Test Series: August, 2017 MOCK TEST PAPER 1 FINAL COURSE ... · The company incurred losses of Rs. 40,000 and Rs. 60,000 in 2015-2016 and 2016-2017 after charging depreciation of

2

exchange notified by RBI is Rs. 62 per US$. XYZ Industries Ltd. expends Rs. 56,000

in India for certain development activities with respect to the imported equipment.

Basic customs duty is 10%, excise duty leviable on similar goods in India is 12.5%

and education cesses are 3% on duty. Additional duty of customs leviable under

section 3(5) of the Customs Tariff Act is exempt.

You are required to:

(i) compute the amount of total customs duty payable by XYZ Industries Ltd.

(ii) determine the amount of CENVAT credit available to XYZ Industries Ltd. How

can such CENVAT credit be utilised by XYZ Industries Ltd.? (5 Marks)

2. (a) Discuss the circumstances when an advance ruling will be void under the Central

Excise Act, 1944. (4 Marks)

(b) Service tax of Rs. 60,000 for the month of March, 2017 was paid on 24th April, 2017

by an HUF. The value of taxable services provided by it during the preceding financial

year was Rs. 12 lakh. Determine the amount of interest payable under section 75 of

the Finance Act, 1994.

Note: The service tax of Rs. 60,000 has not been collected from service receiver.

(8 Marks)

(c) State briefly the provisions of the Customs Act, 1962 relating to payment of interest

in case of provisional assessment. (4 Marks)

3. (a) Whether production of mustard oil and oil cake from mustard seeds amounts to

manufacture? You are required to examine the situation with the help of a decided

case law. (4 Marks)

(b) Protection Services, a proprietorship firm, was engaged in providing security services

to its customers. A show cause notice for demanding service tax was issued to the

firm and the demand was confirmed. The order was challenged in appeal before the

Commissioner, Central Excise (Appeals). The Commissioner (Appeals) denied the

raising of the following three additional legal grounds by Protection Services during

the course of personal hearing:

(i) Being a proprietary firm, it cannot be considered as security agency;

(ii) Issue involved being interpretation of law, penalty is not imposable; and

(iii) As per decisions, staff salary is to be excluded from the gross amount received

for security services.

Discuss with the help of a decided case law (if any), whether the Commissioner

(Appeals) was justified in not allowing the raising of the additional grounds.

(8 Marks)

Page 155: Test Series: August, 2017 MOCK TEST PAPER 1 FINAL COURSE ... · The company incurred losses of Rs. 40,000 and Rs. 60,000 in 2015-2016 and 2016-2017 after charging depreciation of

3

(c) PQR Company Ltd. had imported brine shrimp eggs. This was classified as “prawn

feed’ for customs duty purposes under chapter heading No. 2309 of the Customs

Tariff schedule which includes use as animal feed. The department’s view was that

this should be classified under chapter heading No. 051199 as non-edible animal

products unfit for human consumption.

M/s. PQR Company Ltd. provided literature to support their contention that the

imported material contained little organisms/embryos which later became larva that

were fed to the prawns. Therefore, according to the importers, the nature and

character of the product was not changed or altered by nurturing or incubation.

Hence, the classification should be as prawn feed under chapter heading No. 2309.

Decide with the help of case law, if any, whether the contention of the assessee,

M/s. PQR Co. is correct in law. (4 Marks)

4. (a) Chill Beverages Ltd. has removed the aerated water bottles without declaring the

retail sale price under section 4A of the Central Excise Act, 1944. Discuss briefly how

the retail sale price of these goods shall be ascertained. (4 Marks)

(b) ‘Green Agro Unit’ furnishes the following details with respect to the services provided

by them in the month of June, 2016:

Sl. No. Particulars Amount (Rs.)

(i) Supply of farm labour 78,000

(ii) Warehousing of biscuits 1,65,000

(iii) Sale of rice on commission basis 68,000

(iv) Training of farmers on use of new pesticides and fertilizers developed through scientific research

1,30,000

(v) Renting of vacant land to a stud farm 1,31,500

(vi) Testing undertaken for soil of a farm land 1,10,000

(vii) Leasing of vacant land to a dairy farm 45,500

Compute the service tax liability of ‘Green Agro Unit’ for the month of June, 2016.

Service tax @ 15% [including Swachh Bharat Cess (SBC) & Krishi Kalyan Cess

(KKC)] has been charged separately, wherever applicable. ‘Green Agro Unit’ has paid

service tax of Rs. 6,18,000 during the preceding financial year. (8 Marks)

(c) With reference to section 9A(1A) of the Customs Tariff Act, 1975, mention the ways

that constitute circumvention of antidumping duty imposed on an article which may

warrant action by the Central Government. (4 Marks)

Page 156: Test Series: August, 2017 MOCK TEST PAPER 1 FINAL COURSE ... · The company incurred losses of Rs. 40,000 and Rs. 60,000 in 2015-2016 and 2016-2017 after charging depreciation of

4

5. (a) With reference to the provisions of rule 16 of Central Excise Rules, 2002, explain the

procedure for availment of CENVAT credit when:

(i) the duty paid goods returned to the factory are put through a process not

amounting to manufacture;

(ii) the duty paid goods returned to the factory are put through a process amounting

to manufacture. (4 Marks)

(b) With reference to Place of Provision of Service Rules (PoPS), 2012 answer the

following question:

(i) A movie-on-demand is provided as on-board entertainment during the

Bangalore-Delhi leg of a Singapore-Bangalore-Delhi flight against a charge of

Rs. 800 per passenger in addition to the fare of Rs. 30,000 per passenger. What

will be the place of provision of service in this case? Will your answer change, if

the above service is provided on a Delhi-Bangalore-Singapore-Malaysia flight

during the Singapore-Malaysia leg?

(ii) Mr. Diljeet has a permanent residence at Punjab. He has a savings bank

account with Punjab Branch of Safe and Sound Bank. On April 1, 2014,

Mr. Diljeet opened a safe deposit locker with the Punjab Branch of Safe and

Sound Bank. Mr. Diljeet went to USA for official work in December, 2014 and

has been residing there since then. Mr. Diljeet contends that since he is a non-

resident during the year 2015-16 in terms of the Income-tax Act, service tax

cannot be levied on the locker fee charged by Safe and Sound Bank for the year

2015-16.

Examine the correctness of the contention of Mr. Diljeet. (8 Marks)

(c) State the circumstances under which a revision petition can be filed before the Central

Government under the Customs Act. (4 Marks)

6. (a) Indicate the central excise authorities who can accept bonds executed for various

purposes and obligations of exporter in respect of bond acceptance while filling in

ARE-1 form. (4 Marks)

Or

Explain briefly the provisions relating to Special Audit in certain cases under section

14A of the Central Excise Act, 1944. (4 Marks)

(b) Galaxy Ltd. imports business support services from Green Inc of USA on 13.01.2016.

The relevant invoice for $ 4,20,000 is raised by Green Inc. on 18.01.2016. Galaxy

Ltd. makes the payment against the said invoice as follows:

Case I 22.02.2016

Case II 27.05.2016

Determine point of taxation in each of the aforesaid cases. (8 Marks)

Page 157: Test Series: August, 2017 MOCK TEST PAPER 1 FINAL COURSE ... · The company incurred losses of Rs. 40,000 and Rs. 60,000 in 2015-2016 and 2016-2017 after charging depreciation of

5

(c) Mr. Vipin and his wife (non-tourist Indian passengers) are returning from Dubai to

India after staying there for a period of two years. They wish to bring gold jewellery

purchased from Dubai. Please enumerate provisions of customs laws for jewellery

allowance in their case. (4 Marks)

7. (a) Can remission of duty be granted on duty paid goods which are destroyed by fire in

transit after being cleared from the factory? Explain. (4 Marks)

(b) Tingtong Travels Pvt. Ltd., located in Gujarat, is engaged in providing services of

renting of motorcab and discharges its service tax liability by availing abatement

granted under Notification No. 26/2012 ST dated 20.06.2012. Amount charged for

the services rendered by the company during the month of October, 2016 is

Rs. 6,00,000. The invoices for such services have been issued by Tingtong Travels

Pvt. Ltd. in the month of October, 2016.

The company has sub-contracted a part of its services to Ramlal Cabs Pvt. Ltd., which

is also engaged in providing services of renting of motorcab. Ramlal Cabs Pvt. Ltd.

has raised an invoice for Rs. 1,15,000 dated 25.10.2016 (value of services is Rs.

1,00,000 and service tax payable thereon [including Swachh Bharat Cess (SBC) and

Krishi Kalyan Cess (KKC is Rs. 15,000)] on Tingtong Travels Pvt. Ltd. for the services

sub-contracted to it during the month of October, 2016. Tingtong Travels Pvt. Ltd.

has received the invoice raised by Ramlal Cabs Pvt. Ltd. on 25.10.2016.

Determine service tax (including SBC & KKC) payable in cash by Tingtong Travels

Pvt. Ltd. for the month of October, 2016 assuming that Tingtong Travels Pvt. Ltd.

avails the abatement, if any, available to it under Notification No. 26/2012 –ST dated

20.06.2012. (8 Marks)

(c) State in brief policy for import of samples. (4 Marks)

Page 158: Test Series: August, 2017 MOCK TEST PAPER 1 FINAL COURSE ... · The company incurred losses of Rs. 40,000 and Rs. 60,000 in 2015-2016 and 2016-2017 after charging depreciation of

1

Test Series: August, 2017

MOCK TEST PAPER - 1

FINAL COURSE: GROUP – II

PAPER – 8: INDIRECT TAX LAWS

SUGGESTED ANSWERS / HINTS

1. (a) In order to claim the benefit of exemption under Notification No. 8/2003 C.E. dated

01.03.2003 in a financial year, the aggregate value of clearances of all excisable

goods for home consumption by a manufacturer from one or more factories should

not exceed Rs. 400 lakh in the preceding financial year.

The aggregate value of clearances for home consumption of Hard Manufacturing

Co. is Rs. 440 lakh in the preceding financial year [Refer computation given below].

Therefore, it is not eligible to claim the benefit of exemption under Notification No.

8/2003 C.E. dated 01.03.2003 in the current financial year.

Computation of aggregate value of clearances for home consumption of Hard

Manufacturing Co. for preceding Financial Year

Particulars Rs. (in lakh)

Clearances of excisable goods exempted from payment of duty under a notification other than Notification No. 8/2003 CE

50

Clearances of account books bearing brand name of another person [Note 1]

150

Clearance of excisable goods to United Nations exempted from payment of duty under Notification No. 108/95 CE [Note 2]

Nil

Exports to Bhutan [Note 3] 50

Clearances of goods on which duty has been paid under section 3A of the Central Excise Act

190

Aggregate value of clearances in terms of Notification No. 8/2003 CE 440

Notes:

Notification No. 8/2003 C.E. dated 01.03.2003 provides that while determining the

value of clearances of Rs. 400 lakh:

1. clearances bearing the brand name of another person, which are ineligible for

SSI exemption are excluded. However, account books falling under heading

4820 of the First Schedule of the Central Excise Tariff are entitled to small

scale exemption even if they bear a brand name or trade name whether

registered or not, of another person. Therefore, clearances of such account

Page 159: Test Series: August, 2017 MOCK TEST PAPER 1 FINAL COURSE ... · The company incurred losses of Rs. 40,000 and Rs. 60,000 in 2015-2016 and 2016-2017 after charging depreciation of

2

books will not be excluded.

2. clearances of excisable goods without payment of duty supplied to United

Nations under Notification No.108/95 C.E. are excluded.

3. export turnover is excluded. However, exports to Bhutan are not excluded as

these are treated as “clearance for home consumption”.

(b) (i) Schedule III in the CGST Act is akin to the negative list under the service tax

regime. This schedule specifies transactions/ activities which shall be neither

treated as supply of goods nor a supply of services.

1. Services by an employee to the employer in the course of or in relation to his employment.

2. Services by any court or Tribunal established under any law for the time being in force.

3. (a) Functions performed by the Members of Parliament, Members of State Legislature, Members of Panchayats, Members of Municipalities and Members of other local authorities;

(b) Duties performed by any person who holds any post in pursuance of the provisions of the Constitution in that capacity; or

(c) Duties performed by any person as a Chairperson or a Member or a Director in a body established by the Central Government or a State Government or local authority and who is not deemed as an employee before the commencement of this clause.

4. Services of funeral, burial, crematorium or mortuary including transportation of the deceased.

5. Sale of land and, subject to paragraph 5(b) of Schedule II, sale of building.

6. Actionable claims, other than lottery, betting and gambling.

Explanation - For the purposes of paragraph 2, the term "Court" includes District Court, High Court and Supreme Court.

(ii) Composite supply means a supply made by a taxable person to a recipient and:

comprises two or more taxable supplies of goods or services or both, or any combination thereof.

are naturally bundled and supplied in conjunction with each other, in the ordinary course of business.

This means that in a composite supply, goods or services or both are bundled owing to natural necessities. The elements in a composite supply are dependent on the ‘principal supply’.

Page 160: Test Series: August, 2017 MOCK TEST PAPER 1 FINAL COURSE ... · The company incurred losses of Rs. 40,000 and Rs. 60,000 in 2015-2016 and 2016-2017 after charging depreciation of

3

Determination of tax liability on composite supplies:

A composite supply comprising of two or more supplies, one of which is a

principal supply, shall be treated as a supply of such principal supply.

Mixed supply means:

two or more individual supplies of goods or services, or any combination

thereof, made in conjunction with each other by a taxable person

for a single price where such supply does not constitute a composite

supply.

The individual supplies are independent of each other and are not

naturally bundled

Determination of tax liability on mixed supplies:

A mixed supply comprising of two or more supplies shall be treated as

supply of that particular supply that attracts highest rate of tax.

(c) Computation of customs duty payable by XYZ Industries Ltd.

Particulars Amount

CIF value 6,000 US $

Less: Freight 1,200 US $

Less: Insurance 1,800 US $

FOB value 3,000 US $

Add: Freight (20% of FOB value) [Note 1] 600 US $

Add: Insurance (actual) 1,800 US $

CIF 5,400 US $

Add: 1% for landing charges [Note 2] 54 US$

Value in dollars 5,454 US $

Exchange rate as per CBEC [Note 3] Rs. 60 per US $

Assessable value = Rs. 60 x 5,454 US $ Rs. 3,27,240

Basic customs duty @ 10% Rs. 32,724

Sub-total Rs. 3,59,964

Additional duty of customs u/s 3(1) of the Customs Tariff Act (CVD) @ 12.5% of Rs. 3,59,964 (rounded off) [Note 5]

Rs. 44,996

Education cesses 3% on [Rs. 32,724 + Rs. 44,996] (rounded off)

Rs. 2,332

Total customs duty payable [Rs. 32,724 + Rs. 44,996 + Rs. 2,332]

Rs. 80,052

Page 161: Test Series: August, 2017 MOCK TEST PAPER 1 FINAL COURSE ... · The company incurred losses of Rs. 40,000 and Rs. 60,000 in 2015-2016 and 2016-2017 after charging depreciation of

4

Notes:

1. If the goods are imported by air, the freight cannot exceed 20% of FOB price

[Second proviso to rule 10(2) of the Customs (Determination of Value of

Imported Goods) Rules, 2007].

2. Even if there is no information regarding landing charges, still they are charged

@ 1% of CIF value [Clause (ii) of first proviso to rule 10(2) of the Customs

(Determination of Value of Imported Goods) Rules, 2007].

3. Rate of exchange determined by CBEC is considered [Clause (a) of the

explanation to section 14 of the Customs Act, 1962].

4. Rule 10(1)(b)(iv) of the Customs Valuation (Determination of Value of Imported

Goods) Rules, 2007 inter alia provides that value of development work

undertaken elsewhere than in India is includible in the value of the imported

goods. Thus, development charges of Rs. 56,000 paid for work done in India

have not been included for the purposes of arriving at the assessable value.

5. Since excise duty rate on smiliar goods is 12.5%, CVD will be levied @ 12.5%.

Rs. 44,996 paid as additional duty of customs under section 3(1) of Customs Tariff

Act, 1975 will be available as CENVAT credit and can be utilised for payment of

excise duty or service tax as per the CENVAT Credit Rules, 2004. Basic customs

duty of Rs. 32,724 and education cesses of Rs. 2,332 paid on imported goods will

not be available as CENVAT credit.

2. (a) Where on a representation from the Principal Commissioner of Central Excise or

Commissioner of Central Excise or otherwise, it is proved that an advance ruling

was obtained by the concerned applicant by fraud or misrepresentation of facts, the

authority for advance ruling can declare the advance ruling as void ab initio.

Consequently, all the provisions of the Central Excise Act shall apply to the

applicant as if such ruling had never been made. Copy of such order is sent to the

applicant and the Principal Commissioner of Central Excise or Commissioner of

Central Excise [Section 23F of the Central Excise Act, 1944].

(b) Computation of interest payable under section 75 of Finance Act, 1994

Particulars

Service tax paid belatedly Rs. 60,000

No. of days of delay [Note 1] 24 days

Rate of interest [Note 2] 12%

Quantum of interest (rounded off) [Rs. 60,000 x 24/365 x 12/100]

Rs. 473

Page 162: Test Series: August, 2017 MOCK TEST PAPER 1 FINAL COURSE ... · The company incurred losses of Rs. 40,000 and Rs. 60,000 in 2015-2016 and 2016-2017 after charging depreciation of

5

Notes:

1. As per second proviso to rule 6(1) of Service Tax Rules, 1994, the due date of

payment of service tax for the month of March is 31st day of March. Therefore,

there is a delay of 24 days as service tax is deposited on 24 th April, 2017.

2. As per section 75 of Finance Act, 1994 read with Notification No. 13/2016 ST

dated 01.03.2016 in case of collection of any amount as service tax but failing

to pay the amount so collected to the credit of the Central Government on or

before the date on which such payment becomes due, the simple interest

@ 24% p.a. is payable. However, in all other cases, 15% simple interest p .a. is

payable. Since in the above case, service tax has not been collected, so

simple interest @ 15% p.a. is payable. However, as per proviso to section 75,

the rate of interest gets reduced by 3% if the value of taxable service provided

during the preceding financial year does not exceed Rs. 60 lakh. (Value of

taxable services provided in preceding year in the given case is Rs. 12 lakh).

(c) The provisions of the Customs Act, 1962 relating to payment of interest in case of

provisional assessment are as under:

(I) Interest payable by the importer/exporter on amount payable to the

Central Government, consequent to the final assessment/re-assessment:

(i) The importer or exporter shall be liable to pay interest, on any amount payable to the Central Government, consequent to the final assessment order/re-assessment order under section 18(2).

(ii) The interest shall be payable at the rate prescribed under section 28AA of the Customs Act, 1962. Presently, the rate of interest has been fixed @ 15% p.a.

(iii) The interest shall be payable from the first day of the month in which the duty is provisionally assessed till the date of payment thereof.

(II) Interest payable by the Central Government to the importer/exporter on amount refundable to the importer/exporter on final assessment of duty/re-assessment of duty:

(i) Subject to the provisions of unjust enrichment, if any refundable amount is not refunded to the importer/exporter on final assessment of duty or re-assessment of duty, within three months from the date of final assessment of duty or re-assessment of duty.

(ii) The interest shall be payable at the rate prescribed under section 27A of the Customs Act, 1962. Presently, the rate of interest has been fixed @ 6% p.a.

(iii) The interest shall be payable from the first day immediately succeeding the period of three months from the date of assessment of duty finally or re-assessment of duty till the date of refund of such amount.

Page 163: Test Series: August, 2017 MOCK TEST PAPER 1 FINAL COURSE ... · The company incurred losses of Rs. 40,000 and Rs. 60,000 in 2015-2016 and 2016-2017 after charging depreciation of

6

3. (a) The activity of producing mustard oil and oil cake from mustard seeds amounts to

manufacture. This particular issue has been decided by the Supreme Court in the

case of Jai Bhagwan Oil and Floor Mills v. UOI 2009 (239) ELT 401 (SC) . In the

instant case, the Apex Court held that the true test to ascertain whether a process is

a manufacturing process producing a new and distinct article is whether the article

produced is regarded in the trade, by those who deal in it, as a marketable product

distinct in identity from the commodity/raw material involved in the manufacture.

When mustard seeds were subjected to the process of extraction whereby mustard

oil and oil cake were produced, the process involved manufacture of mustard oil as

also the manufacture of oil cake. It was certainly not a mere process of cleaning,

repairing, reconditioning, recycling or assembling. Oil cake had a distinct and

different identity from mustard seeds and it had a separate name, character and use

different from mustard seed. Oil cake was not a waste to be thrown away, but was

a valuable product with a distinct name, character, use and marketability.

Resultantly, it can be concluded that the said process amounts to manufacture.

(b) As per rule 5 of Central Excise (Appeals) Rules, 2001, an appellant is permitted,

under certain specified circumstances, to produce before the Commissioner

(Appeals) any evidence other than the evidence produced by him during the course

of the proceedings before the adjudicating authority .

The High Court, in Utkarsh Corporate Services v. CEx. & ST 2014 (34) STR 35

(Guj.), has held that when production of additional evidence is permissible, raising

of additional grounds on the basis of relevant facts existing on record is also

permissible. Further, legal grounds can be raised at any stage before any authority.

Therefore, in view of the fact that the additional grounds raised by Protection

Services before Commissioner (Appeals) were legal grounds, the Commissioner

(Appeals) was not justified in denying the raising of such additional grounds.

(c) Yes, the contention of the assessee is justified in law. The facts of the given case

are similar to the case of Atherton Engineering Co. Pvt. Ltd. v. UOI 2010 (256) ELT

358 (Cal.). In the instant case, the Court noted that it was the use of the product

that had to be considered in the instant case. If a product undergoes some change

after importation till the time it is actually used, it is immaterial, provided it remains

the same product and it is used for the purpose specified in the classification.

Therefore, in the instant case, it examined whether the nature and character of the

product remained the same.

The Court opined that if the embryo within the egg was incubated in controlled temperature and under hydration, a larva was born. The larva did not assume the character of any different product. Its nature and characteristics were same as the product or organism which was within the egg.

Page 164: Test Series: August, 2017 MOCK TEST PAPER 1 FINAL COURSE ... · The company incurred losses of Rs. 40,000 and Rs. 60,000 in 2015-2016 and 2016-2017 after charging depreciation of

7

Hence, the Court held that the said product should be classified as feeding materials for prawns under the heading 2309. These embryos might not be proper prawn feed at the time of importation but could become so, after incubation.

4. (a) As per rule 4 of Central Excise (Determination of Retail Sale Price of Excisable

Goods) Rules, 2008, where a manufacturer removes the excisable goods without

declaring the retail sale price (RSP) on the package then, the RSP of such goods

will be ascertained in the following manner, namely:

(i) if the manufacturer has manufactured and removed identical goods, within a

period of one month, before or after removal of such goods, by declaring the

RSP, then, the said declared RSP will be taken as the RSP of such goods;

(ii) if the RSP cannot be ascertained in the above manner, the RSP of such goods

will be ascertained by conducting the enquiries in the retail market on sample

basis where such goods have normally been sold at or about the same time of

the removal of such goods.

If more than one RSP is ascertained, then the highest of the RSP, so

ascertained, will be taken as the RSP of all such goods.

(b) Computation of service tax payable by ‘Green Agro Unit’ for June, 2016

Sl. No.

Particulars Amount (Rs.)

(i) Supply of farm labour [Note 1] -

(ii) Warehousing of biscuits [Note 4] 1,65,000.00

(iii) Sale of rice on commission basis [Note 2] 68,000.00

(iv) Training of farmers on use of new pesticides and fertilizers developed through scientific research [Note 1]

-

(v) Renting of vacant land to a stud farm [Note 3] 1,31,500.00

(vi) Testing undertaken for soil of a farm land [Note 1] -

(vii) Leasing of vacant land to a dairy farm [Note 3] -

Total 3,64,500.00

Service tax @ 14% 51,030.00

Add: SBC [Rs. 3,64,500 × 0.5%]

KKC [Rs. 3,64,500 × 0.5%]

Total service tax payable including cesses

1,822.50

1,822.50

54,675.00

Notes:

(1) Clause (d) of negative list of services [section 66D] covers services relating to

agriculture or agricultural produce by way of inter alia –

(i) supply of farm labour

Page 165: Test Series: August, 2017 MOCK TEST PAPER 1 FINAL COURSE ... · The company incurred losses of Rs. 40,000 and Rs. 60,000 in 2015-2016 and 2016-2017 after charging depreciation of

8

(ii) agricultural extension services. Agriculture extension means application

of scientific research and knowledge to agricultural practices through

farmer education or training.

(iii) agricultural operations directly related to production of any agricultural

produce including testing.

(2) Services relating to agriculture or agricultural produce by way of inter alia

services provided by a commission agent for sale or purchase of agricultural

produce are covered under clause (d) of section 66D. However CBEC vide

Circular No.177/03/2014 ST dated 17.02.2014 , has clarified that the definition

of agricultural produce under section 65(5) of the Finance Act, 1994 covers

‘paddy’; but excludes ‘rice’. Hence, sale of rice on commission basis will be

taxable.

(3) Services relating to agriculture or agricultural produce by way of renting or

leasing of vacant land are covered under clause (d) of section 66D.

Agriculture means the cultivation of plants and rearing of all life-forms of

animals, except the rearing of horses, for food, fibre, fuel, raw material or other

similar products. Thus, leasing of vacant land to a dairy farm will be included

in the negative list but renting of vacant land to a stud farm will be outside the

purview of negative list.

(4) Loading, unloading, packing, storage or warehousing of agricultural produce is

covered under clause (d) of Section 66D. However, agricultural produce

means any produce of agriculture on which either no further processing is

done or such processing is done as is usually done by a cultivator or producer

which does not alter its essential characteristics but makes it marketable for

primary market. Thus, warehousing of biscuit will be taxable as biscuit is not

an agricultural produce.

(5) As ‘Green Agro Unit’ has paid service tax of Rs. 6,18,000 during the preceding

financial year, it is not eligible to small service providers exemption provided

under Notification No. 33/2012 ST dated 20.06.2012 in the current financial

year.

(c) As per section 9A(1A) of the Customs Tariff Act, 1975, following are the ways that

would constitute circumvention (avoiding levy of duty by unscrupulous means) of

antidumping duty imposed on an article that may warrant action by the Central

Government:

(i) altering the description or name or composition of the article subject to such

anti-dumping duty,

(ii) import of such article in an unassembled or disassembled form,

(iii) changing the country of its origin or export, or

Page 166: Test Series: August, 2017 MOCK TEST PAPER 1 FINAL COURSE ... · The company incurred losses of Rs. 40,000 and Rs. 60,000 in 2015-2016 and 2016-2017 after charging depreciation of

9

(iv) any other manner, whereby the anti-dumping duty so imposed is rendered

ineffective.

In such cases, investigation can be carried out by Central Government and then anti

dumping duty can be imposed on such articles.

5. (a) As per rule 16 of the Central Excise Rules, 2002, when duty paid goods are

returned to the factory for being re-made, refined etc., the assessee can avail and

utilize CENVAT credit of the duty paid if he states the particulars of such return in

his records.

If the duty paid goods returned to the factory are subjected to a process which:

(i) does not amount to manufacture, the manufacturer shall pay an amount equal

to the CENVAT credit taken at the time when such goods are returned. Such

amount shall be allowed as CENVAT credit as if it was a duty paid by the

manufacturer who removes the goods.

(ii) amounts to manufacture, the manufacturer shall pay duty on such returned

goods at the rate applicable on the date of removal and on the value as

determined under the relevant provisions.

(b) (i) As per rule 12 of PoPS Rules, the place of provision of services provided on

board a conveyance during the course of a passenger transport operation,

including services intended to be wholly or substantially consumed while on

board, shall be the first scheduled point of departure of that conveyance for the

journey. Hence, in this case the place of provision of this service will be

Singapore, which is outside the taxable territory.

However, if the above service is provided on a Delhi-Bangalore-Singapore-

Malaysia flight during the Singapore-Malaysia leg, then the place of provision

of this service will be Delhi, which is in the taxable territory.

(ii) Leviability of service tax is determined in terms of the provisions of Finance

Act, 1994 and not in terms of Income-tax Act, 1961. The fact that Mr. Diljeet is

a non-resident is irrelevant for determining the taxability of services received

by him.

As per section 66B of Finance Act, 1994, service tax is levied on the value of

all services, other than those services specified in the negative list, provided or

agreed to be provided in the taxable territory by one person to another.

As per rule 9 of PoPS Rules, the place of provision of services provided by a

banking company, or a financial institution, or a non-banking financial

company, to account holders is the location of the service provider.

Account has been defined under rule 2(b) of PoPS Rules to mean an account

bearing interest to the depositor, and includes a non-resident external account

Page 167: Test Series: August, 2017 MOCK TEST PAPER 1 FINAL COURSE ... · The company incurred losses of Rs. 40,000 and Rs. 60,000 in 2015-2016 and 2016-2017 after charging depreciation of

10

and a non-resident ordinary account. Services linked to or requiring opening

and operation of bank accounts such as lending, deposits, safe deposit locker

etc. are few examples of services that are provided by a banking company or

financial institution to an “account holder” in the ordinary course of business.

Since, in the present case, services (safe deposit locker) are provided by

Punjab Branch of Safe and Sound Bank to an account holder (Mr. Diljeet), rule

9 of PoPS Rules will apply. Thus, the place of provision of service would be

Punjab and since Punjab falls in taxable territory, locker fee would be liable to

service tax.

(c) The first proviso to section 129A of the Customs Act, 1962 provides that the

Appellate Tribunal shall not have jurisdiction to decide any appeal in respect of any

order passed by Commissioner (Appeals) i f such order relates to, -

(a) any goods imported or exported as baggage;

(b) any goods loaded in a conveyance for importation into India, but which are not

unloaded at their place of destination in India, or which are short landed at that

destination;

(c) payment of drawback as provided in Chapter X, and the rules made

thereunder.

In such cases, redressal lies with the Central Government. Section 129DD(1)

enables the appellant to get the orders of Appellate Commissioner (on these three

aspects) annulled or modified by the Central Government.

By virtue of section 129DD(1A) the Principal Commissioner of Customs or

Commissioner of Customs may direct the proper officer to make an application on

his behalf to the Central Government for revision of an order. However, such

application can be made only if the Principal Commissioner of Customs or

Commissioner of Customs is of the opinion that the order passed by the

Commissioner (Appeals) under section 128A is not legal or proper.

Therefore, while an importer or exporter (i.e., an assessee) can file a review

application against the orders of Commissioner (Appeals) under section 129DD(1),

the departmental review application is filed under section 129DD(1A).

6. (a) Bonds may be accepted by any of the following officers:

(i) Deputy/Assistant Commissioner of Central Excise having jurisdiction over the

factory or warehouse or any other premises approved by the Principal

Commissioner/Commissioner for storing non-duty paid goods

(ii) Jurisdictional Maritime Commissioner in case of exports or

(iii) Deputy/Assistant Commissioner of Central Excise (Export) as officers

authorized by the Board for this purpose.

Page 168: Test Series: August, 2017 MOCK TEST PAPER 1 FINAL COURSE ... · The company incurred losses of Rs. 40,000 and Rs. 60,000 in 2015-2016 and 2016-2017 after charging depreciation of

11

Exporters are required to indicate on the ARE-1 the complete postal address of the

authority before whom the bond is executed and to whom the documents are to be

submitted for admission of proof of export.

Or

If at any stage of enquiry, investigation or any other proceedings before him, any

Central Excise Officer not below the rank of an Assistant/Deputy Commissioner of

Central Excise having regard to the nature and complexity of the case and the

interest of revenue, is of the opinion that the value has not been correctly declared

or determined by a manufacturer or any person, he may, with the previous approval

of the Principal Chief Commissioner/ Chief Commissioner of Central Excise, direct

such manufacturer of such person to get the accounts of his factory, offices, depots,

distributors or any other place, as may be specified by the said Central Excise

officer, audited by a Cost/Chartered Accountant, nominated by the Principal Chief

Commissioner / Chief Commissioner of Central Excise in this behalf.

The Cost/Chartered Accountant shall submit the audit report duly signed and

certified to the said Central Excise Officer within the period specified by him or the

period further extended on an application made to him. However, in any case the

aggregate of the original and extended period cannot exceed 180 days. The

expenses of audit and audit fees for special audit shall be paid by excise

department.

The manufacturer shall be given an opportunity of being heard in respect of any

material gathered on the basis of audit and proposed to be utilized in any

proceedings under the Central Excise Act or Rules. This special audit can be

conducted notwithstanding that the accounts of the manufacturer or the person

have been audited under any other law for the time being in force or otherwise.

(b) In case of taxable services provided by any person which is located in a non-taxable

territory and received by any person located in the taxable territory, person liable to

pay service tax is the recipient of such service [Notification No. 30/2012 ST dated

20.06.2012]. Hence, in the given case, since the business support service has

been provided by Green Inc (located in non-taxable territory) and has been received

by Galaxy Ltd. (located in the taxable territory), Galaxy Ltd. is liable to pay service

tax (i.e., under reverse charge mechanism).

The point of taxation in respect of persons required to pay tax under the reverse

charge mechanism, is the date on which payment is made. However, where the

payment is not made within a period of 3 months of the date of invoice, the point of

taxation will be the date immediately following the said period of three months. [Rule

7 of the Point of Taxation of Rules, 2011].

Page 169: Test Series: August, 2017 MOCK TEST PAPER 1 FINAL COURSE ... · The company incurred losses of Rs. 40,000 and Rs. 60,000 in 2015-2016 and 2016-2017 after charging depreciation of

12

In view of the aforesaid provisions, the point of taxation in each of the given cases

will be as under:

CASE Point of taxation

CASE I Since the importing company i.e. Galaxy Ltd makes the payment within the three months from the date of invoice, the point of taxation will be date of payment i.e. 22.02.2016

CASE II As Galaxy Ltd. makes the payment after three months from the date of invoice, point of taxation will be the date immediately following the said period of three months.

Thus, point of taxation is 19.04.2016.

(c) As per rule 5 of the Baggage Rules, 2016, a passenger who has been residing

abroad for more than one year and returns to India shall be allowed duty free

clearance of jewellery in bona fide baggage as under:

Jewellery upto a weight of 20 grams with a value cap of Rs. 50,000 for a

gentlemen passenger

Jewellery upto a weight of 40 grams with a value cap of Rs. 1,00,000 for a lady

passenger

Thus, in the given case, Mr. Vipin would be allowed duty free jewellery upto a

weight of 20 grams with a value cap of Rs. 50,000 and his wife would be allowed

duty free jewellery upto a weight of 40 grams with a value cap of Rs. 1,00,000.

Further, in addition to the jewellery allowance, Mr. Vipin and his wife would also be

allowed duty free clearance of jewellery worth Rs. 1,00,000 (Rs. 50,000 per person)

as part of free baggage allowance.

7. (a) Remission of duty is granted when the goods are lost or destroyed by natural

causes or by unavoidable accident or are claimed by the manufacturer as unfit for

consumption, at any time before removal.

Hence, in this case remission of duty cannot be granted under rule 21 of the Central

Excise Rules, 2002 as goods have already been cleared from the factory after

payment of duty.

(b) Computation of value of taxable services of Tingtong Travels Pvt. Ltd. for

October, 2016

Particulars (Rs.)

Amount charged for the services 6,00,000

Value of taxable service @ 40% of the amount charged for the service [Note 1]

2,40,000

Page 170: Test Series: August, 2017 MOCK TEST PAPER 1 FINAL COURSE ... · The company incurred losses of Rs. 40,000 and Rs. 60,000 in 2015-2016 and 2016-2017 after charging depreciation of

13

Computation of service tax (including cesses) payable in cash by Tingtong

Travels Pvt. Ltd. for October, 2016

Particulars Service tax (Rs.)

SBC

(Rs.)

KKC

(Rs.)

Service tax, SBC and KKC 33,600 1,200 1,200

Less: CENVAT credit [Note 2] 5,600 - 200

28,000 1,200 1,000

Service tax (including cesses) payable in cash is Rs. 30,200 (Rs. 28,000 +

Rs. 1,200 + Rs. 1,000)

Notes:

1. Notification No. 26/2012 ST provides that value of taxable services in respect

of services of renting of motor cabs is 40% of the amount charged by the

service provider. In other words, an abatement of 60% of the amount charged

is available in respect of services of renting of motorcab.

2. Notification No. 26/2012 ST provides that up to 40% CENVAT credit of input

service of renting of a motorcab provided by a sub-contractor to the main

contractor (providing service of renting of motorcab) could be availed by the

main contractor if the sub-contractor is paying service tax on full value i.e., no

abatement is being availed by sub-contractor. This credit will be available

even if the main contractor pays the service tax on abated value.

Since Ramlal Cabs Pvt. Ltd. has paid service tax on full value (Rs. 1,00,000 x

15% = Rs. 15,000), TingtongTravels Pvt. Ltd. can avail credit upto 40% of

CENVAT credit of input service [service tax and KKC] as SBC is not

CENVATable.

3. Since TingtongTravels Pvt. Ltd. is a company, reverse charge provisions will

not apply in its case. Further, provisions of partial reverse charge will not

apply in case of Ramlal Cabs Pvt. Ltd. also, as in its case services are

provided in similar line of business.

(c) No authorisation is required for import of bona fide technical and trade samples.

These are importable freely. Samples upto Rs. 3,00,000 can be imported by all

exporters without duty.

Authorisation for import of samples is required only in case of vegetable seeds,

bees and new drugs. Samples of tea upto Rs. 2,000 (CIF) per consignment will be

allowed without authorization.